You are on page 1of 154

BEST SELLER For 2024 Exam

GENERAL TEST
Section III (Compulsory)

Strictly as per the Latest Examination


Pattern issued by NTA

The ONLY book you need to Ace CUET (UG)

1 2 3 4 5
Valuable 100% Exam Extensive Concept Final
Exam Insights Readiness Practice Clarity Boost
With Latest Solved With 10 Solved With 650+ Latest Learn Key Concepts With Tips & Tricks
Papers 2023 Sample Question Typologies Questions through Detailed to ace CUET (UG)
Papers Explanations in 1st Attempt
(1)
3rd EDITION YEAR 2024
ISBN “9789357288729”

SYLLABUS CUET (UG) General Test


COVERED
PUBLISHED BY
OSWAAL BOOKS &
LEARNING PVT. LTD.
C OPYRIG HT
RESERVED 1/11, Sahitya Kunj, M.G. Road,
BY THE PUBLISHERS Agra - 282002, (UP) India
All rights reserved. No part of
this book may be reproduced,
stored in a retrieval system, 1010, Cambourne Business Centre
or transmitted, in any form or Cambridge, Cambridgeshire
by any means, without written
permission from the publishers. CB 236DP, United kingdom
The author and publisher will
gladly receive information
enabling them to rectify any
error or omission in subsequent 0562-2857671
editions.

contact@oswaalbooks.com

www.OswaalBooks.com

DI SC L A IMER
This book is published by Oswaal Books and Learning Pvt Ltd (“Publisher”) and is intended solely for educational use, to
enable students to practice for examinations/tests and reference. The contents of this book primarily comprise a collection
of questions that have been sourced from previous examination papers. Any practice questions and/or notes included by
the Publisher are formulated by placing reliance on previous question papers and are in keeping with the format/pattern/
guidelines applicable to such papers.
The Publisher expressly disclaims any liability for the use of, or references to, any terms or terminology in the book, which
may not be considered appropriate or may be considered offensive, in light of societal changes. Further, the contents of
this book, including references to any persons, corporations, brands, political parties, incidents, historical events and/or
terminology within the book, if any, are not intended to be offensive, and/or to hurt, insult or defame any person (whether
living or dead), entity, gender, caste, religion, race, etc. and any interpretation to this effect is unintended and purely
incidental. While we try to keep our publications as updated and accurate as possible, human error may creep in. We
expressly disclaim liability for errors and/or omissions in the content, if any, and further disclaim any liability for any loss or
damages in connection with the use of the book and reference to its contents”.
Kindle
( 2Edition
)
Preface
The National Testing Agency took over the responsibility for conducting CUET in 2021. The Common University
Entrance Test (CUET) had been introduced as the assessment test for admission to UG programs across all Central
Universities. This has been done to ensure that students across the country, whether from urban areas or rural/
remote areas have access to the same opportunities/environment for assessment. As an all-India test conducted
by the National Testing Agency (NTA), it provides a single-window opportunity for admissions, and streamlines the
admissions process, and standardizes the evaluation process.

A full Computer Based Test (CBT), the question paper is divided into four sections: Sections IA & IB contain
language-specific questions, Section II comprises domain questions, and Section III contains questions on general
topics. All questions are MCQ based.

The curriculum for CUET is based on the National Council of Educational Research and Training (NCERT) syllabus
for class 12 only. CUET (UG) scores are mandatory required while admitting students to undergraduate courses in 44
central universities.

A merit list will be prepared by participating Universities/organizations. Universities may conduct their
individual counselling on the basis of the scorecard of CUET (UG) provided by NTA.

Almost 1.92 million candidates registered for CUET (UG) in 2023. Candidates have been quite anxious about
appearing for CUET (UG), however, with the right preparation strategy and resources, you can secure a good rank in
CUET (UG). This book is a step in that direction, helping candidates refine their examination strategy, and bridging
the gap between their present preparation level and a rank-securing preparation level.

A merit list will be prepared by participating Universities/organizations. Universities may conduct


their individual counselling on the basis of the scorecard of CUET (UG) provided by NTA.

A few benefits of studying from Oswaal Mock Test Papers


• Valuable Exam Insight With Latest Solved Papers 2023 & its Explanations
• 100% Exam Readiness With 10 Mock Test Papers Fully Solved
• Extensive Practice With 650+ Latest Typologies Questions
• Concept Clarity Learn Key Concepts through Detailed Explanations
• Final Boost With Tips & Tricks to ace CUET (UG) in 1st Attempt

Our Heartfelt Gratitude!


Finally, we would like to thank our authors, editors, and reviewers. Special thanks to our students who
send us suggestions and constantly help improve our books. We promise to always strive towards
‘Making Learning Simple’ for all of you.
Wish you all Happy Learning!
-Team Oswaal Books

(3)
Contents
l Oswaal Books Expert Tips to Crack CUET (UG) in the First Attempt 5-5
l Examination Structure for CUET (UG) & Syllabus 6-8

l CUET (UG) Question Paper-2023 - 21st May 2023 Shift-1 1 - 13

Mock Test Papers


l Mock Test Paper - 1 14 - 18
l Mock Test Paper - 2 19 - 23
l Mock Test Paper - 3 24 - 28
l Mock Test Paper - 4 29 - 33
l Mock Test Paper - 5 34 - 38
l Mock Test Paper - 6 39 - 43
l Mock Test Paper - 7 44 - 48
l Mock Test Paper - 8 49 - 53
l Mock Test Paper - 9 54 - 58
l Mock Test Paper - 10 59 - 63

Solutions
l Mock Test Paper - 1 64 - 71
l Mock Test Paper - 2 72 - 79
l Mock Test Paper - 3 80 - 88
l Mock Test Paper - 4 89 - 96
l Mock Test Paper - 5 97 - 104
l Mock Test Paper - 6 105 - 112
l Mock Test Paper - 7 113 - 120
l Mock Test Paper - 8 121 - 128
l Mock Test Paper - 9 129 - 136
l Mock Test Paper - 10 137 - 143
 

(4)
Oswaal
Oswaal Books ExpertBooks Expert
Tips to Crack Tips
CUET toinCrack
(UG) the First empt
CUET (UG) in the First Attempt
Excited about your UG but unsure if you will get admission to your preferred university?
In a major announcement by the chairman of the University Grants Commission, the N onal Te ng Agency will be
conduc g the Common Univers es Entrance Test (CUET (UG) ) for undergraduate programs in Central
Univers es for the upcoming academic session. However, the UGC Chairperson also stated that CUET (UG) will not
just be limited to admissions to Central Univers es. Many prominent private univers es have indicated tha hey
would also like to adopt a common entrance exam for undergraduate admissions and take admissions on the basis of
CUET (UG) scores.
This makes CUET (UG) a very important examina n in itself and hence it becomes mandatory to be aware o he s
& tricks that could help you ace the exam on the first empt.

Make a habit of preparing notes


f ro m t h e b e g i n n i n g o f t h e
prepar on. It will not only help in
The first step is to understand making the study syst c but
The ern of the ex on. also make the revision o he
CUET includes three ons, While preparing for the exam, i s
syllabus easy even when you
on 1 includes que on important to iden y the
might have limited e to revise.
based on languages, sec n 2 importan opics and prac e
includes 27 domain-based important ques ons from those
subjects and sec n 3 includes topics. Pra ce important
General Test. The syllabus of the que ons through Oswaal
upcoming Common University Ques n Bank and Sample Collec g and preparing from the
Entrance Test, CUET is Ques n Papers, Li ng appropriate study material cannot be
completely based on the topics also helps in iden ying the ignored as irrelevant. The books chosen
syllabus of class 12 th . No weak areas that need special by the aspirants to study from should be
ques n will be asked from effort and e. The aspirants can on the lines of the current syllabus and
class 11th syllabus. start preparing to focus on the the ones that could help you with
areas tha hey consider to be revision before the ex on.
tough, followed by the ones that
are their strengths.

Make sure to revise as much as possible. The


Devote a sufficient amount o me to all the
revision will help the aspirants in keeping the
ons of the ex ons. This requires a well-
concepts fresh in their minds un the day of the
made plan and an honest adherence to the said
final ex ons. They may refer to a few good
plan. Priori e the most importan opics or the
pr ce ques ons and concise revision notes to
topics that the aspirants are not familiar with to be
achieve their desired results.
able to master them in e.

With this said, an important ques n tha s gaining ground amongst students who will be appearing for this exam is
if they should take coaching to get themselves ready for the exams. The answer is a simple no, the exam will simply
not require any coaching as it is completely based on the Class 12th syllabus which will be quite fresh in students'
minds as they will be just out of school. All they need is a good revision and prac e of ques ons from Oswaal
Ques n Bank and Sample Ques n Papers for CUET (UG) prepar ons.
1712

(5)
Examination Structure for CUET (UG)

CUET (UG) – is consisted of the following 4 Sections:


 Section IA – 13 Languages
 Section IB – 20 Languages
 Section II – 27 Domain specific Subjects
 Section III – General Test
Choosing options from each Section is not mandatory. Choices should match the requirements of the desired University.
Broad features of CUET (UG) - are as follows:

Questions to be
Section Subjects/ Tests Question Type Duration
Attempted
Section IA - There are 13* different languages. 40 questions to be Language to be tested 45 Minutes
Languages Any of these languages may be attempted out of 50 through Reading for each
chosen. in each language Comprehension (based language
on different types of
Section IB - There are 20** Languages. passages–Factual, Literary
Languages Any other language apart from those and Narrative, [Literary
offered in Section I A may be chosen. Aptitude and Vocabulary]
Section II - There are 27*** Domains specific sub- 40 Questions to be • Input text can be 45 Minutes
Domain jects being offered under this Section. attempted out of 50 used for MCQ Based for each
A candidate may choose a maximum Questions Domain
of Six (06) Domains as desired by the • MCQs based on NCERT Specific
applicable University/Universities. Class XII syllabus only Subjects.
• Input text can be
used for MCQ Based
Questions
• General Knowledge,
Current Affairs,
General Mental Ability,
For any such undergraduate pro-
Section III - Numerical Ability,
gramme/ programmes being offered by 50 Questions to be
General Quantitative Reasoning 60 Minutes
Universities where a General Test is attempted out of 60
Test (Simple application
being used for admission.
of basic mathematical
concepts arithmetic/
algebra geometry/
ensuration/s tat taught
till Grade 8), Logical and
Analytical Reasoning

* Languages (13): Tamil, Telugu, Kannada, Malayalam, Marathi, Gujarati, Odiya, Bengali, Assamese, Punjabi,
English, Hindi and Urdu
** Languages (20): French, Spanish, German, Nepali, Persian, Italian, Arabic, Sindhi, Kashmiri, Konkani, Bodo, Dogri,
Maithili, Manipuri, Santhali, Tibetan, Japanese, Russian, Chinese.
*** Domain Specific Subjects (27): 1.Accountancy/ Book Keeping 2.Biology/ Biological Studies/ Biotechnology/Biochemistry
3.Business Studies 4.Chemistry 5.Computer Science/ Informatics Practices 6.Economics/ Business Economics
7.Engineering Graphics 8.Entrepreneurship 9.Geography/Geology 10.History 11.Home Science 12.Knowledge
Tradition and Practices of India 13.Legal Studies 14.Environmental Science 15.Mathematics 16.Physical Education/ NCC
/Yoga 17.Physics 18.Political Science 19.Psychology 20.Sociology 21.Teaching Aptitude 22.Agriculture 23. Mass Media/
Mass Communication 24.Anthropology 25.Fine Arts/ Visual Arts (Sculpture/ Painting)/Commercial Arts, 26. Performing
Arts – (i) Dance (Kathak/ Bharatnatyam/ Oddisi/ Kathakali/Kuchipudi/ Manipuri (ii) Drama- Theatre (iii) Music General
(Hindustani/ Carnatic/ Rabindra Sangeet/ Percussion/ Non-Percussion), 27. Sanskrit
• A Candidate can choose a maximum of any 3 languages from Section IA and Section IB taken together.However, the
third language chosen needs to be in lieu of 6th domain specific Subject chosen by the candidate - as applicable (so the
maximum number of tests to be taken remains 9 only i.e. 2 Languages+6 Domain Specific Subjects+1 General Test
OR 3 Languages+5 Domain Specific Subjects+1 General Test: flexibility being provided to help a candidate apply for
many Universities depending on their eligibility conditions).
• Section II offers 27 Subjects, out of which a candidate may choose a maximum of 6 Subjects.
• Section III comprises General Test.
(6)
Contd...

• For choosing Languages from Section IA and IB and domain specific Subjects from Section II and General Test under
Section III, the Candidate must refer to the requirements of his/her intended University.

Mode of the Test Computer Based Test-CBT

Test Pattern Objective type with Multiple Choice Questions

Medium 13 languages (Tamil, Telugu, Kannada, Malayalam, Marathi, Gujarati, Odiya, Bengali, Assamese, Punjabi,
English, Hindi and Urdu)
Section IA & IB: Language to be tested through Reading Comprehension (based on different types
of passages–Factual, Literary and Narrative [Literary Aptitude & Vocabulary]
Section II : As per NCERT model syllabus as applicable to Class XII only
Syllabus
Section III : General Knowledge, Current Affairs, General Mental Ability, Numerical Ability,
Quantitative Reasoning (Simple application of basic mathematical concepts arithmetic/algebra
geometry/mensuration/stat taught till Grade 8), Logical and Analytical Reasoning

Level of questions for CUET (UG) :


All questions in various testing areas will be benchmarked at the level of Class XII only. Students having studied Class XII
Board syllabus would be able to do well in CUET (UG).
Number of attempts:
If any University permits students of previous years of class XII to take admission in the current year also, such students
would also be eligible to appear in CUET (UG).
Choice of Languages and Subjects:
Generally the languages/subjects chosen should be the ones that a student has opted in his latest Class XII Board examination.
However, if any University permits any flexibility in this regards, the same can be exercised under CUET (UG) also. Candidates
must carefully refer to the eligibility requirements of various Central Universities in this regard. Moreover, if the subject to be
studied in the Undergraduate course is not available in the list of 27 Domain Specific Subject being offered, the Candidate may
choose the Subject closest to his/her choice for e.g. For Biochemistry the candidate may choose Biology.

SYLLABUS
GENERAL AWARENESS
1. Indian History → Prehistoric Period, The Harappan Civilisation, The Vedic Period, Jainism and Buddhism, Magadha
Empire, The Mauryan Empire, The Sangam Period, The Gupta Period, Age of Harshavarthana, Rashtrakutas, Gangas,
Pallavs, The Cholas
2. World and Indian Geography
→ 2.1. ( World Geography): Universe, Solar System, Planets, Sun, Earth, Structure of Earth’s Interior, Plate Tectonic
Theory, Volcanism, Rocks, Landforms, Atmosphere, Jet Stream, Oceans, Continents, Important Facts
→ 2.2. ( Indian Geography): India at a Glance, Physical Features, Islands, Drainage System of India, Climate, Agriculture,
Soils of India, Mineral Resources, Transport
3. Indian Polity and Constitution → Constitution, Preamble, Sources of the Constitution, Parts of the Constitution,
Amendability of the Preamble, Schedules of the Constitution, Union and its Territories, Citizenship, Rights, Directive
Principles of State Policy, Union Government, State Government, Local Government, Elections, Planning Commission,
NDC, Finance Commission, Amendments of the Constitution
4. Indian Economy → Economy, Economic Growth, Indian Economy, National Income of India, Planning of India,
India’s New Economic Policy, Flagship Programmes of Government of India, Agriculture, Indian Financial System,
Indian Currency System, Macro Economic, Problems in India.
5. General Science
→ 5.1. ( Physics): Units, Motion, Friction, Gravitation, General Properties of Matter, Heat and Thermodynamics,
Thermal Expansion, Waves, Light, Electricity and Magnetism
→ 5.2. ( Chemistry): Physical and Chemical Changes, Matter; Atom, Molecule and of Element, Corrosion, Renewable and
Non-renewable Natural Resources, Coal, acids, Bases and Salts, Some important Compounds in Everyday life,
Polymers
→ 5.3. ( Biology): Living World, The Cell, Nucleic Acids, Human Systems, Respiratory System, Vitamins, Major
Enzymes of Digestion, Blood (Lymphatic System), Central Nervous System, Some Human Diseases Caused by
Viruses and Bacteria / Fungi, Animal/Human Diseases Caused by Fungi, Important Vaccines Discoverer, Ebola
Virus, Ecology, Pollution, Biotechnology, Some Important Branches of Biology, Some Important Discoveries,
Some Important Antibiotics, Environment and Ecology
6. Computer Science and Technology → Computer, Components of Computer, Memory, Hardware, Software,
Networking, Security Threats, Internet, Some commonly used terms, Super Computer.
7. General Knowledge → Art & Culture, Important Inventions & Discoveries Science & Tech, Awards & Honours, Books
& Authors, Decodes & Dates, Important Organizations, Sports, Miscellaneous.
(7)
Contd...

QUANTITATIVE APTITUDE
1. Number System → Introduction to Numbers; Various types of Numbers; Even numbers; Odd numbers, Prime numbers;
Rational numbers; Irrational number; Tests of divisibility; Standard results; Division algorithm; Arithmetic Progression;
Geometric Progression; Unit place value in the Product of Numbers; Problems on Numbers & H.C.F and L.C.M.
2. Operations → BODMAS Rule; Squares; Cubes; Square Roots; Cube Roots; Fractions; Types of fractions; Decimals;
Surds and Indices; Properties of Surds and Indices.
3. Algebraic Expressions → Constants; Variables; Algebraic expressions; Equations; Linear equation; Quadratic equa-
tions; Factors; Solutions.
→ 3.1. Average and problems on Ages → Average; Properties of Averages; Problems on Ages
→ 3.2. Percentage, Profit-Loss and Discount → Percentage meaning; Fraction representation, Consumption & Expendi-
ture, Reducing & Exceeding prices, Voters in an Election; Percentage increase or increasing and decreasing prob-
lems ; Profit, Loss; Buy-sell problems ; Discounts, Successive discount problems; Tax Miscellaneous Questions.
→ 3.3. Ratio and Proportion, Alligation & partnership → Ratio; Proportion; Properties of Ratio and Proportion; Alli-
gation; Partnership; Active Partner; Sleeping Partner; Types of Partners; Shares and Partners; Coins and Rupees;
Income and Expenditure
→ 3.4. Simple Interest & Compound Interest → Interest; Principal; Rate of Interest; Time; Compound Interest; Difference
between Compound and Simple Interest; Miscellaneous questions on Simple Interest and Compound Interest.
→ 3.5. Time & Work and Pipes & Cisterns → Time and Work; Work and Wages; Efficiency of worker; worker leaves
or joins; Man−day work done; Pipes and Cisterns; Inlet; Outlet; Quantity and Time; Miscellaneous Question.
→ 3.6. Speed, Time and Distance → Speed; Time; Conversion of Units; Average Speed; Speed of Stream; Upstream;
Downstream; Problems on Trains in same direction; Problem on train cross each other in opposite direction;
Relative Speed.
4. Mensuration → Area and Perimeter of triangles, Rhombus, Quadrilaterals, Circles; Volume of 3D figures; Surface area;
Total surface area; Cylinder, Sphere, Hemisphere, Cube, Cuboid, Cone, Frustum of a cone; Standard Results; Equilat-
eral triangle; Parallelogram; Rhombus, Trapezium; Regular Hexagon etc.
5. Geometry → Lines; Angles; Bisectors; Polygons; Coordinate geometry; Types of triangles; Congruency; Similar tri-
angles; Circles; Chord; Tangents; Secant; Cyclic quadrilateral; Circumcircle; Incircle.
6. DATA Interpretation → Data Interpretation; Tables; Bar Graphs; Line Graphs; Pie Charts; Mix Graph.

LOGICAL REASONING
1. Analogy → Alphabet Analogy; Number Analogy; Word Analogy; Analogy based on prime number and mixed opera-
tion based Analogy.
→ 1.1. Alphabet Series → Finding the next or missing alphabet in a series.
→ 1.2. Number Series: N2 series, N2 + 1 series, N2 – 1 series, Cube series, Prime number series, Mixed Operation series etc.
2. Direction and Distance : Concept of direction, Different types of directions, Orientation, Turns, Concept of Shadow,
Finding distance, Finding shortest distance, Different types of questions- Based on turns and rotations, Based on
shadow, Based on finding shortest distance, Based on coded form, Based on seating arrangements, Based on clock
timing indication.
Coding and Decoding : Letter coding, Alphabetical coding, Direct letter coding, Number coding, Deciphering message
coding/Message coding, Substitution coding, Mixed coding and Alpha-Numeric coding.
3. Ranking : Age-Based Comparison, Height Based Comparison, Integrated Height and Age-Based comparison, Linear
seating arrangement comparison. No. of elements between two-element, Position interchanges in a queue, Puzzle
based comparison.
4. Blood Relation : Dialogue/Conversation Based, Puzzles, Coding-Decoding, Linear seating arrangement base blood
relations, Circular seating arrangement based blood relations, Mixed blood relation.
5. Mathematical Operations : Interchanging operators, Symbols, Balancing the equation and Trick based mathematical
operations.
Venn Diagram : Relation Based Venn Diagram, Analysis Based Venn Diagram.
6. Seating Arrangement → Facing towards the center, facing outwards/opposite/outside the center, facing inside and
outside the center Rectangular or square arrangement. Triangular arrangement (facing the centre, facing opposite to
the centre).
7. Syllogism → Positive conclusion, Negative conclusion, Either-OR/Complementary pair, No conclusion.
8. Statement and Conclusion → One statement multiple conclusions, Two or more than two statements, Multiple con-
clusions. Statement and Assumptions, Statement and Course of Action, Cause and Effect, Statement and Arguments,
Inference.
→ 8.1. F
 igure Completion → Based on movement, Based on Rotation 45°/90°/135°/180°, Symmetry of image/ Water and
Mirror Concept, Miscellaneous Concept.
→ 8.2. F igure Formation → Assembling main figure from split figures, Fragmentation of the main figure into pieces.
Making up a figure from given components, Making up a three dimensional figure by paper folding.
→ 8.3. F  igure Counting → Based on triangles, Based on squares, Based on Rhombus, Based on rectangles, Based on
the circle, Based on combined figures, Counting of straight lines.
9. Non-Verbal Series → Figure series, Figure series based on rotation, Based on addition of symbols, Based on deletion of
symbols, Based on replacement, Based on replacement and arrangements, Similar Figure present in questions figures,
Simultaneous operations of rotation, addition and replacement.

(8)
CUET (UG) Question Paper - 2023
National Testing Agency
21st MAY 2023 – SHIFT 1
Section - III (General Test)

Examination Duration: 60 Minutes Maximum Marks - 250

General Instructions:
Marking scheme of the test:
(a) There are 60 questions asked in the section- III. But there are 50 questions to be attempted in the section - III.
(b) Correct answer or the most appropriate answer will be given five marks (+5).
(c) Any incorrect option marked will be given minus one mark (–1).
(d) Unanswered/Marked for review will be given no mark (0).

1. If the word ‘LEADER’ is coded as 20-13-9-12-13-26. How would you write “LIGHT”
(A) 20-16-15-17-22 (B) 20-17-15-16-28 (C) 20-15-16-18-23 (D) 20-16-17-15-27
Ans. Option (B) is the correct.
Explanation:
Logic:
Table of Alphabetical series (point to remember)
Alphabets A B C D E F G H I J K L M

Positional 1 2 3 4 5 6 7 8 9 10 11 12 13
value
Alphabets Z Y X W V U T S R Q P O N

Positional 26 25 24 23 22 21 20 19 18 17 16 15 14
value
L ⇒ 12 + 8 = 20
E ⇒ 5 + 8 = 13
A⇒1+8=9
D ⇒ 4 + 8 = 12
E ⇒ 5 + 8 = 13
R ⇒ 18 + 8 = 26
Therefore, LIGHT can be coded as
L ⇒ 12 + 8 = 20
I ⇒ 9 + 8 = 17
G ⇒ 7 + 8 = 15
H ⇒ 8 + 8 = 16
T ⇒ 20 + 8 = 28
So “LIGHT” will be coded as = 20 - 17 - 15 - 16 - 28.
2. In a class boys stand in a single line. One of the boys 3. If the 2nd half of the letters of the word
is seventeenth in order from both the ends. How INTERMEDIATE are reversed and placed before
many boys are in the class? 1st half of the letters, which letter will be 2nd to the
(A) 34 (B) 33 right of 10th letter from the right?
(A) A (B) D
(C) 32 (D) 27
(C) E (D) I
Ans. Option (B) is correct. Ans. Option (B) is correct.
Explanation: Explanation:
Given: One of the boys is seventeenth in Word: INTERMEDIATE
order from both ends. Reverse 2nd half and placed before 1st half
Hence, ETAIDEINTERM
Total boys in the class = 16 + 1 + 16 = 33 10 - 2 = 8th from right, which is D.
2 OSWAAL CUET (UG ) 10 Mock Test Papers GENERAL TEST
4. Arrange the following in meaningful sequence: Choose the most appropriate answer from the
(A) Key (B) Door options given below:
(C) Lock (D) Room (A) A-I, B-III, C-II, D-IV
Choose the most appropriate answer from the (B) A-II, B-III, C-I, D-IV
options given below: (C) A-III, B-II, C-IV, D-I
(A) A, C, B, D (B) D, C, A, B (D) A-II, B-III, C-IV, D-I
(C) B, A, D, C (D) C, B, D, A Ans. Option (D) is the correct.
Ans. Option (A) is correct. Explanation: Quit India Movement was started
Explanation: by Mahatma Gandhi in which he demanded the
end of British rule in India. This was launched
(A) Key (C) Lock (B) Door (D) Room
at the Bombay session of All India Congress
5. Which of the following is a metal Committee on 8 August 1942. Gandhiji’s famous
slogan of ‘Do or Die’ was the driving force of
(A) Carbon (B) Mercury
this movement. The phrase ‘Quit India’ was
(C) Sulphur (D) Iodine
coined by Yusuf Meherally. However, this
Ans. Option (B) is the correct.
movement was suppressed by British forces.
Explanation: Mercury is a metal. It is a chemical The Civil Disobedience Movement was launched
element represented by the symbol Hg with under the leadership of Mahatma Gandhi in 1930
atomic number 80 and an atomic mass 200.59. It as part of the larger Indian National Congress’s
is a d-block element which is present in period campaign for freedom. He urged Indians to
6 and group 12 of the periodic table. It is the peacefully protest by refusing to cooperate with
only metal liquid at room temperature, which British authorities. Indians boycotted British
is why it is used in thermometers to detect body institutions, refused to pay taxes, and participated
temperature. in acts of non-cooperation. There were mass
Carbon is a non-metal which is represented protests, demonstrations, and acts of civil
by the symbol C. It has an atomic number disobedience taking place throughout the country.
of 6 and an atomic mass of 12. It is a p-block The Salt March or Dandi March was one of the
element belonging to period 2 and group 14 of important events of Civil Disobedience Movement.
the periodic table. Non-Cooperation Movement was a movement
Sulphur is a non-metal which is represented in which Indians united against British colonial
by the symbol S. It has an atomic number of rule with the suspension of cooperation with
16 and an atomic mass of 32. It is a p-block British-run institutions such as schools, colleges,
element belonging to period 3 and group 16 courts, and legislative councils. Indians boycotted
of the periodic table. It is yellow-coloured and British goods and promoted the use of Indian-
crystalline in nature. made products. However, this movement was
Iodine is a non-metal which is represented by suspended in 1922 due to a violence incidence
the symbol I. It is a halogen with an atomic at Chauri-Chaura, Uttar Pradesh.
number 53 and atomic mass 126.9. It is a p-block Home rule league movement was started by
element and belongs to the periodic table’s group Annie Besan and it aimed to achieve self-
17 and period 5. government in India within the British Raj.
She formed the All India Home Rule League
6. Match List I with List II in September 1916. She was the first woman
LIST I LIST II president of the Indian Congress.
Movement Purpose 7. Choose the one which is different from the rest
three.
A. Quit India I. To achieve self
(A) 431 (B) 162
Movement Government in India of
(C) 831 (D) 232
demand for large political
Ans. Option (C) is correct.
representation
B. Civil II. Compelled British’s to Explanation:
Disobedience leave India Logic:
Movement 431  4  3  1  12
C. Non III. Refusal to obey certain 162  1  6  2  12
Cooperation laws orders or Commands 831  8  3  1  24  12
Movement of the Government 232  2  3  2  12
D. Home rule IV. Indians resigning their Hence, we can see that option (C) is not
league title boycotting foreign following the same pattern as others.
Movement goods and Government 8. X got 98 marks in his exam which is 56 % of the total
institutions refused to pay marks. What is the maximum marks of the exam?
taxes (A) 150 (B) 175
(C) 200 (D) 225
CUET(UG) Solved Paper 2023 (21st May Shift -1) 3
Ans. Option (B) is correct. 12. What is the smallest square number which is
divisible by 4,6 and 32?
Explanation:
Let the total marks be x. (A) 100 (B) 196
According to question, (C) 96 (D) 576
56% of x = 98 Ans. Option (D) is correct.
56 Explanation:
x = 98
100 LCM of 4, 6, 32
98  100 =2×2×3×8
x  175
56 = 96
Hence,the maximum marks of the exam is Now find the multiple of 96.
175. 96 is not a perfect square number.
9. Which of the following speeds is the least? Hence, 576 is correct answer.
(A) 50 meter / second 13. The only Indian who received noble prize in
(B) 50 meter/minute literature is.
(C) 70 km / hour
(A) Bankim Chandra Chatterjee
(D) 5 km / minute (B) Toradutt
Ans. Option (B) is correct. (C) R.K. Narayan
Explanation: (D) Rabindra Nath Tagore
Logic: Ans. Option (D) is the correct.
Solving by options, Explanation: Rabindra Nath Tagore is the only
(A) 50 m/sec Indian to receive the noble prize in Literature in
the year 1913 for his novel Gitanjali. He was the
50  m  first Indian and Asian to be awarded the prize.
(B) 50 m/minute = 
60  s  Rabindra Nath Tagore was a Bengali writer,
70 km 70  1000 700  m  poet, composer, philosopher, painter, and social
(C)  
hour 3600 36  s  reformer. He is the writer of India’s national
anthem-Jana Gana Mana and Bangladesh’s
5 km 5  1000 500  m  national anthem- Amar Shonar Bangla. Some of
(D)  
min 60 6  s  his best known works are Ghare-Baire , Gore etc.
Hence, option (B) is correct answer. The Nobel Prize in Literature is a literary
honour given by the Swedish Academy and is
10. When seen through a mirror, a clock shows 3:30. one of the five Nobel Prizes established by the
What is the correct time? will of Alfred Nobel in 1895. The Nobel laureate
(A) 2: 30 (B) 8: 30 is honoured with a gold medal, reward money
(C) 5: 30 (D) 4: 30 and a diploma with citation. The first Nobel
Ans. Option (B) is correct. Prize in literature was given in 1901 to the
Explanation: French poet and essayist Sully Prudhomme.
The correct time is: 14. Who won the ‘Noble Prize’ for in the field of
(11 : 60 - 03 : 30) = 8 : 30 “physiology or medicine”?
(A) C.V. Raman
11. A is 3 years younger than C but one year older than
(B) Jagdish Chandra Bose
D. D is one year older than B but 4 years younger
(C) Homi Jehangir Bhabha
than C. C is 15 years old. What is the age of B in
(D) Har Gobind Khorana
years
Ans. Option (D) is the correct.
(A) 13 (B) 12
(C) 11 (D) 10 Explanation: Har Gobind Khorana won the
Ans. Option (D) is correct. Noble Prize in the field of physiology or
medicine in the year 1968. He shared the prize
Explanation: along with Marshall W Nirenberg and Robert
Let the present age of C = x W. Holley. The three of them were awarded for
Present age of D = y their interpretation of the genetic code and its
Age of A = x - 3 function in protein synthesis.
Age of C = 15 years Har Gobind Khorana was an Indian American
A is 3 years younger than C biochemist who has won many national and
Hence, A’ s age = 15 - 3 = 12 years international awards for his research.
Age of D = C - 4 = 15 - 4 = 11 years The Nobel Prize in Medicine is awarded by
the Nobel Assembly at the Karolinska Institute
Age of D = Age of B + 1
for outstanding discoveries in physiology or
Age of B = 11 - 1 = 10 years medicine every year. It is one of the five Nobel
4 OSWAAL CUET (UG ) 10 Mock Test Papers GENERAL TEST

Prizes that are a part of the will of Alfred Nobel Ans. Option (C) is correct.
in 1895. The Nobel laureate is honoured with Explanation:
a medal, reward money and a diploma with
citation. The first Nobel Prize in literature was A A A
given in 1901 to the German physiologist, Emil B B B
von Behring, for his work on serum therapy
and the development of a vaccine against
diphtheria. B B B

15. The relationship between the values of a country’s


A A
imports and its exports is called.
(A) Balance of Trade (B) Balance of Payment B B ?
(C) Balance of currency (D) Bill of exchange
Ans. Option (A) is the correct. Logic:
Follow the pattern and the symmetry.
Explanation: The relationship between the
Hence,
value of country’s imports and exports is called
Balance of Trade(BOT). It is simply the difference
between country’s imports and exports of goods A
is also known as net exports or commercial
B
balance. If the import is more than the export, it
is known as trade deficit and if the exports are
more than imports, it is known as trade surplus. 17. Match List I with List II
Balance of Payment is basically the summary of
country’s economic transactions with the world LIST I River LIST II City
during an accounting year. This includes all the A. Mahanadi I. Ludhiana
inflow and outflow of funds that have taken from
the country. The BOP statement is indicative of B. Godavari II. Cuttack
country’s surplus or deficit of funds. C. Sutlej III Lucknow
The Bill of exchange is defined under the
D. Gomti IV. Nasik
Negotiable Instruments Act 1881. It is a written
instrument that directs a person to pay the Choose the most appropriate answer from the
designated sum of money to the bearer of options given below:
the instrument. It basically ensures a timely (A) A-II, B-IV, C-III, D-I
payment. It involves three parties- the drawee is (B) A-II, B-IV, C-I, D-III
the party that pays the sum, the payee receives (C) A-IV, B-II, C-I, D-III
that sum, and the drawer is the one that obliges (D) A-III, B-I, C-II, D-IV
the drawee to pay the payee. Ans. Option (B) is the correct.
16. Find out which of the answer figures (1), (2), (3) and Explanation: Mahanadi River flows through
(4) completed the figure matrix? Odisha and Chattisgarh and ends in Bay of
Bengal. It is a Peninsular river in East Central
A A A India. The city of Cuttack is situated on the
B B B banks of the Mahanadi river.
Godavari is the second longest River of
India after Ganga. It passes from the states
of Maharashtra, Andhra Pradesh, Telangana,
B B B
Chhattisgarh, Odisha and empties into Bay
of Bengal. Nasik is situated on the banks of
Godavari river.
A A
B B ? Sutlej is the longest of the five Rivers that flows
through Punjab in India and Pakistan. It is the
easternmost tributary of Indus River and is also
(A) (B) known as Satadru. Ludhiana stands on the old
A A bank of Sutlej River.
Gomti River is a tributary of the River Ganges
B B and originates in Pilibhit, Uttar Pradesh. It flows
through Lucknow and supply water to the city.
(C) (D)
18. The point (-2, 3) lies in which quadrant?
A A
(A) I (B) II
B B (C) III (D) IV
Ans. Option (B) is correct.
CUET(UG) Solved Paper 2023 (21st May Shift -1) 5

Explanation: 21. Which of the following players didn’t receive Medal


in Tokyo Olympics 2020?
3
(A) PV Sindhu
(2, 3)
(B) Neeraj Chopra
(C) Abhinav Bindra
x axis
2 (D) Ravi Kumar Dahiya
Ans. Option (C) is correct.
Explanation: Abhinav Bindra didn’t receive
medal in Tokyo Olympics 2020. He is a retired
shooter and the first Indian to win a gold medal
(2,3)3)lies
(-2, lies
inin
II II quadrant.
quadrant. at the Olympics. He had won the gold medal in
shooting at the 2008 Summer Olympics.
19. If selling price of 80 articles is equal to the cost price
PV Sindhu won the Bronze medal in women’s
of 100 articles, then find the gain percentage.
singles badminton at Tokyo Olympics 2020. She
(A) 30 % (B) 25% is the first Indian woman and second athlete to
(C) 40 % (D) 50 % win two individual Olympic medals.
Ans. Option (B) is correct. Ravi Kumar Dahiya won the Silver medal in
Explanation: men’s 57 kg freestyle wrestling at Tokyo Olympics
Selling Price of 80 articles = C.P of 100 2020, freestyle wrestling.
articles Neeraj Chopra became India’s second individual
SP 100 5 Olympic champion, after Abhinav Bindra, with
= = his men’s javelin throw gold at Tokyo 2020. It
CP 80 4
was India’s first track-and-field medal at any
SP  CP Olympic Games.
Gain%   100
CP
22. Which book is Written by Dr. S. Radha Krishnan?
 SP 
  1   100 (A) The world’s largest democracy
 CP 
5  (B) India divided
   1   100
4  (C) Indian Philosophy
54 (D) India Priceless heritage
  100
4 Ans. Option (C) is the correct.
= 25% Explanation: Dr. Sarvapalli Radhakrishnan
wrote the book, ‘Indian Philosophy’. He was
20. A railway half-ticket costs half the full ticket.
the second President (1962 to 1967) of India.
However the reservation charge for all the tickets
The History of the World’s Largest Democracy
is constant. One full reserved ticket for a journey
is a non-fiction book which is written by Indian
is ` 525. If the cost of one full and one half reserved
historian Ramachandra Guha. The famous book
ticket for the same journey is ` 850, then what is the
‘India divided’ was written by Dr. Rajendra
reservation charge per ticket?
Prasad. The book “India Priceless heritage”
(A) ` 120 (B) ` 150 was written by N.A. Palkhivala, was an Indian
(C) ` 125 (D) ` 115 lawyer and a noted jurist.
Ans. Option (C) is correct.
x x x x
23. If the median of , x , , and (where x > 0 ) is 8,
Explanation: 5 4 2 3
Let the reservation charge be x then the value of x will be
Cost of full ticket = y (A) 24 (B) 32
Price of one full reserved ticket = 525 (C) 8 (D) 16
x + y = 525 Ans. Option (A) is the correct.
y
Cost of 1 full and 1 half ticket  x  y  x  Explanation:
2
3y Arrange in increasing order
2x   850
2 x x x x
3y , , , ,x
2  525  y    850 5 4 3 2
2
Since number of terms = 5 ⇒ odd
2(1050 - 2y) + 3y = 850 × 2
y = 2100 - 1700 = 400 n1
Hence, Median  th observation
x = 525 - y 2
x = 525 - 400 = `125
6 OSWAAL CUET (UG ) 10 Mock Test Papers GENERAL TEST

51 20  25
8 th  20  25 
2 100
x = 50%
=8 3=
rdobservation
 Single equivalent % increase if 50%, 30%
3
x = 8 × 3 = 24 50  30
 50  30 
100
24. The minimum number of colours to required paint
= 95%
all sides of a cube that no two adjacent faces may
have the same colour is. 28. Find out which of the figures out of given option
(A) 5 (B) 4 can be formed from the pieces given in fig (X)
(C) 3 (D) 6
Ans. Option (C) is correct.
Explanation: Opposite face has the same colour
and there are six faces in a cube so we need 3
different colours to do the need full.
25. What sum of money will amount to ` 520 in 5 years
and to ` 568 in 7 years on simple interest?
(A) ` 400 (B) ` 120
(C) ` 510 (D) ` 220
Ans. Option (A) is correct. Fig X
(A) (B)
Explanation:
A = P + SI
P×R×5
520 = P + ...(1)
100
PRT (C)
568  P 
100
2 PR
568  520 
100
48 PR
= Ans. Option (B) is correct.
2 100
Explanation:
Now,put in equation 1
520 = P + 24 × 5
P = 520 - 120 = 400
26. Here are some words translated from an artificial
language
Holo polo means base ball
Moto prot means my India
Prot shot means India won
Which word could be mean “All India Radio” Fig X
Logic:
(A) Holo polo prot (B) Kud prot nid Follow the pattern and the symmetry.
(C) Prot polo nid (D) Polo nid prot Hence,
Ans. Option (C) is correct.
Explanation: Code for India is Prot. Hence, code
for All Radio should be different from the codes
given in the question.
Hence, All India Radio-Kud Prot nid
is correct answer.
27. Find a single equivalent increase if the number is
successively increased by 20 %, 25 % and 30%? 29. Match List I with List II
(A) 75 % (B) 85 % LIST I Physical LIST II
(C) 95 % (D) 35 % Quantity Units
Ans. Option (C) is correct.
A. Electric charge I. Newton
Explanation: B. Force II. Coulamb
Single equivalent % increase if 20%, 25% C. Power III. Joule
ab D. Energy IV. Watt
ab
100
CUET(UG) Solved Paper 2023 (21st May Shift -1) 7
Choose the most appropriate answer from the The entry-level rank for the Indian navy is Sub-
options given below: Lieutenant. The Person then gets promoted to
(A) A-I, B-II, C-IV, D-III Lieutenant and then to Lieutenant Commander.
(B) A-II, B-I, C-IV, D-III
(C) A-II, B-IV, C-I, D-III 32. Statement I : Constitution is the frame work for the
(D) A-III, B-II, C-IV, D-I governance of a country which delegates power
and authority to the executive, legislative and
Ans. Option (B) is correct.
judiciary.
Explanation: The SI unit of electric charge Statement II : It serves a country in maintaining
is Coulomb and is represented by the good relationships with her neighboring countries.
symbol ‘C’. Coulomb is named after French In the light of the above statements, choose the
physicist Charles-Augustin de Coulomb. 1C is most appropriate answer from the options below.
approximately equal to the charge of 6.242 × 1018
(A) Both Statement I and Statement II are correct
protons or electrons.
The SI unit of Force is Newton and is represented (B) Both Statement I and Statement II are incorrect
by N. It is defined in the Second Law of Motion (C) Statement I is correct but Statement II is
and is named after Issac Newton. 1N is defined incorrect
as the force required to accelerate 1 kilogram of (D) Statement I is incorrect but Statement II is
mass at the rate of one meter per second in the correct
direction of applied force. Ans. Option (C) is the correct.
The SI unit of Power is Watt and is named after
James Watt. 1W is the power produced when Explanation: Constitution provides the
1 Joule of work is done for 1 second. framework for governance and delegates the
power and authority to the judiciary, executive,
The SI unit of Energy is Joule and is named after
and legislative of the country. It outlines the
the English physicist James Prescott Joule. 1 Joule
rights and duties of a citizen in order to build a
is the amount of work done when a force of
prosperous nation.
1 newton displaces a mass through a distance of
1 metre in the direction of the force applied. 33. How many terms are there in the A.P. 3, 7, 11, ...
407?
30. In a row of 40 children, A is 13th from the left end
(A) 100 (B) 101
and B is ninth from the right end.
How many children are there between A and C if C (C) 99 (D) 102
is forth to the left of B. Ans. Option (D) is correct.
(A) 13 (B) 14 Explanation:
(C) 15 (D) 16 3, 7, 11…………………407
Ans. Option (B) is the correct. = =
an 407 ,a 3
Explanation: d  a2  a1  7  3  4
12 A 14 C B 8
Left Right an  a   n  1  d
13th
13 9
th th

from left from right 407  3   n  1 4


40 407  3
Hence,14 is correct answer.   n  1
4
31. Major is related to Leiutenant in the same way as 404
‘Squadron Leader is Related to?  n1
4
(A) Pilot officer
(B) Flying Attendant 101 + 1 = n
(C) Group captain n = 102
(D) Flying officer 34. What is the probability that any non-leap year will
Ans. Option (D) is the correct. have 53 Sundays?
Explanation: Major is related to Lieutenant in the 1 2
(A) (B)
same way as Squadron leader is related to Flying 53 53
Officer. A lieutenant is an entry-level rank for the
commissioned officer in the Indian army. The 1 2
(C) (D)
person then gets promoted to Captain and then 7 7
to Major. Ans. Option (C) is correct.
Similarly, the entry-level rank for the Indian
Explanation: As a result, there are 52 Sundays in
air force is Flying officer. The person then gets
a non-leap year. But one leftover day apart from
promoted to Flight Lieutenant and then to
those 52 weeks can be either a Monday, Tuesday,
Squadron Leader.
8 OSWAAL CUET (UG ) 10 Mock Test Papers GENERAL TEST

Wednesday, Thursday, Friday, Saturday, or a 37. A man walks 2 km towards East and then he turns
Sunday. Therefore, the probability of getting 53 to South and walks 6 km. Again he turns to East and
Sundays in a non-leap year is 1/7. walks 4 km, after this he turns to North and walk 14
km. How far is he from his starting point?
35. Match List I with List II (A) 10 km (B) 15 km
LIST I LIST II (C) 20 km (D) 25 km
Ans. Option (A) is correct.
A. Blue I. Increase in crop yield
Revolution and Agricultural Explanation:
Products End
C
B. White II. Increase in Oil-Seeds
Revolution Production
8 km
C. Yellow III. Increase of Fish 4 km
Revolution Production Start A B
2 km
D.Green IV. Increase in the field of
6 6 km
Revolution milk production
Choose the most appropriate answer from the
options given below: 4 km
(A) A-IV, B-III, C-II, D-I (B) A-III, B-IV, C-II, D-I In Δ ABC,
 AC   8  6
2 2 2
(C) A-I, B-III, C-II, D-IV (D) A-III, B-I, C-II, D-IV
Ans. Option (B) is the correct. = 64 + 36
Explanation: Blue Revolution is the improvisation, = 100
domestication, production and cultivation of =
AC =
100 10 km
fish, aquatic plants and animals in order to get
economic benefits from the same. It was also 38. Find the perimeter of a rhombus whose one
known as Neel Kranti Mission and was launched diagonal is 16 cm long and area is 240 cm2.
by Dr. Hiralal Chaudhari and Dr. Arun Krishnsan (A) 68 cm (B) 30 cm
in 1985.
(C) 24 cm (D) 36 cm
White Revolution focused on increasing the
milk production of the country. It is also known Ans. Option (A) is correct.
as Operation Flood and was launched by Dr Explanation:
Verghese Kurien, who is also known as the father 1
of white revolution.  R hom bus 
Areaof  d1  d2
2
Yellow Revolution was launched to increase the
production of edible oil in the country, especially 1
240   16  d2
from mustard and sesame seeds in 1986. Sam 2
Pitroda is known as the father of the Yellow
d2 = 30 cm
Revolution in India.
In rhombus diagonal bisect each other
Green Revolution was launched to increase crop 
yield and agricultural products in India. It led sideof
   8   15    64  225
2 2

to an increase in the production of food grains r hom bus  


especially wheat and rice. MS Swaminathan is   289  17
known as the father of Green Revolution in India.
Perimeter of Rhombus = 4 × 17 = 68 cm
36. A bag contains 5 black, 3 white and 2 red balls.
39. From the figure, what is the value of x?
Three balls are drawn in suecession. What is the
probability that the first ball is red, the second ball A
is black and the third ball is white?
1 3 50
(A) (B)
24 10
1 1
(C) (D) x
10 2 120
Ans. Option (A) is correct. B C D
Explanation: E: event that first ball is red, second (A) 50° (B) 120°
ball is black and 3rd ball is white. (C) 60° (D) 70°
2 5 3 1 Ans. Option (D) is correct.
P  E    
10 9 8 24
CUET(UG) Solved Paper 2023 (21st May Shift -1) 9

Explanation: Ans. Option (C) is the correct.


A Explanation: The Major constituent of natural
gas is Methane. Natural gas is a fossil fuel energy
50˚ source. It is composed mainly of methane (CH4),
smaller amounts of natural gas liquids (NGLs,
which are also hydrocarbon gas liquids), and
nonhydrocarbon gases, such as carbon dioxide
and water vapor. It is formed from the remains
of plants and animals that have been buried
x 120˚ for millions of years under the surface of earth.
These mix with sand, calcium carbonate, silt
B C D and change into natural gas as a result of high
Exterior angle of a triangle = Sum of temperature and pressure.
opposite interior angles
120  50  x
42. Statements.
x  120  50  70 I. Some cars are black
II. Some Lions are cars
40. Consider the Diagram.
Conclusion
500 Candidates appeared in an Examination
comprising test in English, Hindi and Maths. The I. Some blacks are Lions
Diagram gives number of students who failed in II. No black is Lion
different tests. What is the % percentage of student (A) Only I follow
who failed at least two subjects? (B) Only II follow
(C) Either I or II follows
English Hindi
(D) None follows
30 75 Ans. Option (C) is correct.
10
Explanation:
5
Lion Car Black
12 12
Since there is no definite relation between
lion and Black. Hence, either some blacks
are lions or no black is lion
50
43. Match List I with List II
LIST-I Books LIST-II Authors
Math
(A) 6.8% (B) 7.8 % A. India wins freedom I. R K Naravan
(C) 1.0 % (D) 0.078 %
Ans. Option (B) is correct. B. The guide EL Mahatma Gandhi

Explanation: C. India from midnight III Abdul Kalam


English Hindi
to Millenium Azad
D. Conquest of self IV. Shashi Tharoor
30 75
10 Choose the most appropriate answer from the
options given below:
5
(A) A-I, B-III, C-IV, D-II
12 12
(B) A-III, B-I, C-IV, D-II
(C) A-III, B-I, C-II, D-IV
(D) A-I, B-IV, C-II, D-III
50
Ans. Option (B) is the correct.

Math Books Authors


Number of students who failed in at least
India Wins Freedom Maulana Azad
39
subjects = × 100 = 7.8% The Guide R. K. Narayan.
500
India from Midnight Shashi Tharoor
41. Major Constituent of natural gas is. to Millennium
(A) Propane (B) Butane
Conquest of self Mahatama Gandhi
(C) Methane (D) Carbon
10 OSWAAL CUET (UG ) 10 Mock Test Papers GENERAL TEST
44. Match List I with List II Ans. Option (C) is correct.

LIST-I Deficiency LIST-II Diseases Explanation:


Doctors
A. Insulin I. Kwashiorka 1

B. Protein II. Scurvy Professor


5 6 2 3
C. Thyroxin III. Diabetes 4 Married People
1 number indicates doctors who are not
D. Vitamin C IV Goitre married.
Choose the correct answer from the options given
below: 47. The average of 12 numbers is 15 and the average of
(A) A-III, B-I, C-IV, D-II (B) A-I, B-III, C-IV, D-II the first two numbers is 14. What is the average of
the remaining numbers?
(C) A-II, B-I, C-IV, D-III (D) A-IV, B-III, C-II, D-I
(A) 15 (B) 15.2
Ans. Option (A) is the correct.
(C) 14 (D) 14.2
Deficiency Diseases Ans. Option (B) is correct.

Insulin Diabetes Explanation:


sum of all numbers
Protein Kwashiorkor Average = 
Total number of all numbers
Thyroxin Goitre Sum of first 2 numbers = 14 × 2 = 28
Vitamin C Scurvy 28  remaining number sum
15 
12
45. Find the angle of elevation of the Sun, when the Remaining number sum = 15 × 12 - 28
1 = 152
length of the shadow of a tree is times the 152 152
height of the tree. 3 Average remaining    15.2
12  2 10
(A) 30° (B) 45°
(C) 60° (D) 90° 48. Match List-I with List-II
Ans. Option (C) is correct.
LIST-I LIST-II
Explanation:
Scientists Discoveries
Sun
A. Har Gobind I. Discovery of Zero
Khorana
Tree
B. C.V Raman II. Genetic composition
x of cell
C. Jagdish Chandra III. Scattering of light
 Bose
Tree shadow 1 x D. Aryabhata IV Measurement of
3
plant growth
 1  Choose the most appropriate answer from the
tan   x /  x
 3  options given below:
(A) A-II, B-III, C-I, D-IV (B) A-III, B-II, C-IV, D-I
⇒ tan    3
(C) A-II, B-III, C-IV, D-I (D) A-I, B-III, C-II, D-IV
⇒ tan   tan60 Ans. Option (C) is the correct.
⇒   60
Scientists Discoveries
Har Gobind Genetic composition of cell
46. In figure out which Number indicate doctors who Khorana
are not married CV Raman Scattering of Light
(A) 2 (B) 4 Jagdish Chandra Measurement of Plant
(C) 1 (D) 6 Bose growth
Aryabhatta Discovery of Zero
CUET(UG) Solved Paper 2023 (21st May Shift -1) 11
49. Bhoodan-Gram Dan Movement was initiated by. Ans. Option (A) is the correct.
(A) Mahatma Gandhi
(B) Vinoba Bhave Cities Nickname
(C) Shri Ram Chandra Reddy Nagpur Orange City
(D) Sardar Patel
Surat Diamond City
Ans. Option (B) is the correct.
Jaisalmer Golden city
Explanation: Bhoodan Gram Dan Movement
or Land Gift Movement was initiated by Jaipur Pink City
Acharya Vinoba Bhave. It was a voluntary
land reform movement initiated at 52. Which of the following is not a ‘state’?
Pochampally village, in the then Andhra
(A) Nagaland
Pradesh. It was launched in 1951 and as a part
(B) Manipur
of this movement, wealthy
(C) Laddakh
land owners voluntarily gave some percentage (D) Meghalaya
of their land to landless people. These landless Ans. Option (C) is the correct.
labourers could use this and for growing crops
or settling down on the same. They were not Explanation: Ladakh is not a State. It is a Union
allowed to resell it or use it for any commercial Territory and became one on 31 October 2019 as
activity. a part of the Indian Government’s Jammu and
Kashmir Reorganisation Act 2019. The erstwhile
50. The ratio of ages of 2 boys is 3: 7. After 2 years, the Jammu and Kashmir was stripped of its special
ratio of their ages will become 5: 9. The ratio of their status and reconstituted into two Union
ages after 10 years will be Territories – Ladakh without legislature and
(A) 15 : 16 (B) 5 : 17 Jammu and Kashmir with the legislature. The
(C) 17 : 18 (D) 13 : 17 first Lt. Governor of Ladakh Union Territory
Ans. Option (D) is correct. was RK Mathur. This is for the first time in the
Explanation: history of India that a state is being converted
Let the age of first boy = 3x into two Union Territories. India has 28 states
and 9 union territories.
Let the age of second boy = 7x
31 October is observed as the ‘National Unity
After 2 years Day’ to mark the birth anniversary of the
country’s first home minister Sardar Vallabhbhai
3x  2 5
 Patel.
7x  2 9
53. Find the angle traced by hour hand of a correct
27x + 18 = 35x + 10 clock between 7 pm o’ clock and 2 am o’ clock.
8x = 8 (A) 200°
(B) 210°
x=1 (C) 310°
Ratio of their ages after 10 years (D) 290°
Ans. Option (B) is correct.
3x  10 3  1  10 13
    13 : 17
7 x  10 7  1  10 17 Explanation:
Angle made by hour hand in 1 hour = 30°
51. Match List I with List II Cities with their nicknames 7 pm to 2 am = 7 hours.
Angle traced = 30° × 7 = 210°
LIST I Cities LIST II Nickname
54. Find the angle of elevation of the Sun, when the
A. Nagpur I. Diamond City length of the shadow of a tree is 3 times the
B. Sural II. Pink City height of the tree.
(A) 30°
C. Jaisalmer III. Orange City (B) 45°
(C) 60°
D. Jaipur IV. Golden City (D) 90°
Choose the most appropriate answer from the
options given below:
(A) A-III, B-I, C-IV, D-II
(B) A-I, B-III, C-IV, D-II
(C) A-III, B-I, C-II, D-IV
(D) A-II, B-I, C-III, D-IV
12 OSWAAL CUET (UG ) 10 Mock Test Papers GENERAL TEST
Ans. Option (A) is correct. Ans. Option (B) is correct.
Explanation: Explanation:
Sun Area of circle = πr2
Circumference of circle = 2πr
Tree According to question
πr2 = 2πr
x r=2
Hence, diameter = 2r = 2 × 2 = 4 units

Tree shadow  3 x 57. Find the next term in the alpha-numeric series D4T,
F9R, H20P, J43N.
tan   x /  3x  (A) L 90 M
(C) L 90 L
(B) N 90 N
(D) J 90 L
1 Ans. Option (C) is correct.
⇒ tanθ =
3 Explanation:
⇒ tan   tan30 D4T, F9R, H20P, J43N.
Logic:
⇒   30 D  2  F, F  2  H , H  2  J , J  2  L
55. Which of the following is not the satellite launched T  2  R , R  2  P, P  2  N , N  2  L
by India?
4 × 2 + 1 → 9 , 9 × 2 + 2 → 20 , 20 × 2 + 3 → 43,
(A) Explorer
(B) APPLE 43 × 2 + 4 → 90
(C) Bhaskar Hence, L90L is the correct answer.
(D) INSAT
58. Match List I with List II
Ans. Option (A) is the correct.
LIST I LIST II
Explanation: Explorer 1 was the first successful Diet deficiency Disease
satellite launched by the United States on
A. Deficiency of I. Anaemia
31 January 1958. It was launched as a response
Vitamin B
to Soviet Union’s Sputnik 1. It is credited for the
B. Deficiency of II. Beri-Beri
discovery of radiation belts around Earth held in
Vitamin A
place by the planet’s magnetic field. These were
C. Deficiency of Iron III. Goitre
later named as Van Allen belts in honor of their
discoverer. D. Deficiency of IV Nisht blindness
Iodine
APPLE stands for Ariane Passenger Payload Choose the most appropriate answer from the
Experiment. It was India’s first communication options given below:
satellite launched by ISRO on 19 June 1981. The
(A) A-IV, B-II, C-III, D-I (B) A-II, B-IV, C-I, D-III
satellite was carried on a non-metallic bullock
cart to avoid reflections off the metal plates, (C) A-I, B-III, C-IV, D-II (D) A-III, B-I, C-II, D-IV
affecting the satellite’s antenna. Ans. Option (B) is the correct.
Bhaskara I was the first experimental remote
Explanation:
sensing satellite built by ISRO and named after
Indian mathematician Bhaskara. The satellite Diet Deficiency Disease
was launched to collect data on hydrology, Deficiency of Beri Beri
oceanography, forestry and telemetry. It was Vitamin B
launched from Volgograd Launch Station,
Russia. Deficiency of Night Blindness
Vitamin A
INSAT is Indian National Satellite System.
It is a space-based satellite system that Deficiency of Iron Anaemia
provides television, communication, and Deficiency of Iodine Goitre
meteorological services to India and its
neighbouring countries. 59. A and B can do a work in 9 days and 12 days
56. The area of a circle is numerically equal to its respectively. If they work on alternate days starting
circumference. Find the diameter of the circle. with A, then in how many days will the work be
completed?
(A) 2 unit (B) 4 unit
(C) 1 unit (D) 5 unit (A) 36 days (B) 10 days
1
(C) 10 days (D) 13 days
4
CUET(UG) Solved Paper 2023 (21st May Shift -1) 13
Ans. Option (C) is correct. 60. If today is Saturday then what will be the day on 363rd
day?
Explanation:
(A) Sunday (B) Monday
A B
9 12 (C) Thursday (D) Friday
Ans. Option (D) is the correct.
days Explanation:
Considering that there are 7 days in a week.
4 3 (efficiency) Since 363 is not an exact multiple of 7, we
need to find the remainder when dividing
(Total work =LCM(9, 12)) 363 by 7.
36 363 ÷ 7 = 51 with a remainder of 6.
So, if they work on alternate days for two This means that after 51 weeks, there will
days, they will complete a total of 4 + 3 = be 6 days left. Adding these 6 days to
7 units of work. Saturday, we get:
Therefore, if they work on alternate days Saturday + 6 days = Friday
for 10 days, they will complete 7 × 5 = So, the day on the 363rd day will be Friday.
35 units of work.
The remaining work will be 36 - 35 = 1 unit.
Time taken by A to finish the remaining
1
work =
4
1 1
So, total time 10   10 days
4 4

14 OSWAAL CUET (UG ) 10 Mock Test Papers GENERAL TEST

MOCK
Test Paper 1
Maximum Marks : 250 Time : 60 Minutes

General Instructions :
(i) This paper consists of 60 MCQs, attempt any 50 out of 60
(ii) Correct answer or the most appropriate answer: Five marks (+5)
(iii) Any incorrect option marked will be given minus one mark (-1)
(iv) Unanswered/Marked for Review will be given no mark (0)
(v) If more than one option is found to be correct then Five marks (+5) will be awarded to only those who have
marked any of the correct options
(vi) If all options are found to be correct then Five marks (+5) will be awarded to all those who have attempted
the question.
(vii) Calculator / any electronic gadgets are not permitted.

1. Which of the following is not correctly matched? 5. In which part of the Indian Constitution are centre-
Sikh Guru: Contribution state relations mentioned?
(1) Guru Nanak Dev Ji: Contemporary of Mughal (1) Part IV (Article 227 to 234)
emperor Babur. (2) Part XI (Article 245 to 255)
(2) Guru Arjan Dev Ji: Compiled the Adi Granth (3) Part X (Article 234 to 240)
(the primary scripture of Sikhism) and
(4) Part XII (Article 265 to 277)
completed the construction of the Harmandir
Sahib (Golden Temple). 6. Manipur, Meghalaya and Tripura became states
(3) Guru Amardas Sahib Ji: Introduced the concept under _____________.
of “Langar” (community kitchen). (1) North Eastern Retention (Reconstruc­tion) Act,
(4) Guru Ram Das Ji: Founded the city of Amritsar 1971
and designed the Harmandir Sahib (Golden (2) North Eastern Republic of India Act, 1972
Temple).
(3) North Eastern Region New State Act, 1972
2. Fertile riverine alluvial soil is best suited for
(4) North-Eastern Areas (Reorganisation) Act, 1971
producing _________.
(1) Rice (2) Tea (3) Cotton (4) Corn 7. The amount of light entering into eye can be
controlled and regulated by _____________.
3. Andaman and Nicobar Islands is a group of ______
islands/islets? (1) Iris (2) Cornea (3) Pupil (4) Retina
(1) 275 (2) 450 (3) 780 (4) 572 8. Which of the following is not correctly matched?
4. Match the following International boundaries with (1) Treaty of Madras 1769: The Treaty of Madras
their countries. was signed between the British and Hyder Ali
International Countries of Mysore to bring to an end the first Mysore
Boundaries War.
1. 17th Parallel A. Egypt and Sudan (2) Treaty of Wadgaon 1779: Signed between the
2. 38th Parallel B. Pakistan and British and the Marathas bringing the second
Afghanistan phase of the First Anglo-Maratha War to an
3. 22nd Parallel C. South Vietnam and end.
North Vietnam (3) Treaty of Salbai 1782: Signed between the
4. Durand Line D. South Korea and British and the Marathas bringing the First
North Korea Anglo-Maratha War to an end.
(1) 1 - A, 2 - B, 3 - C, 4 -D
(4) Treaty of Seringapatam 1792: Signed between
(2) 1 - C, 2 - D, 3 - A, 4 - B the British and the Marathas bringing the first
(3) 1 - C, 2 - D, 3 - B, 4 - A phase of the First Anglo-Maratha War to an
(4) 1 - C, 2 - A, 3 - B, 4 - D end.
MOCK TEST Paper-1 15
9. Which of the following pair is correctly matched? 17. Which one of the following is NOT a natural fiber?
(1) First Five-Year Plan (1951-1956): It is based on (1) Terylene (2) Jute (3) Flax (4) Wool
Mahalanobis Model. 18. Which of the following is good insulator?
(2) Second Five-Year Plan (1956-1961): It is based (1) Mercury (2) Cadmium
on Harrod-Domar Model. (3) Germanium (4) Glass
(3) Fourth Five-Year Plan (1969-1974): It is based on 19. In which nation did the Indian team achieve its
Ashok Rudra Alon Model.
inaugural bronze medal in the ‘Asian Surfing
(4) Fifth Five-Year Plan (1974-1979): It is based on Championship’?
Gadgil Model.
(1) Malaysia (2) Russia
10. Which of the following is/are correctly matched? (3) Greece (4) Maldives
International Headquarters
organization
20. The Biraja Temple, the Rajarani Temple and the
Samaleswari Temple are all located in ____________
1. UNICEF New York
(1) Odisha (2) Assam
2. World Bank Washington DC
(3) Tamil Nadu (4) Kerala
3. World Trade Geneva
Organization
21. If the 8-digit number 789x531y is divisible by 72.
Then the value of (5x – 3y) is:

Choose the correct option.
(1) 0 (2) –1 (3) 2 (4) 1
(1) 1 only (2) 2 only
(3) 3 only (4) 1, 2 and 3
22. The angle of elevation at the top of an unfinished
tower at a point distant 78 m from its base is
11. The Aligarh Movement was started by _________ . 30°. How much higher does the tower be raised
(1) Dr. Maghroor Ahmad Ajazi (in m) so that the angle of elevation of the top of
(2) Muhammad Ali Jinnah the finished tower at the same point will be 60°?
(3) Maulana Manzoor Ahsan (1) 78 3 (2) 80 (3) 52 3 (4) 26 3
(4) Syed Ahmed Khan 23. The arithmetic mean of the following numbers
12. Which of the following is not a computer language? 1, 2, 2, 3, 3, 3, 4, 4, 4, 4, 5, 5, 5, 5, 5, 6, 6, 6, 6, 6, 6 and
I. C++ 7, 7, 7, 7, 7, 7, 7, is
II. Java (1) 4 (2) 5 (3) 14 (4) 20
III. Linux 24. A dealer sells a machine having marked price as
(1) Only I (2) Only III ` 3840 at a discount of 20%. What is the selling
price (in `) of the machine?
(3) II and III (4) I and II
(1) 3072 (2) 3500 (3) 4608 (4) 3240
13. Which of the following individuals or organizations
was not honoured with the Nobel Peace Prize in 25. Two pipes of length 1.5 m and 1.2 m are to be cut
2022? into equal pieces without leaving any extra length
(1) Ales Bialiatski of pipes. The greatest length of the pipe pieces of
same size which can be cut from these two lengths
(2) Russian human rights organisation Memorial
will be
(3) Maria Ressa
(1) 0.13 metre (2) 0.4 metre
(4) Ukrainian human rights organisation Center
(3) 0.3 metre (4) 0.41 metre
for Civil Liberties
14. Surendranath Banerjee and Ananda Mohan Bose 26. If the simple interest on Re. 1 for 1 month is 1 paisa,
then the rate per cent per annum will be
founded ___________ in Bengal in 1876.
(1) 10% (2) 8% (3) 12% (4) 6%
(1) Bengal Association
(2) Indian Association 27. Two numbers whose sum is 84 can not be in the
ratio
(3) Tathagat Association
(1) 5 : 7 (2) 13 : 8 (3) 1 : 3 (4) 3 : 2
(4) Oriental Association
28. If m – n = 16 and m2 + n2 = 400, the value of mn
15. The rover of the Chandrayaan 3 mission is referred
is =
to as:
(1) 72 (2) 25 (3) 144 (4) 192
(1) Vikram (2) Bheem
(3) Pragyaan (4) Dhruv
29. A boat moves downstream at the rate of 8 km per
hour and upstream at 4 km per hour. The speed of
16. Which of the following is the study of soil? the boat in still waters is:
(1) Mineralogy (2) Pedology
(1) 4.5 km per hour (2) 5 km per hour
(3) Glaciology (4) Cosmology
(3) 6 km per hour (4) 6.4 km per hour
16 OSWAAL CUET (UG ) 10 Mock Test Papers GENERAL TEST

30. A takes three times as long as B and C together 39. Two trains leave Varanasi for Lucknow at 11:00 AM
to do a job. B takes four times as long as A and C and at 11:30 AM respectively and travel at speeds
together to do the work. If all the three, working of 110 km/h and 140 km/h, respectively. How many
together can complete the job in 24 days, then the kilometres from Varanasi will both trains meet?
number of days. A alone will take to finish the job
(1) 256 2 km (2) 255 1 km
is 3 3
1
(1) 100 (2) 96 (3) 95 (4) 90 (3) 246 km (4) 238 2 km
31. If p, q, r are in Geometric Progression, then which is 3 3
true among the following? 40. In how many years will a sum of ` 800 at 10%
pr per annum compounded semi-annually become
(1) q  (2) p2 = qr ` 926.10?
2
p r (1) 2 1 years (2) 3 years
(3) q = pr (4) = 2
r q
(3) 2 years (4) 1 1 years
32. ABC is an isosceles triangle where AB = AC which 2
is circumscribed about a circle. If P is the point 41. Select the option in which the words share the
where the circle touches the side BC, then which same relationship as that shared by the given pair
of the following is true? of words.
(1) BP = PC (2) BP > PC Dentist : Doctor
1 (1) Biology : Astrology
(3) BP < PC (4) BP = PC
2 (2) Chemistry : Science
(3) Algebra : Geometry
33. In an isosceles triangle ABC, AB = AC, XY || BC.
If ∠A = 30°, then ∠BXY = ? (4) Line : Circle

(1) 75° (2) 30° (3) 150° (4) 105°


42. In a code language, if ‘I like chocolates’ is written as
‘958’, ‘we bought chocolates’ is written as ‘153’ and
34. Between 10 pm and 6 am, a fitness band records ‘we like them’ is written as ‘816’, then how would
the following data. Refer the following data table ‘I bought them’ be written in this language?
and answer the question. (1) 859 (2) 251 (3) 936 (4) 951
Deep sleep 15 43. In a class of 42 students, Swati’s rank is 19th from the
Dreaming 15 bottom. Purshottam is 6 ranks below Swati. What is
Purshottam’s rank from the top?
Light sleep 5
(1) 30th (2) 32nd (3) 33rd (4) 34th
Extremely light sleep 30
Awake 35
44. If the minute hand of a clock points towards the
South at 12 : 00 a.m., then in which direction will

How long was the user in Deep Sleep or was the minute hand point at 3 : 00 a.m. in the same
Awake? clock?
(1) 3.5 hours (2) 2.5 hours (1) North (2) South (3) West (4) East
(3) 4.5 hours (4) 4 hours 45. Select the option in which the number set shares
35. The milk and water in two vessels A and B are in the same relationship as that shared by the given
the ratio 4 : 3 and 2 : 3 respectively. In what ratio, number set.
the liquids in both the vessels be mixed to obtain a (17, 24, 45)
new mixture in vessel C containing half milk and (1) (19, 26, 34) (2) (18, 23, 46)
half water? (3) (15, 20, 43) (4) (12, 19, 40)
(1) 7 : 5 (2) 5 : 2 (3) 3 : 11 (4) 1 : 2 46. In a code language ‘BLOCK’ is written as ‘31316412’.
1 How will ‘SUPREME’ be written in that language?
36. If x  3  2 2 ,then x + is equal to
x (1) 202217196146 (2) 192116185605
(1) 0 (2) 1 (3) 2 (4) 2 2 (3) 202317206146 (4) 192217196156

37. The radius of the incircle of an equilateral DABC of 47. Select the Venn diagram that best illustrates the
side 2 3 units is x cm. The value of x is: relationship among the following classes: Uncle,
Relatives, Rich
1 1
(1) (2) (3) 1 (4) 3
3 2
38. If diagonals of a rhombus are 16 cm and 30 cm. (1) (2)
What is the perimeter (in cm) of the rhombus?
(1) 32 (2) 64 (3) 34 (4) 68
MOCK TEST Paper-1 17
(1) Only Conclusion II follows
(3) (2) Conclusion I and II both follow
(4)
(3) Neither I nor II follow
(4) Only Conclusion I follows
48. Read the given statements and conclusions 52. Select the correct option that is related to the third
carefully. Assuming that the information given term in the same way as the second term is related
in the statements is true, even if it appears to be at to the first term.
variance with commonly known facts, decide which Television : Telecast :: Radio : ?
of the given conclusions logically follow(s) from the
(1) Broadcast (2) Friendship
statements.
(3) Compose (4) Talking
Statements:
53. Mohini is taller than Nita but not shorter than
I. All polygons are angles.
Sarita. Sarita and Malini are of same height. Mohini
II. All angles are diagonals. is shorter than Hema. Among them, who is the
III. All cones are cubes. second tallest?
IV. All cubes are decagons. (1) Mohini
V. No diagonal is a cube. (2) Nita
Conclusions: (3) Hema
I. Some diagonals are polygons. (4) Cannot be determined
II. All diagonals are decagons. 54. Ramesh is the father of Manideep. Ramesh has only
III. No polygon is a cone. two children. Manideep is the brother of Niharika.
IV. Some cubes are angles. Niharika is the daughter of Kavita. Ananya is the
granddaughter of Kavita. Sujit is the father of
(1) Bothe conclusions II and IV follow.
Ananya. How is Sujit related to Manideep?
(2) Only conclusion I follows.
(1) Soni (2) Brother
(3) Both conclusions I and II follow.
(3) Brother-in-law (4) Son-in-law
(4) Both conclusions I and III follow.
55. Which two signs/numbers need to be interchanged
49. Which number will follow next in the given series? to make the given equation correct?
3, 16, 29, 42, ?

8 ÷ 2 + 4 = 10
(1) 57 (2) 55 (3) 54 (4) 61
(1) ÷ and +, 8 and 4 (2) ÷ and +
50. Six friends A, B, C, D, E and F are sitting around a (3) + and ÷, 8 and 2 (4) + and ÷, 2 and 4
round table facing the centre. C is seated between
A and B, B is in immediate left of D. If C is facing E, 56. From a point D Sarita Walked 50 metres to
who is facing F? the North, then after turning to right walked
50 metres. She again turned right and walked
(1) E (2) A (3) D (4) B
another 70 metres and finally turned to right and
51. In the following question, two statements are walked 50 metres. In which direction and at what
given each followed by two conclusions I and II. distance is she from the starting point D?
You have to consider the statements to be true (1) 20 metres to the North
even if they seem to be at variance from commonly
(2) 40 metres to the South
known facts. You have to decide which of the
given conclusions, if any, follows from the given (3) 20 metres to the South
statements. (4) 30 metres to the South
Statements: 57. Pointing to a lady, Diwakar said, “Her mother’s
(I) 
Major companies are nowadays surviving only grandson is my son.” How is that lady related
because of the strong supply chain system. to Diwakar?
(II) 
Supply chain comprises of logistics, retailers (1) Aunty (2) Sister
and distributors. (3) Mother (4) Wife
Conclusions: 58. Question figure
(I) Retailers play most important role because they
interact with customers on first hand.
(II) 
If an organization has strong logistics, then
product can easily be available to the customers
in such an era of competition.
18 OSWAAL CUET (UG ) 10 Mock Test Papers GENERAL TEST

Answer figures and walks another 10 km. He takes another right
turn and walks 10 km to reach his school. How
far (in km) and in which direction is he from his
(1) (2) home?
(1) 10, North (2) 10, South
(3) 20, North-East (4) 20, South-West
60. Select the term that will come next in the following
(3) (4) series.
YCL, MQZ, ......, OSB
(1) AEN (2) BFQ
59. Kamal starts walking from his home facing West
direction. After walking 10 km he takes a right turn (3) ADM (4) BEM



Answer Key

1. (3) 2. (1) 3. (4) 4. (3) 5. (2) 6. (4) 7. (1) 8. (4) 9. (3) 10. (4)
11. (4) 12. (2) 13. (3) 14. (2) 15. (3) 16. (2) 17. (1) 18. (4) 19. (3) 20. (1)
21. (2) 22. (3) 23. (2) 24. (1) 25. (3) 26. (3) 27. (4) 28. (1) 29. (3) 30. (2)
31. (3) 32. (1) 33. (4) 34. (4) 35. (1) 36. (4) 37. (3) 38. (4) 39. (1) 40. (4)
41. (2) 42. (3) 43. (1) 44. (2) 45. (4) 46. (1) 47. (4) 48. (4) 49. (2) 50. (4)
51. (1) 52. (1) 53. (4) 54. (3) 55. (4) 56. (3) 57. (2) 58. (2) 59. (1) 60. (1)
MOCK TEST Paper-1 19

MOCK
Test Paper 2
Maximum Marks : 250 Time : 60 Minutes

General Instructions :
(i) This paper consists of 60 MCQs, attempt any 50 out of 60
(ii) Correct answer or the most appropriate answer: Five marks (+5)
(iii) Any incorrect option marked will be given minus one mark (-1)
(iv) Unanswered/Marked for Review will be given no mark (0)
(v) If more than one option is found to be correct then Five marks (+5) will be awarded to only those who have
marked any of the correct options
(vi) If all options are found to be correct then Five marks (+5) will be awarded to all those who have attempted
the question.
(vii) Calculator / any electronic gadgets are not permitted.

1. Which of the following is/are correctly matched 5. Who among the following was the last Nawab of
with their respective states: Awadh?
1. Somnath Temple: Gujarat (1) Amjad Ali Khan (2) Muhammad Mukim
2. Kamakhya Temple: Assam (3) Wajid Ali Shah (4) Saadat Ali Khan
3. Ramnathaswamy Temple: Tamil Nadu 6. Rajat has hypermetropia. What type of lens will
4. Mahabodhi Temple: Bihar the ophthalmologist recommend to correct his
Choose the correct option: vision?
(1) 1 and 2 only (2) 2 and 3 only (1) Bifocal (2) Concave
(3) 1, 2 and 4 only (4) 1, 2, 3 and 4 (3) Progressive (4) Convex
2. Which of the following is/are the official language 7. Which of the following is/are correctly matched?
of the United nation?
List I (Port) List II (Feature)
1. Arabic 2. Hindi 3. English 4. Hebrew
1. Mumbai Biggest port in India
Choose the correct option:
(1) 1 and 2 only (2) 1 and 3 only 2. Visakhapatnam Landlocked harbour
(3) 2 and 3 only (4) 1, 2, 3 and 4 3. Chennai Oldest port on the Eastern
3. Which of the following is/are correctly matched? Coast
Battles Year 4. Kochi Queen of the Arabian Sea
1. Battle of Chausa (1) 1539
Select the correct option:
2. Battle of Saraighat (2) 1671 (1) 1 and 2 only (2) 2 and 3 only
3. Battle of Saragarhi (3) 1897 (3) 1, 2 and 4 only (4) 1, 2, 3 and 4
4. The first battle of Panipat (4) 1526
8. Which among the following is not an input device?
Choose the correct option:
(1) Plotter
(1) 1, 2 and 3 only (2) 2, 3 and 4 only
(2) Magnetic Ink Character Recognition (MICR)
(3) 1 and 4 only (4) 1, 2, 3 and 4
4. “Regarding TRIMS, which of the following (3) Optical Mark Recognition (OMR)
statements is true? (4) Barcode Reader
(1) Trade-Related Investment Measures (TRIMS) 9. The largest invertebrate is a/an
prohibit quantitative restrictions on imports by
(1) Sponge (2) Mollusc
foreign investors.
(2) They apply to investment measures related to (3) Arthropod (4) Echinoderm
trade in both goods and services. 10. How does a Unicellular Organism reproduce?
(3) Both 1 and 2 (1) Cell division (2) Cell reproduction
(4) Neither 1 nor 2 (3) Cell synthesis (4) Fragmentation
20 OSWAAL CUET (UG ) 10 Mock Test Papers GENERAL TEST

11. Which of the following is/are correctly matched? 19. Who invented vaccination for ‘Small Pox’?
1. Jnanpith Award: Literature (1) Sir Fredrick Grant Banting
2. Pritzker Architecture Prize Architecture (2) Sir Alexander Fleming
3. Grammy Award Outstanding achievements in (3) Edward Jenner
the music industry (4) Louis Pasteur
4. Abel Prize Outstanding scientific work in the 20. Which city in India is world renowned for one of
field of mathematics the most traditional embroidery styles, Chikankari?
Choose the correct option:
(1) Ahmedabad (2) Lucknow
(1) 1 and 4 only (2) 2 and 3 only (3) Hyderabad (4) Udaipur
(3) 1 and 3 only (4) 1, 2, 3 and 4 21. How much simple interest will ` 4000 can in
18 months at 12% per annum?
12. In our country the ‘Van Mahotsav’ Day is observed
(1) ` 216 (2) ` 360
on:
(3) ` 720 (4) ` 960
(1) 10th August (2) 1st July
22. The remainder when 321 is divided by 5 is
(3) 2nd October (4) 1st December
(1) 1 (2) 2 (3) 3 (4) 4
13. The first woman of Indian origin who undertook a
space journey 23. The marked price of a piano was `15,000. At the
time of sale, there were successive discounts of
(1) Sunita Williams
20%, 10% and 10% respectively on it. The sale price
(2) Kalpana Chawla was
(3) Darshan Ranganath (1) ` 9,720 (2) ` 9,750 (3) ` 9,760 (4) ` 9,780
(4) Avani Chaturvedi 24. Three numbers are in the ratio 3 : 4 : 5. The sum of
14. Which Commission was appointed by the central the largest and the smallest equals the sum of the
government to examine issues related to Centre- second and 52. The smallest number is
State relations? (1) 20 (2) 27 (3) 39 (4) 52
(1) Mandal Commission 25. A boat goes a certain distance at 30 km/hr and
(2) Sarkaria Commission comes back the same distance at 60 km/hr. What is
(3) Nanavati Commission the average speed (in km/hr) for the total journey?
(4) Kothari Commission (1) 45 (2) 50 (3) 40 (4) 35
15. The three-tier system of local governance does not 26. 84 Maths books, 90 Physics books and 120
encompass the? Chemistry books have to be stacked topic-wise.
(1) Panchayat Samiti (2) Village Committee How many books will be there in each stack so that
each stack will have the same height too?
(3) Gram Panchayat (4) Zila Parishad
(1) 12 (2) 18 (3) 6 (4) 21
16. To neutralize too acidic soil ______________ is
used. 27. If the mean of the numbers 2, (2p + 2), 7, 13, 17, 4,
and (p – 1) is 8, then find their median.
(1) Quick lime
(1) 4 (2) 6 (3) 7 (4) 13
(2) Slaked lime
(3) Both quick and slaked lime 28. A can do 1 of a work in 10 days. B can do 1 of the
(4) None Option Is correct 4 3
17. Which of the following primarily produces work in 20 days. In how many days can both A and
B together do the work?
Nitrogen dioxide leading to air pollution?
(1) 30 days (2) 32 days
(1) CFL Lamp (2) Automobile Battery
(3) 24 days (4) 25 days
(3) Polymer (4) Diesel Engine
18. Which of the following is/are correctly matched? 29. Tickets numbered from 1 to 20 are mixed and a
ticket is drawn at random. What is the probability
(1) Cricket: Pinch hitter
that the ticket drawn bears a number which is a
(2) Hockey: Clapper multiple of 3?
(3) Baseball: Duck (1) 3/10 (2) 3/20 (3) 2/5 (4) 1/2
(4) Lawn Tennis: Deuce
30. The sum of the series (1 + 0.6 + 0.06 + 0.006 +
Choose the correct option:
0.0006 + ...) is
(1) 1 and 2 only (2) 2 and 3 only 2 1 1 2
(1) 1 (2) 1 (3) 2 (4) 2
(3) 2 and 4 only (4) 1, 2, 3 and 4 3 3 3 3
MOCK TEST Paper-2 21
31. In the figure (not drawn to scale) given below, if 40. A rice trader buys 16 quintals of rice for ` 5632. 20%
AD = DC = BC and ∠BCE = 96°, then ∠DBC is: rice is lost in transportation. At what rate should he
E sell to earn 25% profit?
C (1) ` 225.2 per quintal (2) ` 550 per quintal
96°
(3) ` 440 per quintal (4) ` 563.2 per quintal
41. Richa starts walking from her home and walks
50 m towards West and then turns left and walks
A D B 90 m. Then she again turns left and walks 30 m.
(1) 32° (2) 84° (3) 64° (4) 96° After that she again turns left and goes 120 m.
2 She finally turns right walks 20 m and reaches her
32. If (10.15) = 103.0225, then the value of
final destination. How far is she from her home
1.030225 + 10302.25 is
now and in which direction with reference to her
(1) 1025·15 (2) 103·515 (3) 102·515 (4) 102·0515 home?
33. Refer the following data table and answer the (1) 40 m, North West (2) 30 m, South
question.
(3) 40 m, North East (4) 30 m, North
Weight (kg) Height (m) 42. If 18th February 1997 fell on Tuesday, then what
Apeksha 59 1.8 was the day of 18th February 1999?
Apsara 51 1.72 (1) Friday (2) Monday
Aradhana 62 1.57 (3) Tuesday (4) Thursday

Archana 74 1.68
43. In a certain code language. ‘AROUND’ is coded as
‘52182412144’ and ‘FIX’ is coded as ‘63624’. How
Who has the least weight to height ratio?
will ‘PLASTIC’ be coded in that language?
(1) Apeksha (2) Apsara (1) 1612261920183 (2) 1612521920363
(3) Aradhana (4) Archana (3) 1812521920383 (4) 1612522021363
34. A bicycle wheel makes 5000 revolutions in moving 44. A # B’ means ‘A is the son of B’. ‘A @ B’ means ‘A is
11 km. Then the radius of the wheel (in cm) is
22 the mother of B’.
(Take π = )
7 “A & B’ means ‘A is the wife of B’. ‘A% B’ means ‘A is

(1) 70 (2) 35 (3) 17.5 (4) 140 the sister of B’.
35. What is the ratio of in-radius and circum-radius of If ‘M @ R % K # G # N & T’ which of the following

an equilateral triangle? statements is NOT correct?
(1) 1 : 2 (2) 1 : 3 (3) 1 : 4 (4) 3 : 2 (1) N is paternal grandmother of K.
36. In two blends of mixed tea, the ratios of Darjeeling (2) T is paternal grandfather of M.
and Assam tea are 4 : 7 and 2 : 5. The ratio in which (3) M is the mother of K.
these two blends should be mixed to get the ratio
(4) R is the daughter of G.
of Darjeeling and Assam tea in the new mixture as
6 : 13 is 45. Select the option in which the numbers are related
in the same way as are the numbers in the given
(1) 22 : 35 (2) 26 : 35 (3) 35 : 78 (4) 13 : 22
set.
37. A train without stoppage travels with an average
speed of 72 km/h and with stoppage, it travels (109, 114, 139)

with an average speed of 60 km/h. For how many (1) (313, 318, 343) (2) (579, 534, 549)
minutes does the train stop on an average per (3) (268, 302, 237) (4) (419, 424, 439)
hour? 46. In a certain code language, ‘FRENCH’ is coded as
(1) 10 (2) 12 (3) 6 (4) 8 ‘114’ and ‘LOSS’ is coded as ‘47’. How will ‘COURSE’
38. The compound interest on ` 64,000 for 3 years, be coded in that language?
compounded annually at 7.5% p.a. is (1) 81 (2) 120 (3) 103 (4) 87
(1) ` 14,400 (2) ` 15,705 47. Select the Venn diagram that best illustrates the
(3) ` 15,507 (4) ` 15,075 relationship among the following classes :
39. If P = 7 + 4 3 and PQ = 1, then what is the value Snakes, Reptiles, Poisonous

1 1
of 2 + 2 ?
P Q (1) (2)

(1) 196 (2) 194 (3) 206 (4) 182


22 OSWAAL CUET (UG ) 10 Mock Test Papers GENERAL TEST

54. Which number will appear on the face opposite


to 1?
(3) (4)

2 2

48. Read the given statements and conclusions 4 3 4 1


carefully. Assuming that the information given
in the statements is true, even if it appears to be at
variance with commonly known facts, decide which (1) 6 (2) 4 (3) 5 (4) 3
of the given conclusions logically follow(s) from the 55. P, L, T, B, N and D are six members of a business
statements. family. N is the son of B, who is not the mother of N.
L is the brother of B. D and B are a married couple.
Statements: T is the daughter of D, who is the sister of P. How is
I. No bank is an office. N related to T?
II. All offices are stalls. (1) Sister (2) Mother
Conclusions: (3) Brother (4) Father
56. In the following question, two statements are
I. No bank is a stall. given each followed by two conclusions I and II.
II. No stall is a bank. You have to consider the statements to be true
III. Some stalls are offices. even if they seem to be at variance from commonly
known facts. You have to decide which of the
IV. All the stalls are offices. given conclusions, if any, follows from the given
(1) Only conclusion I follows statements.
(2) Both conclusions I and II follow Statements:
(I) Pre-school culture is prevailing in all parts of
(3) Only conclusions III follows the country now.
(4) Both conclusions I and IV follow (II) Parents have less time to spend with their
49. Select the option that will correctly replace the children so toddlers are admitted to school at a
question mark (?) in the series. very early age.
Conclusions:
PRL, QPM, RNN, SLO, ?

(I) Number of pre-schools is decreasing in all parts
(1) TIO (2) TKP (3) TJP (4) TIP of country.
50. Five students are standing in a line. Suman is (II) Children are becoming very active and
standing next to Pranav but not next to Deena. intelligent because of this pre-schooling.
Kumar is standing next to Raju, who is standing on (1) Only Conclusion II follows
the extreme left, and Deena is not standing next to (2) Conclusion I and II both follow
Kumar. Who is standing in the middle? (3) Neither I nor II follow
(1) Deena (2) Raju (3) Kumar (4) Suman (4) Only Conclusion I follows
57. Pointing towards a girl, Chetan said, “She is the
51. Select the correct combination of mathematical daughter of the only child of my grandmother”.
signs to replace ‘*’ signs and to balance the given How is Chetan related to that girl?
equation. (1) Father (2) Son
32 * 8 * 14 * 7 * 16
(3) Brother (4) Husband
(1) × − + = (2) ÷ × − = 58. Question figure
(3) + = ÷× (4) − × ÷ =
52. P and Q start cycling from the same point.
P cycles 18 km North, then turns to her right and
cycles 3 km. Q cycles 9 km East, then turns North
and cycles 10 km North, then turns left and cycles
6 km. Where is Q now with respect to the position Answer figures

of P?
(1) 8 km South (2) 8 km North
(1) (2)
(3) 28 km South (4) 28 km North
53. Select the correct option that is related to the third
term in the same way as the second term is related
to the first term.
Bees : Hum :: Snakes : ?
(3) (4)
(1) Hiss (2) Hoot (3) Roar (4) Talk
MOCK TEST Paper-2 23
59. Which answer figure will complete the pattern in
the question figure?
Question Figure:
(3) (4)

60. A migrating bird flies 40 km North, then turns East


and flies 50 km, then turns North and flies 110 km,
? and turns to its left and flies 50 km. Where is it now
with reference to its starting position?
(1) 150 km South (2) 150 km North
Answer Figures:

(3) 70 km North (4) 70 km South

(1) (2)



Answer Key

1. (4) 2. (2) 3. (4) 4. (1) 5. (3) 6. (4) 7. (4) 8. (1) 9. (2) 10. (1)
11. (2) 12. (2) 13. (2) 14. (2) 15. (2) 16. (3) 17. (4) 18. (3) 19. (3) 20. (2)
21. (3) 22. (3) 23. (1) 24. (3) 25. (3) 26. (3) 27. (3) 28. (3) 29. (1) 30. (1)
31. (3) 32. (3) 33. (2) 34. (2) 35. (1) 36. (1) 37. (1) 38. (3) 39. (2) 40. (2)
41. (4) 42. (4) 43. (2) 44. (2) 45. (1) 46. (4) 47. (1) 48. (3) 49. (3) 50. (4)
51. (4) 52. (1) 53. (1) 54. (4) 55. (3) 56. (3) 57. (3) 58. (3) 59. (2) 60. (2)
24 OSWAAL CUET (UG ) 10 Mock Test Papers GENERAL TEST

MOCK
Test Paper 3
Maximum Marks : 250 Time : 60 Minutes

General Instructions :
(i) This paper consists of 60 MCQs, attempt any 50 out of 60
(ii) Correct answer or the most appropriate answer: Five marks (+5)
(iii) Any incorrect option marked will be given minus one mark (-1)
(iv) Unanswered/Marked for Review will be given no mark (0)
(v) If more than one option is found to be correct then Five marks (+5) will be awarded to only those who have
marked any of the correct options
(vi) If all options are found to be correct then Five marks (+5) will be awarded to all those who have attempted
the question.
(vii) Calculator / any electronic gadgets are not permitted.

1. Which of the following is/are correctly matched? 4. Who ruled Mysore under a subsidiary alliance
Part Function with the British after the defeat of Tipu Sultan at
1. Ribosomes Making protein the Battle of Seringapatam?
2. Golgi Apparatus Involved in transporting (1) Gangas (2) Kadambas
substances in and out (3) Wodeyars (4) Pandyans
of the cell 5. Which of the following water current is correctly
3. Mitochondria Produces energy for matched?
the cell (1) Kuroshio Current: It is a warm current of the
Choose the correct code: Indian Ocean.
(1) 1 only (2) 2 only (2) Humboldt Current: It is a Cold current of the
Atlantic Ocean.
(3) 3 only (4) 1, 2 and 3
(3) Labrador Current: It is a Cold current of the
2. Which of the following is/are correctly matched Pacific Ocean.
1. The plastids: It is the outermost covering of (4) Canaries Current: It is a cold current of the
animal cells. Atlantic Ocean.
2. Lysosomes: It is also known as a suicide bag of 6. Who became the Nawab of Bengal after the death
the cell. of Alivardi Khan in 1756?
3. Nucleus: It is defined as a double-membraned (1) Mir Jafar (2) Murshid Quli Khan
eukaryotic cell organelle that contains genetic (3) Siraj ud-Daulah (4) Sujan Khan
material.
7. Which of the following is/are correctly matched
4. Cell Membrane: It is the cell organelles found regarding Extreme points of India:
in all plant cells.
1. Easternmost: Kibithu in Arunachal Pradesh.
Choose the correct code:
2. Westernmost: Ghuar Mota in the Kutch region
(1) 1 and 4 only (2) 2 and 3 only of Gujarat.
(3) 2, 3, and 4 only (4) 1, 2, 3 and 4 3. Northernmost: Indira Col
3. Which of the following pair is correctly matched 4. Southernmost (Mainland): Kanyakumari is also
Organisation: Headquarter known as Cape Comorin.
(1) National Bank for Agriculture and Rural Choose the correct code
Development (NABARD): Delhi (1) 1 and 2 only (2) 2, 3, and 4 only
(2) National Housing Bank (NHB): Mumbai (3) 1, 2, 3 and 4 (4) 2 and 4 only
(3) Insurance Regulatory and Development 8. In Microsoft Excel, the ____________ function
Authority (IRDAI): Hyderabad returns the smallest value among the values passed
(4) Small Industries Development Bank of India as arguments.
(SIDBI): Prayagraj (1) LEAST (2) LESS (3) MIN (4) LOW
MOCK TEST Paper-3 25
9. Which type of image is always formed by a convex 19. Which of the following is/are correctly matched?
mirror? 1. Article 23: Prohibition of traffic in human
I. Real beings and forced labour.
II. Virtual 2. Article 25: Freedom of conscience and free
II. Enlarged profession, practice and propagation of
(1) Only I (2) Only II religion.
(3) Only I and III (4) Only II and III 3. Article 19: Protection of certain rights regarding
10. Which of the following is/are correctly matched? freedom of speech, etc.
Battle: Year 4. Article 15: Prohibition of discrimination on
(1) Battle of Chamkaur: 1704 grounds of religion, race, caste, sex or place of
(2) Battle of Swally: 1612 birth.
(3) Battle of Plassey: 1764 Choose the correct code.

(4) Battle of Buxar: 1757 (1) 1, 3 and 4 only (2) 2, 3 and 4 only
Choose the correct code.
(3) 1, 2 and 4 only (4) 1, 2, 3 and 4
(1) 1 and 3 only (2) 2 and 3 only
20. “Bulls eye” is used in the game of:
(3) 3 and 4 only (4) 1 and 2 only
(1) Boxing (2) Basketball
11. Which of the following gas contributes the
(3) Polo (4) Shooting
maximum to the phenomenal of global warming?
(1) Methane 21. LCM of 2 , 4 , 5 is
(2) Chlorofluorocarbon (CFC) 3 9 6
(3) Nitrogen dioxide 8 20
(1) (2) (3) 10 (4) 20
(4) Carbon dioxide 27 3 3 27
12. Who discovered electron? 22. A boat can row 35 km, upstream in 7 hours and the
(1) E. Ghostein (2) J. J. Thomson same distance downstream in 5 hours. What is the
(3) Ernest Rutherford (4) J. Chadwick speed (in km/hr.) of the boat in still water?
13. From which country does Dr Thani Al Zeyoudi, (1) 5 (2) 7 (3) 9 (4) 6
who has been chosen as the chair of the 13th
Ministerial Conference of the WTO, hail?
23. If a 10-digit number 1330x558y2 is divisible by 88.
Then the value of (x + y) is:
(1) Greece (2) Qatar
(3) Finland (4) UAE (1) 7 (2) 9 (3) 6 (4) 8
14. Sir C. V. Raman was awarded Nobel Prize for his 24. Find the mean of 12, 15, 7, 8 and ‘x + 13’, if x = 2
work on: (1) 11.2 (2) 9.5 (3) 11.4 (4) 12
(1) Light Scattering (2) Radioactivity
25. A rice trader buys 16 quintals of rice for ` 5632. 20%
(3) Cryogenics (4) Sonometer rice is lost in transportation. At what rate should he
15. J.J. Thomson received the Nobel Prize in physics sell to earn 25% profit?
for the discovery of __________ (1) ` 225.2 per quintal (2) ` 550 per quintal
(1) Protons (2) Electrons
(3) ` 440 per quintal (4) ` 563.2 per quintal
(3) Neutrons (4) Positrons
16. Rehuke khim or ‘cowrie shawl’ are traditional 26. Terms a, 1, b are in Arithmetic Progression and
textiles of terms 1, a, b are in Geometric Progression. Find ‘a’
and ‘b’ given a ≠ b.
(1) Assam (2) Jharkhand
(3) Odisha (4) Nagaland (1) 2, 4 (2) –2, 1 (3) 4, 1 (4) –2, 4
17. Who is the recipient of the Lokmanya Tilak Award 27. ∆ABC is an isosceles right angled triangle having
2023? ∠C = 90°. If D is any point on AB, then AD2 + BD2
(1) Arvind Kejriwal (2) Piyush Goyal is equal to
(3) Amit Shah (4) Narendra Modi (1) CD2 (2) 2CD2 (3) 3CD2 (4) 4CD2
18. The Malimath Committee Report deals with:
(1) Judicial delays 28. A kite flying at a height of 120 m is attached to a
string which makes an angle of 60° at horizontal.
(2) Textile sector reforms
What is the length (in m) of the string?
(3) Criminal justice system reforms
(4) Stock market reforms (1) 90 3 (2) 75 3 (3) 84 3 (4) 80 3
26 OSWAAL CUET (UG ) 10 Mock Test Papers GENERAL TEST

29. Refer the following data table and answer the 37. To do a certain work, A and B work on alternate
question. days, with B beginning the work on the first day.
A can finish the work along in 48 days. If the work
Division/Standard Boys Girls
1
gets completed in 5 days, then B alone can finish
A/5 40 10 3
B/5 35 25 4 times the same work in:
C/5 15 10 (1) 24 days (2) 32 days

A/6 35 35 (3) 27 days (4) 30 days


38. A is as efficient as B and C together. Working
B/6 40 30 together A and B can complete a work in 36 days
C/6 20 20 and C alone can complete it in 60 days. A and C
work together for 10 days. B alone will complete
What is the ratio of boys to girls?
the remaining work in:
(1) 110 days (2) 88 days
(1) 26:37 (2) 39:28
(3) 84 days (4) 90 days
(3) 28:39 (4) 37:26
1 39. Three circles of diameter 10 cm each, are bound
30. If x = 5 + 2 6 , then what is the value of x + ? together by a rubber band, as shown in the figure.
x
(1) 2 3 (2) 3 2 (3) 2 6 (4) 6 2

31. A train without stoppage travels with an average


speed of 80 km/h and with stoppage, it travels
with an average speed of 72 km/h. For how many
minutes does the train stop on an average per
hour? The length of the rubber band, (in cm) if it is

(1) 8 (2) 6 (3) 7 (4) 9 stretched as shown is
32. A train without stoppage travels with an average (1) 30 (2) 30 + 10π
speed of 80 km/h and with stoppage, it travels (3) 10π (4) 60 + 20π
with an average speed of 64 km/h. For how many
minutes does the train stop on an average per 40. The compound interest on ` 4000 for 4 years at 10%
hour? per annum will be
(1) ` 1856.40 (2) ` 1600
(1) 12 (2) 8 (3) 10 (4) 14
(3) ` 1856 (4) ` 1756.60
33. There are two containers of equal capacity. The
ratio of milk to water in the first container is 3 : 1, in 41. Which answer figure will complete the pattern in
the second container 5 : 2. If they are mixed up, the the question figure?
ratio of milk to water in the mixture will be Question Figure:

(1) 28 : 41 (2) 41 : 28 (3) 15 : 41 (4) 41 : 15
34. An article is listed at ` 2375. A man purchases it
at two successive discounts of 50% and 25% and
spends Rs. 165 on repairing of article. If he sells the
article at a profit of 62.5%, what is the selling price
?
(in `) of the article? Answer Figures:

(1) 1467.6 (2) 1492.6 (3) 1715.39 (4) 1467.6
35. The ratio of number of balls in bags x, y is 2:3. Five
balls are taken from bag y and are dropped in (1) (2)
bag x. Number of balls are equal in each bag now.
Number of balls in each bag now is
(1) 45 (2) 20 (3) 30 (4) 25
36. For what sum will the simple interest at R% per (3) (4)
annum for 2 years will be R?
100
(1) ` (2) ` 50
2R 42. If 1st January 2001 was Monday. Then which day of
the week was it on 31 December 2001?
100 200
(3) ` (4) ` (1) Monday (2) Wednesday
R R
(3) Friday (4) Saturday
MOCK TEST Paper-3 27
43. In a certain code language ‘REASON’ is coded as 50. Eight friends A, B, C, D, E, F, G and H are sitting
‘5410129’ and ‘HEALTHY’ is coded as ‘716231216’. around a circular table facing each other for a
How will ‘DIVULGE’ be coded in that language? lunch. A is opposite to D and third to the right of
(1) 10733494 (2) 21435438 B. G is between A and F. H is to the right of A. E is
(3) 13704349 (4) 21434383 between C and D. Who is sitting between B and E?
44. P is the brother of Q. S is the father of P. R is the (1) D (2) E (3) F (4) C
brother of S. If T is the mother of R, then how is P
51. From a point, Lokesh starts walking towards South
related to T?
and after walking 30 metres he turns to his right
(1) Brother (2) Uncle
and walks 20 metres, then he turns right again
(3) Grandson (4) Father
and walks 30 metres. He finally turns to his left
45. Select the suitable option that is related to the third and walk 40 metres. In which direction is he with
number is the same way as the second number is reference to the starting point?
related to the first number.
(1) North-West (2) East
45 : 315 :: 37 : ?

(1) 144 (2) 259 (3) 169 (4) 221 (3) West (4) South

46. In a certain code language, ‘FORENSIC’ is coded 52. Pointing towards a man, Ritika said, “He is the
as ‘61218221419183’ and ‘DORM’ is coded as son of my grandfather’s only son”. How is Ritika
‘4121813’. How will ‘CARAMEL’ be coded in that related to that man?
language? (1) Aunty (2) Sister
(1) 3011801132212 (2) 3260926130512 (3) Mother (4) Wife
(3) 3261826142212 (4) 3261826132212
53. Select the correct option that is related to the third
47. Select the Venn diagram that best illustrates the term in the same way as the second term is related
relationship between the following classes: to the first term.
Parents, Rich persons, Farmers

Innocent : Guilty :: Moisten : ?

(1) Drench (2) Dried
(1) (2)
(3) Desire (4) Wet
54. In a line of boys, Aman is 12th from the top and
Baman is l8th from the bottom. If there are
(4) 6 boys between Aman and Baman, then how many
(3)
maximum boys are there in the row?
48. Read the given statements and Conclusions (1) 34 (2) 36 (3) 35 (4) 37
carefully. Assuming that the information given 55. In the following question, if a mirror is placed on
in the statements is true, even if it appears to be at line AB, then which of the answer figures is the
variance with commonly known facts, decide which right image of the given figure?
of the given Conclusions logically follow(s) from the
A
statements.
Statements: A A A A
A
I. Some cards are postcards. BC CB CB B C
C B
II. Some cards are envelopes. a b c d
III. All envelopes are copies. B
Conclusions: (1) a (2) b (3) c (4) d
I. Some copies are envelopes.
56. Select the correct combination of mathematical
II. Some postcards are copies.
signs to replace ‘*’ signs and to balance the given
III. Some cards are copies.
equation.
(1) Only conclusions II and III follow.
54 * 6 * 5 * 14

(2) Only conclusions I and III follow.
(1) +÷ = (2) = ×+ (3) − = × (4) ÷+ =
(3) Only conclusions I follow.
(4) Only conclusions I and II follow. 57. Question figures
49. Select the correct option that will fill in the blank
and complete the series.
pmtz, rkvx, tixv, vgzt, .........

(1) xebr (2) zdcw (3) xeyv (4) yerb
28 OSWAAL CUET (UG ) 10 Mock Test Papers GENERAL TEST
Answer figures

(3) (4)
(1) (2)

60. In the following question, two statements are


given each followed by two conclusions I and II.
(3) (4) You have to consider the statements to be true
even if they seem to be at variance from commonly
known facts. You have to decide which of the
58. Identify the correct option to complete the given conclusions, if any, follows from the given
following series. statements.
50, 49, 47, 44, 40, 35, ........, 22
Statements:
(1) 30 (2) 27 (I) Many rituals, if incorporated in daily routine
(3) 29 (4) 25 can make positive changes in one’s life.
59. Which answer figure will complete the pattern in (II) Bending forward in mosque or prostrating
the question figure? before God are great exercises as these involve
Question Figure:
backstretch, squat, bend and rise again.
Conclusions:

? (I) Religion offers great scope for exercise.


(II) If these religious rituals are practiced daily,
they can make positive changes in health of an
individual.
(1) Only Conclusion II follows


Answer Figures: (2) Conclusion I and II both follow
(3) Neither I nor II follow
(4) Only conclusion I follows
(1) (2)



Answer Key

1. (4) 2. (2) 3. (3) 4. (3) 5. (4) 6. (3) 7. (3) 8. (3) 9. (2) 10. (4)
11. (4) 12. (2) 13. (4) 14. (1) 15. (2) 16. (4) 17. (4) 18. (3) 19. (4) 20. (4)
21. (2) 22. (4) 23. (2) 24. (3) 25. (2) 26. (4) 27. (2) 28. (4) 29. (4) 30. (1)
31. (2) 32. (1) 33. (4) 34. (3) 35. (4) 36. (2) 37. (3) 38. (1) 39. (2) 40. (1)
41. (4) 42. (1) 43. (2) 44. (3) 45. (2) 46. (4) 47. (2) 48. (2) 49. (1) 50. (1)
51. (3) 52. (2) 53. (2) 54. (2) 55. (1) 56. (4) 57. (3) 58. (3) 59. (4) 60. (2)
MOCK TEST Paper-1 29

MOCK
Test Paper 4
Maximum Marks : 250 Time : 60 Minutes

General Instructions :
(i) This paper consists of 60 MCQs, attempt any 50 out of 60
(ii) Correct answer or the most appropriate answer: Five marks (+5)
(iii) Any incorrect option marked will be given minus one mark (-1)
(iv) Unanswered/Marked for Review will be given no mark (0)
(v) If more than one option is found to be correct then Five marks (+5) will be awarded to only those who have
marked any of the correct options
(vi) If all options are found to be correct then Five marks (+5) will be awarded to all those who have attempted
the question.
(vii) Calculator / any electronic gadgets are not permitted.

1. Which of the following pair is/are correctly 3. Warren Hastings He was the 1st
matched? Governor General of
Dynasty
Regional Area India.
4. Lord Dufferin The formation of INC
1) Ahom Dynasty ASSAM
took place during his
2) Parmar Dynasty MALWA
tenure in 1885.
3) Gurjar Pratihara Dynasty UJJAIN
Choose the correct code.

4) Chandel Dynasty KHAJURAHO (1) 1 only (2) 2 and 3 only
Choose the correct code:
(3) 3 only (4) 3 and 4 only
(1) 1 and 2 only (2) 2 and 3 only
5. Which of the following option is not correct?
(3) 1 and 4 only (4) 1, 2, 3 and 4 1. In terms of geographical expanse, the district
2. Which of the following international organisation of Mahe in Puducherry holds the distinction of
headquarter is not located in New York? being the smallest in India.
(1) United Nations Organisation (UNO) 2. The largest district in India by area is Kutch in
(2) United Nations Children's Emergency Fund Gujarat.
(UNICEF) 3. The second largest district in India by area is
(3) United Nations Security Council (UNSC) Leh.
4. Madhya Pradesh is the largest Indian state in
(4) United Nations Educational Scientific and
terms of area.
Cultural Organisation (UNESCO)
Choose the correct code.

3. Representatives of the Maratha Empire and the
(1) 1 and 2 only (2) 2 only
British East India Company signed the ‘Treaty of
Salbai’ in which year? (3) 4 only (4) 1 and 4 only

(1) 1782 (2) 1769 (3) 1758 (4) 1771 6. Which of the following pair is/are correctly
matched?
4. Which of the following pair is not correctly
matched? Major crops
Top-producing countries:
Viceroy/ Worked during 1. Rice China
Governor General
his rule 2. Corn USA
1. Lord Canning He served as the first 3. Jute India
Viceroy of India. 4. Wheat India
2. Lord Ripon The Vernacular Press Choose the correct option:

Act was repealed in (1) 1, 2 and 3 only (2) 2 and 4 only
1882. (3) 2, 3 and 4 only (4) 1, 2, 3 and 4
30 OSWAAL CUET (UG ) 10 Mock Test Papers GENERAL TEST

7. Which of the following pair is/are correctly (3) Territory under the immediate political control
matched? of another state
1. Fiscal deficit = Total expenditures – Total (4) None of these
receipts excluding borrowings 15. According to Article 361 of the Constitution of
2. Revenue deficit = Total revenue expenditure – India, a criminal proceeding cannot be instituted in
Total revenue receipts a court against the _____________ during his term
3. Primary deficit = Fiscal deficit – Interest of office.
payments (1) Vice-president (2) Prime Minister
Choose the correct option.
(3) Chief Minister (4) Governor
(1) 1 only (2) 2 only
16. Which of the following is the major component of
(3) 2 and 3 only (4) 1, 2 and 3 vinegar?
8. Which of the following pair is INCORRECT? (1) Nitric acid (2) Lactic acid
I. Touch Pad – Locator device (3) Citric acid (4) Acetic acid
II. Microphone – Output device
17. Methane is a colorless. Odorless, non-toxic but
III. Monitor – Output device flammable gas. What is its common name?
(1) I and II (2) I and III (1) Blue vitriol (2) Laughing gas
(3) Only II (4) All options are correct. (3) Heating gas (4) Marsh gas
9. Which angiosperm is vessel less? 18. Match List-I with List-II
(1) Hydrilla (2) Trochodendron List-I (Games)
List-II (Trophies)
(3) Maize (4) Wheat A. Hockey Aga I. Khan Cup
10. Hind limbs thumping on ground by a rabbit is a B. Football II. Begum Hazrat Mahal Cup
behaviour related to C. Table Tennis III. Ramanuja trophy
(1) Courtship
D. Golf IV. Walker cup
(2) Preparation for a duet
Choose the correct answer from the options given

(3) Warning signal to members below:
(4) Surrender to a stronger opponent (1) a-III, b-I, c-IV, d-II (2) a-I, b-II, c-III, d-IV
11. Which of the following option is correct? (3) a-IV, b-I, c-II, d-III (4) a-III, b-II, c-IV, d-I
1. The provision of Bharat Ratna was introduced 19. Where is the satellite launching center of India
in 1954. located?
2. The first Bharat Ratna award was awarded
(1) Ahmedabad (2) Hassan
to politician C.Rajagopalachari, philosopher
Sarvepalli Radhakrishnan, and scientist CV (3) Sriharikota (4) Thumba
Raman in the year 1954. 20. Match list-I with list-II
3. The first two Non-Indians who receive the List-I (State)
List-II (Festivals)
Bharat Ratna are Khan Abdul Ghaffar Khan in A. Andhra Pradesh I. Flamingo Festival
1987 and Nelson Mandela in 1990.
B. Karnataka II. Pongal
4. The award had been awarded to a naturalized
C. Tamil Nadu III. Kambala Festival
Indian citizen Mother Teresa in 1980.
D. Kerala IV. Theyyam Festival
Choose the correct code.

Choose the correct answer from the options given

(1) 1, 3 and 4 only (2) 2, 3 and 4 only
below:
(3) 1, 2 and 3 only (4) 1, 2, 3 and 4
(1) a-III, b-I, c-IV, d-II (2) a-I, b-III, c-II, d-IV
12. The theme of Cancer Day 2023 is
(3) a-IV, b-I, c-II, d-III (4) a-III, b-II, c-IV, d-I
(1) We can, I can (2) Close the care gap
(3) I Am and I Will (4) Not Beyond Us. 21. 6 1 % of 1600 + 12 1 % of 800 equals
4 2
13. Laser was invented by
(1) 100 (2) 200 (3) 300 (4) 400
(1) A.H. Taylor (2) T.H. Maiman
(3) Lee De Forest (4) Thomas Edison
22. A shop-keeper sold a sewing machine for ` 1,080 at
a loss of 10%. At what price should he has to sold it
14. Coup is ______________. so as to gain 10% on it? (in `)
(1) Sudden overthrow of a government illegally
(1) 1,069 (2) 1,200 (3) 1,230 (4) 1,320
(2) A system of rules that taken effect when a
military authority takes control of normal 23. From a point P on the ground, the angle of
administration of justice. elevation of the top of a 10 m high building is 30°.
MOCK TEST Paper-4 31
A flag is hoisted at the top of the building and the 34. Refer the following data table and answer the
angle of elevation of the top of the flagstaff from question.
the point P is 45°. The length of the flagstaff is:
Marks Number of students
(Take 3 = 1.732)

40 and above 25
(1) 7 m (2) 7.24 m (3) 7.56 m (4) 7.32 m 30 and above 44
24. For the data 11, 15, 13, 12, 10, 8, 11, 7, 15, 11, 13, 7, 20 and above 61
11, which of the following is true?
10 and above 88
(1) Mean = Median = Mode
0 and above 105
(2) Mean > Median > Mode
(3) Mean > Mode = Median How many students have scored marks 20 or more

but less than 40?
(4) Mean < Median < Mode
(1) 44 (2) 61 (3) 36 (4) 105
25. Two successive discounts of 10% and 5%, in this
order, are given on a bill of ` 110. Find the net 35. If the square of the sum of two numbers is equal
amount of money payable to clear the bill. (answer to 4 times of their product, then the ratio of these
to the nearest rupee) numbers is :

(1) ` 94 (2) ` 95 (3) ` 96 (4) ` 97 (1) 2 : 1 (2) 1 : 3 (3) 1 : 1 (4) 1 : 2

26. The sum of money that will yield ` 60 as simple 36. If each side of an equilateral triangle is 12 cm then
interest at the rate of 6% per annum in 5 years is its altitude is equal to:

(1) ` 200 (2) ` 225 (3) ` 175 (4) ` 300 (1) 6 2 cm (2) 3 2 cm

27. In a stream running at 3 km/h, a motorboat goes (3) 6 3 cm (4) 3 6 cm


12 km upstream and back to the starting point in 37. The average weight of L, M and N is 93 kg. If the
60 minutes. Find the speed of the motorboat in still average weight of L and M be 89 kg and that of
water, (in km/h). M and N be 96.5 kg, then the weight (in kg) of M
is_____________.
(1) 2( 2 + 17 ) (2) 2( 4 + 15 )
(1) 92 (2) 86 (3) 101 (4) 95
(3) 3( 2 + 17 ) (4) 3( 4 + 17 ) 38. A train covers a distance between station A and
station B in 45 minutes. If the speed of the train
28. In a school, 391 boys and 323 girls have been is reduced by 5 km/hr, then the same distance is
divided into the largest possible equal classes, so
covered in 48 minutes. The distance between
that each class of boys numbers the same as each
station A and B is
class of girls. What is the number of classes?
(1) 60 km (2) 64 km (3) 80 km (4) 55 km
(1) 23 (2) 19 (3) 44 (4) 17
39. The compound interest on a certain sum of money
29. A and B together can do a piece of work in 9 days. for 2 years at 5% is ` 328, then the sum is
If A does thrice the work of B in a given time, the
time A alone will take to finish the work is (1) ` 3000 (2) ` 3600 (3) ` 3200 (4) ` 3400
(1) 4 days (2) 6 days (3) 8 days (4) 12 days 40. An alloy contains copper, zinc and nickel in the
ratio of 5 : 3 : 2. The quantity of nickel (in kg) that
30. A canal of a village can be cleaned by 24 villagers in must be added to 100 kg of this alloy to have the
12 days. The number of days in which 36 villagers
new ratio 5 : 3 : 3 is
can clean the canal is
(1) 8 (2) 10 (3) 12 (4) 15
(1) 18 (2) 8 (3) 72 (4) 16
41. Select the Venn diagram that best illustrates the
31. If x  3  5 , then the value of x2 – 16x + 6 is relationship between the following classes:
Graduates, Literates, Hardworking

(1) 0 (2) – 2 (3) 2 (4) 4

32. The digit in the unit place in the square root of


(1) (2)
66049 is
(1) 3 (2) 7 (3) 8 (4) 2
33. In an isosceles ∆ABC, AD is the median to the
unequal side meeting BC at D. DP is the angle
bisector of ∠ADB and PQ is drawn parallel to BC (3) (4)
meeting AC at Q.Then the measure of ∠PDQ is:
(1) 130° (2) 90° (3) 180° (4) 45°
32 OSWAAL CUET (UG ) 10 Mock Test Papers GENERAL TEST

42. Read the given statements and conclusions 47. Preeti is the only sister of Virat. Manoj is the son of
carefully. Assuming that the information given Dileep, who is the husband of Maaya. Virat is the son
in the statements is true, even if it appears to be of Devansh. Devansh is Manoj’s father-in-law. How
at variance with commonly known facts, decide is Maaya related to Preeti?
which of the given Conclusions logically follow(s)
(1) Daughter-in-law (2) Mother-in-law
from the statements.
(3) Mother (4) Sister in-law
Statements:
I. Some bowls are cups. 48. Select the suitable option that is related to the third
number in the same way as the second number is
II. All cups are glasses.
related to the first number.
III. Some glasses are plates.
All plates are utensils. 22 : 242 :: 18 : ?

Conclusions: (1) 275 (2) 169 (3) 162 (4) 221
I. All bowls cannot be utensils. 49. In a certain code language, “who are you” is
II. All glasses cannot be utensils. written as “432”, “they is you” is written as “485”
(1) Only conclusion I follows. and “they are dangerous” is written as “295”. How
(2) Both the conclusions follow. is “dangerous” written in that code language?
(3) Only conclusion II follows. (1) 2 (2) 4 (3) 5 (4) 9
(4) Neither conclusion I nor II follows. 50. Select the letter that will come next in the following
series.
43. Choose the most appropriate mirror image of the
figure. O, R, U, X, A, .....

(1) D (2) C (3) B (4) E
51. Given below are two statements: one is labelled as
Assertion (A) and the other is labelled as Reason
(R).
Assertion (A): Leakages in household gas cylinders

can be detected.
Reason (R): LPG has a strong smell.

In the light of the above statements, choose the

(1) (2)
most appropriate answer from the options given
below:
(1) Both (A) and (R) are correct and (R) is the
correct explanation of (A)
(2) Both (A) and (R) are correct but (R) is not the
(3) (4) correct explanation of (A)
(3) (A) is correct but (R) is not correct
(4) (A) is not correct but (R) is correct
44. Select the option in which the word shares the 52. If ‘P’ means ‘subtracted from’, ‘X’ means ‘multiplied
same relationship as that shared by the given pair by’, ‘Y‘ means ‘added to’, and ‘Z’ means ‘divided
of words by’, then: 54 Z 3 Y 22 X 5 = ?
Turmeric Spices (1) 135 (2) 128 (3) 110 (4) 144
(1) Fig Cashew Nuts 53. X and Y both start from a same point X walks 17
(2) Dates Dry Fruits m West, then turns to his right and walks 13 m. At
(3) Apricot Pear the same time, Y walks 9 m North, then turns East
(4) Nutmeg Chili Flakes and walks 7 m, then turns to his left and walks 4 m.
45. In a certain code language, ‘RAJ’ is coded as ‘87’ Where is Y now with respect to the position of X?
and ‘GITA’ is coded as ‘148’. How will ‘VARUN’ be (1) 24 m West (2) 10 m East
coded in that language? (3) 10 m West (4) 24 m East
(1) 234 (2) 380 (3) 176 (4) 403 54. Select the correct option that will fill in the blank
46. In a row of cars, blue car is at the bottom end. There and complete the series.
are 18 cars between yellow and blue car. There are
T, R, P, N, L, ...., .......

8 cars between red and blue car. If there are 6 cars
above yellow car, then how many minimum cars are (1) J, H (2) K, I (3) J, G (4) K, H
there in the row? 55. Which answer figure will complete the pattern in
(1) 34 (2) 26 (3) 32 (4) 35 the question figure?
MOCK TEST Paper-4 33
Question Figure:
Conclusions:
(I) Pollution is resulting into rise in temperature of
the world.
(II) Pollution is the only reason for depletion of the
nature.

? (1) Only Conclusion II follows


(2) Conclusion I and II both follows
(3) Neither I nor II follows
Answer Figures:
(4) Only Conclusion I follows
57. Select the correct option that is related to the third
term in the same way as the second term is related
to the first term.
(1) (2)
Lion : Roar :: Elephant :?

(1) Bark (2) Bray
(3) Trumpet (4) Grunt
58. Select the correct option that will fill in the blank
and complete the series.
(3) (4) 2, 8, 32, 128, 512, ?

(1) 2507 (2) 2536 (3) 2067 (4) 2048
59. A paper boy cycles 10 km East, then turns South
and cycles 3 km, then turns West and cycles 6 km,
56. In the following question, two statements are then turns to his right and cycles 3 km. Where is he
given each followed by two conclusions I and II. now with reference to his starting position?
You have to consider the statements to be true
(1) 4 km West (2) 14 km East
even if they seem to be at variance from commonly
known facts. You have to decide which of the (3) 4 km East (4) 14 km West
given conclusions, if any, follows from the given 60. Pointing to a lady, Anup said, “She is the only
statements. daughter of the lady who is the mother of my
Statements: mother’s only grandson”. How is the lady which
(I) Glaciers are receding and the temperature rise of is pointed related to Anup?
the world is more than last year this time. (1) Daughter (2) Wife
(II) Pollution level of the Earth is rising a lot
(3) Sister (4) Mother
resulting in avalanches.


Answer Key

1. (4) 2. (3) 3. (1) 4. (3) 5. (3) 6. (2) 7. (4) 8. (3) 9. (2) 10. (3)
11. (4) 12. (2) 13. (2) 14. (1) 15. (4) 16. (4) 17. (4) 18. (2) 19. (3) 20. (2)
21. (2) 22. (4) 23. (4) 24. (3) 25. (1) 26. (1) 27. (4) 28. (4) 29. (4) 30. (2)
31. (3) 32. (2) 33. (2) 34. (3) 35. (3) 36. (3) 37. (1) 38. (1) 39. (3) 40. (2)
41. (1) 42. (4) 43. (1) 44. (2) 45. (2) 46. (2) 47. (2) 48. (3) 49. (4) 50. (1)
51. (4) 52. (2) 53. (4) 54. (1) 55. (1) 56. (3) 57. (3) 58. (4) 59. (3) 60. (1)
34 OSWAAL CUET (UG ) 10 Mock Test Papers GENERAL TEST

MOCK
Test Paper 5
Maximum Marks : 250 Time : 60 Minutes

General Instructions :
(i) This paper consists of 60 MCQs, attempt any 50 out of 60
(ii) Correct answer or the most appropriate answer: Five marks (+5)
(iii) Any incorrect option marked will be given minus one mark (-1)
(iv) Unanswered/Marked for Review will be given no mark (0)
(v) If more than one option is found to be correct then Five marks (+5) will be awarded to only those who have
marked any of the correct options
(vi) If all options are found to be correct then Five marks (+5) will be awarded to all those who have attempted
the question.
(vii) Calculator / any electronic gadgets are not permitted.

1. Which of the following statement is/are correct? 4. Which of the following is NOT a twin city of India?
1. When the ruler of Avanti, Pradyota suffered (1) Delhi and New Delhi
from Jaundice bimbisara sent royal physician
(2) Durgapur and Asansol
Jivaka.
2. The famous weapons Mahasilakantaka and (3) Pune and Pimpri-Chinchwad
Rathamusala were introduced by Ajatshatru. (4) Durg and Bhilai
3. Udayin made Pataliputra the capital of 5. What will be the focal length when human eye
Magadha and also built a fort at the confluence lens becomes thin?
of the Ganges and Son at Patna.
(1) Increases (2) Decreases
4. The second Buddist council was held at Vaishali
during kallashoka reign. (3) Remain same (4) No option is correct
Choose the correct code. 6. Which of the following is/are correct?
(1) 1 only (2) 2 and 4 only 1. The 42nd Amendment to the Indian constitution,
(3) 3 and 4 only (4) 1, 2, 3 and 4 enacted in 1976, established Administrative
2. Which of the following is/are correct? Tribunals in India.
1. X-Rays were discovered by Wilhelm Conrad 2. The 24th constitutional amendment made it
Roentgen in 1895. obligatory for the president to grant assent to a
2. JJ Thomson, while investigating cathode rays constitutional amendment bill.
in the 19th century, discovered the electron. 3. The 44th constitutional amendment removed
3. Ernest Rutherford discovered protons in 1917 the right to property from the list of
during his gold foil experiment. fundamental rights.
Choose the correct code.
4. The First Amendment was passed in 1951 by
(1) 1 only (2) 1, 2 and 3
the Provisional Parliament, whose members
(3) 2 and 3 only (4) 3 only had just completed drafting the Constitution as
3. Match list I with list II part of the Constituent Assembly.
List I (Wind) List II (Location)
Choose the correct code.

1. Foehn A. The Alps
(1) 1 and 2 only (2) 2, 3 and 4 only
2. Sirocco B. Sahara to the Mediterranean
(3) 1, 2 and 4 only (4) 1, 2, 3 and 4
Sea
3. Chinook C. The Rockies mountains 7. According to the NITI Aayog, how many people
4. Mistral D. The Alps and France moved out of multidimensional poverty between
2015-16 and 2019-21?
Choose the correct code.
(1) 1-A, 2-B, 3-C, 4-D (2) 1-B, 2-C, 3-B, 4-D (1) 10 crore (2) 12 crore
(3) 1-D, 2-B, 3-C, 4-A (4) 1-C, 2-D, 3-B, 4-A (3) 13.5 crore (4) 15 crore
MOCK TEST Paper-5 35
8. Which of the following is/are correct? 13. Which of the following international organisation
1. National Electronic Funds Transfer (NEFT) headquarter is not located in Geneva?
is a nationwide centralized payment system (1) United Nations Conference on Trade and
owned and operated by the Reserve Bank of Development (UNCTAD)
India (RBI). (2) World Health Organisation (WHO)
2. IFSC or Indian Financial System Code is an (3) International Labour Organisation (ILO)
11 digits alpha-numeric code that uniquely (4) Organisation for the Prohibition of Chemical
identifies a bank branch participating in the Weapons
NEFT system. 14. The award given for outstanding performance in
3. MICR Code or Magnetic ink character sports is
recognition(MICR) is a 9 digits code printed (1) Dronacharya Award
on the cheque which enables identification of (2) Bharat Ratna
the cheques and identifies the bank and branch (3) Padma Shri Award
participating in an Electronic Cheque Clearing
(4) Arjuna Award
system.
4. Swift Code (Society for Worldwide 15. ‘Agha Khan Cup’ is related with which of the
following sports?
Interbank Financial Telecommunication) is
an internationally recognized system for (1) Football (2) Cricket
identifying banks and financial institutions (3) Hockey (4) Table Tennis
globally. 16. Radar is used to:
Choose the correct code. (1) Locate submerged submarines.
(1) 1 and 4 only (2) 1 and 3 only (2) Receive signal from radio receivers.
(3) 2 and 3 only (4) 1, 2, 3 and 4 (3) Detect and locate distant objects.
9. Which of the following is TRUE? (4) Locate geostationary satellites.
(1) Primary memory is non-volatile 17. Which of the following acid found in Apple?
(2) Secondary memory is volatile. (1) Malic Acid (2) Nitric Acid
(3) ROM is non-volatile memory. (3) Formic Acid (4) Sulphuric Acid
(4) RAM is non-volatile memory. 18. Which of the following has a strong fruity
10. In which communication mode, data can be fragrance?
transmitted in both directions at same time? (1) Methyl chloride (2) Methanoic acid
(1) Simplex (2) Full duplex (3) Methanol (4) Ethyl acetate
(3) Half duplex (4) Multiplex V 19. ‘World Ozone Day’ is celebrated every year on
11. Clove, the commonly used spice, is obtained from ______ September.
the: (1) 8 (2) 16 (3) 12 (4) 6
(1) root (2) stem 20. Bhavai is a traditional dance form of ____________
(3) flower bud (4) fruit (1) Gujarat (2) Bihar
12. Match List I with List II (3) Punjab (4) Maharashtra
List I List II 21. A man sold 20 apples for ` 100 and gained 20%.
(Neighbouring (Bordering How many apples did he buy for ` 100?
Country) States)
(1) 20 (2) 22 (3) 24 (4) 25
A. China I. Jammu and Kashmir
(POK Part) 22. If the arithmetic mean and the geometric mean
of the roots of a quadratic equation are 8 and 5
B. Bangladesh II. West Bengal, Mizoram, respectively, then the quadratic equation is:
Meghalaya, Tripura,
(1) x2 – 8x + 5 = 0 (2) x2 – 8x + 25 = 0
and Assam
C. Bhutan III. Arunachal Pradesh, (3) x2 – 16x + 25 = 0 (4) 2x2 – 8x + 25 = 0
Assam, Sikkim, and 23. The sum of squares of three natural numbers
West Bengal is 1029, and they are in the proportion 1 : 2 : 4.
D. Afghanistan IV. Jammu and Kashmir, The difference between greatest number and the
Himachal Pradesh, smallest number is:
Uttrakhand, Sikkim, (1) 21 (2) 15 (3) 18 (4) 31
and Arunachal Pradesh 24. The length of the shadow on the ground of a tall
Choose the correct code. tree of height 30 m is 10 3 m. What is the angle (in
(1) A-1, B-2, C-3, D-4 (2) A-4, B-2, C-3, D-1 degrees) of elevation of the sun?
(3) A-1, B-3, C-2, D-4 (4) A-4, B-3, C-2, D-4 (1) 60 (2) 15 (3) 30 (4) 45
36 OSWAAL CUET (UG ) 10 Mock Test Papers GENERAL TEST

25. A can do as much work in 4 days as B can do in 5 cc) that should be added to the mixture so that the
days, and B can do as much work in 6 days as, C new ratio of the volumes of water and glycerine
in 7 days. In what time will C do a piece of work become 2 : 3 is
which A can do in a week? (1) 55 cc (2) 60 cc (3) 62.5 CC (4) 64 cc
5 4
(1) 10 days (2) 4 days 34. What is the unit digit of the sum of first 111 whole
24 5
numbers?
8 6
(3) 6 days (4) 12 days (1) 4 (2) 6 (3) 5 (4) 0
15 19
35. The average weight of X, Y and Z is 74 kg. if the
26. If 10-digit number 6712y76x2 is divisible by 88 then
average weight of X and Y be 68 kg. and that of
the value of (7x – 2y)
Y and Z be 78 kg. then the weight (in kg) of Y
(1) 10 (2) 7 (3) 3 (4) 5 is____________.
27. A train, 150 m long, passes a pole in 15 seconds and (1) 72 (2) 70 (3) 68 (4) 66
another train of the same length travelling in the
opposite direction in 12 seconds. The speed of the 36. A piece of wire when bent to form a circle will
second train is have a radius of 84 cm. If the wire is bent to form a
(1) 45 km./hr (2) 48 km./hr square, the length of a side of the square is
(3) 52 km./hr (4) 54 km./hr (1) 152 cm (2) 132 cm (3) 168 cm (4) 225 cm
28. Let x be the least number which when divided by 37. The area of a triangle is 15 sq. cm and the radius if
15, 18, 20 and 27, the remainder in each case is 10 its incircle is 3 cm. Its perimeter is equal to:
and x is a multiple of 31. What least number should
(1) 12 cm (2) 20 cm (3) 5 cm (4) 10 cm
be added to x to make it a perfect square?
1 1 1 1
(1) 39 (2) 37 (3) 43 (4) 36 38. The value of + + + ........ +
1+ 2 2+ 3 3+ 4 8+ 9
29. If ABC is an equilateral triangle and D is a point on
1 1 1 1
BC such that AD ⊥ BC, then + + + ........ + is
(1) AB : BD = 1 : 1 1 + 2 2
(2) AB : BD = 1 : 2 + 3 3 + 4 8 + 9

(3) AB : BD = 2 : 1 (4) AB : BD = 3 : 2 (1) 1 (2) 0 (3) 2 (4) 2


30. In a big garden 60% of the trees are coconut trees, 39. If two successive discounts of 20% and 30% are
25% of the number of coconut trees are mango given, what is the net discount (in %)?
trees and 20% of the number of mango trees are (1) 40 (2) 44 (3) 56 (4) 60
apple trees. If the number of apple trees are 1500,
then the number of trees in the garden is: 40. A principal of ` 10,000, after 2 years compounded
annually, the rate of interest being 10% per annum
(1) 48000 (2) 50000 (3) 51000 (4) 45000
during the first year and 12% per annum during
31. Refer the following data table and answer the the second year (in rupees) will amount to:
question.
(1) ` 12,000 (2) ` 12,320
Years GDP growth rate for the (3) ` 12,500 (4) ` 11,320
year (in %)
41. Pointing to the photograph of Sanchi, Nitin said,
2011 –7
“Her mother’s father’s son’s wife is my mother-
2012 –5 in-law’s only daughter”. How is Nitin related to
2013 7 Sanchi’s mother?
2014 4 (1) Brother (2) Paternal Uncle
2015 –4 (3) Maternal Uncle (4) Grandfather

If the GDP of the country was $3 trillion at the end


42. Select the Venn diagram that best illustrates the
of 2013, what was it in the beginning of 2015? relationship between the following classes:
(1) $3.21 trillion (2) $2.88 trillion Doctors, Salaried persons, Parents

(3) $3.12 trillion (4) $3 trillion


(1) (2)
32. If x2 – 3x + 1 = 0, then the value of x 2 + x + 1 + 12 is
x x
(1) 10 (2) 2 (3) 6 (4) 8
33. The ratio of the volume of water and glycerine in (3) (4)
240cc of a mixture is 1 : 3. The quantity of water (in
MOCK TEST Paper-5 37
43. Read the given statements and conclusions Choose the correct answer from the options given

carefully. Assuming that the information given below :
in the statements is true, even if it appears to be at (1) (a), (c), (d), (e), (b) (2) (e), (b), (c), (d), (a)
variance with commonly known facts, decide which
(3) (a), (c), (d), (b), (e) (4) (d), (e), (b), (a), (c)
of the given conclusions logically follows from the
statements. 48. Select the option in which the numbers are related
Statements:
in the same way as are the numbers in the given
I. All lockets are chains. set.
II. 100% medals are lockets. (11, 165, 209)
III. Some rings are chains. (1) (15, 225, 275) (2) (12, 180, 228)
Conclusions:
(3) (14, 210, 276) (4) (17, 245, 323)
I. All chains are medals. 49. In the following diagram, the triangle represents
II. All medals being rings is a possibility. doctors, the circle represents players and the
(1) Only conclusion II follows. rectangle represents singers.
(2) Neither conclusion I nor II follows.
(3) Only conclusion I follows.
A
(4) Both conclusions I and II follows.
B C D
44. Which answer figure will complete the pattern in
the question figure?
Question Figure:

Which region represents doctors who are singers

but not players?
(1) A (2) B (3) C (4) D
50. A, B, C, D, E and F are sitting on the round table
? with equal distances. F is sitting opposite E and
between A and D. C is sitting right side of E and
opposite to A. Who are the neighbours of A?
Answer Figures:
(1) F and D (2) E and F
(3) E and C (4) B and F
51. Vikram started from point R and walked straight
(1) (2) 7 km West, then turned left and walked 2 km and
again turned left and walked straight 4 km. In
which direction is he from R?
(1) North-East (2) South-West
(3) South-East (4) North-West
(3) (4) 52. Question figure

45. Select the option that is related to the third word in



the same way as the second word is related to the
Answer figures
first word.
Jackal : Howl : Rain : ?
(1) Hustle (2) Thunder (1) (2)
(3) Patter (4) Drops
46. Sunil’s position from the left in a row of boys is 20th
and Deepak’s position from the right is 36th. After
interchanging their position, Sunil becomes 28th
from the left. How many boys are there in the row? (3) (4)
(1) 52 (2) 63 (3) 59 (4) 48
47. Arrange the given words in the sequence in which 53. In the following question, two statements are
they occur in the dictionary. given each followed by two conclusions I and II.
(a) Amphibian (b) Amorphous You have to consider the statements to be true
(c) Amphidextrous (d) Ambiguous even if they seem to be at variance from commonly
(e) Ambivalent known facts. You have to decide which of the
38 OSWAAL CUET (UG ) 10 Mock Test Papers GENERAL TEST
given conclusions, if any, follows from the given
statements.
Statements:
(I) Life is a journey with few known people, few (4)
(3)
partially known and mostly strangers.
(II) In a journey, the importance of destination
is as equal as the pathway of reaching that
destination. 56. Select the related word pair from the given
Conclusions: alternatives.
(I) Importance of people in life is considerable, as Teeth : Cut :: ....... : .........

they are our journey mates.
(II) Life teaches us many lessons to face various (1) Legs : Walk (2) Broom : Fly
known and unknown difficulties. (3) Tubelight : Heat (4) Food : Smell
(1) Only Conclusion II follows 57. In a row of cars, Maruti is 20th from the left end of
(2) Conclusion I and II both follow the row. Honda is 10th to the right from Maruti and
(3) Neither I nor II follow is at the exact center of the row. How many cars are
(4) Only Conclusion I follows there in the row?
54. If ‘P’ means ‘−’, ‘Q’ means ‘÷’, ‘R’ means ‘+’, and (1) 54 (2) 59 (3) 57 (4) 56
‘S’ means‘×’, then: 525 Q 25 R 42 S 4 = ? 58. Arsh is Shivam’s father and Dhruv is the son of
(1) 178 (2) 187 (3) 189 (4) 198 Bimla. Eshwar is the father of Arsh. If Shivam is the
55. Which answer figure will complete the pattern in brother of Dhruv, how is Bimla related of Eshwar?
the question figure? (1) Sister-in-law (2) Mother
Question Figure:
(3) Daughter-in-law (4) Wife
59. Select the term that will come next in the following
? series.
BMO, EOQ, HQS, KSU, ...........

(1) MTV (2) NUW (3) NTV (4) NVW
60. Given below are numbers in the first line and
symbols in the second line. Numbers and symbols

are codes are each other. Choose the correct code
Answer Figures:
for given symbols.
1 2 3 4 5 6 7 8 9
〈 ∗ 〉 ∆ ◊

(2)
(1)
Given:
〈 ◊
(1) 91486 (2) 91846 (3) 94816 (4) 94846



Answer Key

1. (4) 2. (2) 3. (1) 4. (1) 5. (1) 6. (4) 7. (3) 8. (4) 9. (3) 10. (2)
11. (3) 12. (2) 13. (4) 14. (4) 15. (3) 16. (3) 17. (1) 18. (4) 19. (2) 20. (1)
21. (3) 22. (3) 23. (1) 24. (1) 25. (1) 26. (2) 27. (4) 28. (1) 29. (3) 30. (2)
31. (3) 32. (1) 33. (2) 34. (3) 35. (2) 36. (2) 37. (4) 38. (3) 39. (2) 40. (2)
41. (1) 42. (2) 43. (1) 44. (3) 45. (3) 46. (2) 47. (4) 48. (2) 49. (1) 50. (4)
51. (2) 52. (4) 53. (4) 54. (3) 55. (1) 56. (1) 57. (2) 58. (3) 59. (2) 60. (1)
MOCK TEST Paper-1 39

MOCK
Test Paper 6
Maximum Marks : 250 Time : 60 Minutes

General Instructions :
(i) This paper consists of 60 MCQs, attempt any 50 out of 60
(ii) Correct answer or the most appropriate answer: Five marks (+5)
(iii) Any incorrect option marked will be given minus one mark (-1)
(iv) Unanswered/Marked for Review will be given no mark (0)
(v) If more than one option is found to be correct then Five marks (+5) will be awarded to only those who have
marked any of the correct options
(vi) If all options are found to be correct then Five marks (+5) will be awarded to all those who have attempted
the question.
(vii) Calculator / any electronic gadgets are not permitted.

1. Which of the following option is not correct related 4. During which of the following session did the
to the Nanda dynasty? All India Muslim League adopt the Pakistan
1. It was founded by Mahapadma Nanda around Resolution on 23 March 1940?
467 BC.
(1) Karachi Session (2) Lahore Session
2. Dhanananda was the last ruler of the Nanda
Dynasty and was overthrown by Chandra (3) Lucknow Session (4) Nagpur Session
Gupta Maurya. 5. Which city of India is known as ‘The Athens of The
3. Alexander invaded India in 326 BC when Dana East’?
Nanda was the ruler of the Nanda dynasty.
(1) Allahabad (2) Patna
Choose the correct code.
(3) Madurai (4) Kochi
(1) 1 only (2) 2 and 3 only
(3) 1 and 3 only (4) 1, 2 and 3 6. Which river passes through maximum number of
2. Which of the following is the correct chronological countries?
order of the following events of Indian history? (1) Rhine (2) Danube
1. Lucknow Pact (3) Amazon (4) Volga
2. Formation of the Swaraj Party 7. Which of the following state government launched
3. Jallianwala Massacre Nand Baba Milk Mission?
4. Gandhi Irwin pact
(1) Uttar Pradesh (2) Madhya Pradesh
Choose the correct code.
(3) Bihar (4) Chhatisgarh
(1) 1324 (2) 2341 (3) 1234 (4) 2134
3. Which of the following is/are correctly matched 8. Which Indian state launched the Namo Shetkari
Types Of Clouds Features Mahasanman Yojana?
1. Stratus These are layered clouds (1) Punjab (2) Maharashtra
covering large portions of (3) Assam (4) Haryana
the sky.
9. In which year was the first-ever motion to remove
2. Nimbus They form at middle levels
or very near to the surface a Supreme Court Justice signed, by 108 members of
of the earth. the Parliament?
3. Cumulonimbus These are the “thunderheads” (1) 1984 (2) 1991 (3) 1978 (4) 1996
that can be seen on a warm
10. The first stable product of photosynthesis is
summer day and can bring
strong winds, hail, and rain. (1) Starch
Choose the correct code. (2) Sucrose
(1) 1 only (2) 2 only (3) Phosphoglyceric acid
(3) 1 and 3 only (4) 1, 2 and 3 (4) Glucose
40 OSWAAL CUET (UG ) 10 Mock Test Papers GENERAL TEST

11. Which of the following enzyme pairs is not 22. The mean of scores obtained by 50 students is
functional in an acidic environment? found to be 79.5. Later on, it was found that the
(1) Trypsin and Pepsin score of one student was read as 94 in place of 49
(2) Trypsin and Amylase and the score of another student was read as 69 in
place of 89. Find the correct mean.
(3) Chymotrypsin and Pepsin
(4) Pepsin and Amylase. (1) 85.52 (2) 79.35 (3) 79 (4) 78
12. How many nations have officially attained 23. If an 11-digit number 5y 5884805x6 where x ≠ y is
recognition from the World Health Organization divisible by 72. Then the value of xy is:
(WHO) that they have successfully eliminated
trachoma as a public health concern? (1) 7 (2) 3 (3) 7 (4) 2 7
(1) 14 (2) 16 (3) 18 (4) 20 24. Speed of boat along and against the current are
13. Which among the following was the first satellite 14 km/h and 8 km/h respectively. The speed of the
launched by Indian Space Research Organization? current is
(1) Bhaskar (2) Aryabhatta (1) 11 h (2) 6 h
(3) Rohini (4) INSAT-1A
(3) 5.5 h (4) 3 h
14. Which of the following is NOT a vocal form of
Hindustani classical music?
25. If two numbers are in the ratio 2 : 3 and the ratio
becomes 3 : 4 when 8 is added to both the numbers,
(1) Pakhawaj (2) Drupad
then the sum of the two numbers is
(3) Tarana (4) Dhamar
(1) 10 (2) 80 (3) 40 (4) 100
15. Who, among the following individuals, has
recently achieved the distinction of becoming the 26. A man travelled a distance of 80 km in 7 hrs partly
first-ever Indian to secure an individual World on foot at the rate of 8 km per hour and partly on
Title at the World Archery Championships? bicycle at 16 km per hour. The distance travelled on
(1) Rakhi Sharma (2) Deepika Kumari the foot is
(3) Savita Rathi (4) Aditi Swami (1) 32 km (2) 48 km (3) 36 km (4) 44 km
16. Ability to distinguish two closely placed objects is 27. The sum of an Infinite geometric series is 4 and the
(1) Resolving power (2) Video power sum of the cubes of the terms of the same GP is 192.
(3) Distinguish power (4) Magnifying power The Common Ratio of the original geometric series
17. Which of the following particles were made to fall is:
on a thin gold foil by Ernest Rutherford?
(1) 1/2 (2) –1/2 (3) 1/4 (4) –1/4
(1) Gamma (2) Beta
(3) Electron (4) Alpha 28. The least number which is exactly divisible by 4, 5,
8, 10 and 12 is:
18. Which gas is used as a fuel in a rocket?
(1) Radon (2) Hydrogen (1) 150 (2) 240 (3) 180 (4) 120
(3) Chlorine (4) Nitrogen 29. If 9 x  12  147 then x = ?
19. Which of the following is/are correctly matched? (1) 2 (2) 3 (3) 4 (4) 5
Days and events
30. The base and altitude of an isosceles triangle are 10
1. 2 February – World Wetlands Day cm and 12 cm respectively. Then the length of each
2. 22 March – World Water Day equal side is:
3. 7 April – World Health Day
(1) 10 cm (2) 7.5 cm (3) 8.5 cm (4) 13 cm
4. 22 April – World Earth Day
Choose the correct code. 31. If the perimeter of an isosceles right triangle is
(1) 1 and 4 only (2) 2, 3 and 4 only ( 8 2 + 1) cm, then the length of the hypotenuse of
(3) 1, 2 and 3 only (4) 1, 2, 3 and 4 the triangle is:

20. Who invented the jet engine? (1) 8 cm (2) 12 cm (3) 10 cm (4) 24 cm
(1) Karl Benz (2) Sir Frank Whittle 32. The fourth root of 24010000 is
(3) Thomas Savery (4) Michael Faraday (1) 7 (2) 49 (3) 490 (4) 70
21. Average of 36 results is 18. If 2 is subtracted from 33. Find the wrong number in the following number
each result what will be the new average of the
series.
results?
3 7
16 35 70 153
(1) 9 (2) 16 (3) 17 (4) 17.5
(1) 70 (2) 16 (3) 153 (4) 35
MOCK TEST Paper-6 41
34. Simplify: 43. Two statements are given, followed by three
conclusions numbered I, II and III.
 0.73    0.27 
3 3

Assuming the statements to be true, even if they



 0.73    0.27    0.73    0.27 
2 2

seem to be at variance with commonly known


(1) 1 (2) 0.4047 (3) 0.73 (4) 0.27 facts, decide which of the conclusions logically
follow(s) from statements.
35. When a discount of 25% is given on a cruise trip,
the profit is 41%. If the discount is 26%, then the Statements:
profit is I. All photographs are newspapers.
(1) 39.12 per cent (2) 67 per cent II. Some photographs are computers.
(3) 94.88 per cent (4) 11.24 per cent Conclusions:
I. Some computers are photographs.
36. A circular road runs around a circular ground. If
the difference between the circumference of the II. Some computers are newspapers.
outer circle and the inner circles is 66 metres, the III. Some newspapers are photographs.
width of the road is: (1) All of the conclusions follow.
22 (2) Only conclusions I and II follow.
(Take π =
)
7 (3) Only conclusions II and III follow.
(1) 10.5 metres (2) 7 metres (4) Only conclusions I and III follow.
(3) 5.25 metres (4) 21 metres 44. Select the correct option that will fill in the blank
37. If the area of a square is 24 sq units then what is the and complete the series.
perimeter of the square? Mhz, Ngy, Ofx, Pew, ............

(1) 2 6 (2) 4 6 (3) 16 6 (4) 8 6 (1) Qrm (2) Rrn (3) Qdv (4) Qqn

38. Two toys are sold at ` 504 each. One toy brings the 45. Find the number of rectangles in the below figure
dealer a gain of 12% and the other a loss of 4%. The
gain or loss per cent by selling both the toys is
5
(1) 3 5 % Profit (2) 4 % Profit
13 13
1 3
(3) 5 % Profit (4) 2 % loss
13 13
39. A father can do a job as fast as his two sons working (1) 25 (2) 125 (3) 225 (4) 55
together. If one son does the job in 3 hours and the
other in 6 hours, the number of hours taken by the 46. Select the potion in which the words share the
father, to do the job alone is same relationship as that shared by the given pair
of words.
(1) 1 (2) 2 (3) 3 (4) 4
Hive : Bee
40. In what time will `8,000, at 3% per annum, produce (1) Burrow : Hare (2) Eyrie : Bear
the same interest as `6,000 does in 5 years at 4%
simple interest? (3) Sty : Dog (4) Stable : Cow
(1) 5 years (2) 6 years (3) 3 years (4) 4 years 47. In a certain code language, ‘apple’ is called ‘pear’,
‘pear’ is called ‘orange’, ‘orange’ is called ‘guava’
41. Identify the correct option to complete the and ‘guava’ is called ‘melon’. In this language,
following series. which one of the following will be a citrus fruit?
1, 3, 5, 11, 21, 43, 85, 171, .............

(1) Pear (2) Melon (3) Guava (4) Orange
(1) 354 (2) 247 (3) 341 (4) 342
48. Hitesh, Sunny, Vicky, Nitin and Bharat are
42. Choose the Venn diagram that best illustrates the arranged in ascending order of the height from
relationship among the following classes: the top. Hitesh is at third place. Bharat is between
Women, Entrepreneurs, Engineers
Nitin and Hitesh while Nitin is not at the bottom.
Who has the maximum height among them?
(1) Hitesh (2) Sunny (3) Vicky (4) Nitin
(1) (2) 49. Select the option in which the numbers are related
in the same way as are the numbers in the given
set.
(269, 278, 296)
(3) (4)
(1) (109, 118, 128) (2) (577, 586, 598)
(3) (313, 322, 340) (4) (419, 430, 448)
42 OSWAAL CUET (UG ) 10 Mock Test Papers GENERAL TEST

50. In a code language, if ‘you are there’ is written as


‘ter der jer’, ‘we stay here’ is written as ‘yer mer
(3) (4)
ner’, ‘we are late’ is written as ‘ser ner der’, and
‘I stay there’ is written as ‘yer fer jer’, then how
would ‘you stay late’ be written in this language? 55. If ‘÷‘ means ‘+‘, ‘−‘ means ‘ב, ‘ב means ‘−‘, and
(1) ter yer ser (2) ter yer mer ‘+‘ means ‘÷‘, then: 664 + 4 ÷ 34 × 28 =?
(3) der yer ser (4) ter mer ser (1) 165 (2) 162 (3) 172 (4) 185
51. Which answer figure will complete the pattern in 56. In the following question, two statements are
the question figure? given each followed by two conclusions I and II.
Question Figure: You have to consider the statements to be true
even if they seem to be at variance from commonly
known facts. You have to decide which of the
? given conclusions, if any, follows from the given
statements.
Statements:
(I) Corruption has complete establishment in each
Answer Figures:
sector of our country.
(II) Corruption has gone into their deep roots of
every process and system.
(1) (2) Conclusions:
(I) People think about corruption in a routine way
and not especially.
(II) The eradication of corruption depends upon
every individual. No government can enforce
(3) (4) rules or regulation on people.
(1) Only conclusion II follows.
(2) Conclusion I and II both follow.
(3) Neither I nor II follow.
52. In a dice, 1, 2, 3, and 4 are written on the adjacent
faces, in a clockwise order, and 5 and 6 at the top (4) Only conclusion I follow.
and bottom. When three is at the top, what will be 57. Select the related word pair from the given
at the bottom? alternatives.
Celsius : Temperature :: ........... : ..........
5
(1) Kilometer : Distance
(2) Bank note : Office
4
1 (3) Meter : Water
(4) Bulb : Electric current

(1) 4 (2) 2 (3) 6 (4) 1


58. There are five students - P, Q, R, S and T having
different heights in a class. P’s height is more than
53. Select the option that will correctly replace the only one student. Q’s height is more than S and P but
question mark (?) in the series. not more than R. S’s height is more than P. R is not the
RMB, POZ, NQX, LSV, ?
smallest. Who is having the maximum height in the
(1) JUT (2) JVT (3) JUS (4) JVS class.
54. Find out which of the answer figures can be formed (1) Q (2) R (3) S (4) T
from the pieces given in problem figure. 59. A%B’ means ‘A is the mother of B’.
Question figure ‘A $ B’ means ‘A is the father of B’.
‘A #B’ means ‘A is the brother of B’.
‘A & B’ means ‘A is the sister of B’.
If J $ H # R % N & T # U % P,
then which of the following statements is NOT
correct?
Answer figures

(1) J is the maternal grandfather of N.
(2) R is the maternal grandfather of P.
(1) (2)
(3) R is the mother of U.
(4) N is the sister of P.
MOCK TEST Paper-6 43
60. Arrange the following words in the correct order Choose the correct answer from the options given

(a) Omelete (b) Hen below:
(c) roll (d) serve (1) b, e, a, d, c (2) e, b, a, c, d
(e) egg (3) b, e, a, c, d (4) b, a, e, c, d



Answer Key

1. (4) 2. (1) 3. (4) 4. (2) 5. (3) 6. (2) 7. (1) 8. (2) 9. (2) 10. (3)
11. (2) 12. (4) 13. (2) 14. (1) 15. (4) 16. (1) 17. (4) 18. (2) 19. (4) 20. (2)
21. (2) 22. (3) 23. (3) 24. (4) 25. (3) 26. (1) 27. (2) 28. (4) 29. (2) 30. (4)
31. (1) 32. (4) 33. (1) 34. (1) 35. (1) 36. (1) 37. (4) 38. (1) 39. (2) 40. (1)
41. (3) 42. (3) 43. (1) 44. (3) 45. (1) 46. (1) 47. (3) 48. (4) 49. (3) 50. (1)
51. (1) 52. (3) 53. (1) 54. (1) 55. (3) 56. (3) 57. (1) 58. (2) 59. (3) 60. (1)
44 OSWAAL CUET (UG ) 10 Mock Test Papers GENERAL TEST

MOCK
Test Paper 7
Maximum Marks : 250 Time : 60 Minutes

General Instructions :
(i) This paper consists of 60 MCQs, attempt any 50 out of 60
(ii) Correct answer or the most appropriate answer: Five marks (+5)
(iii) Any incorrect option marked will be given minus one mark (-1)
(iv) Unanswered/Marked for Review will be given no mark (0)
(v) If more than one option is found to be correct then Five marks (+5) will be awarded to only those who have
marked any of the correct options
(vi) If all options are found to be correct then Five marks (+5) will be awarded to all those who have attempted
the question.
(vii) Calculator / any electronic gadgets are not permitted.

1. Which of the following is/are correctly matched? 4. Which type of federalism is seen in India?
Chinese traveller
visited the court (1) Coming together federation

1. Fa Hein Chandragupta II (2) Holding together federation


(3) C
oming together federation and Holding
2. Hiuen Tsang Harsha Vardhana
together federation both
3. I-Tsing Chandragupta I
(4) None of these
Choose the correct code
5. Which of the following is/are correctly matched?
(1) 2 only (2) 1 and 3 only Articles
Provisions
(3) 1 and 2 only (4) 1, 2 and 3 1. Article 360 Financial Provisions
2. Which of the following is/are not correctly 2. Article 324 Election Commission of India
matched? 3. Article 161 Pardoning powers of Governor
Territories
Year of Creations 4. Article 143 Advisory jurisdiction of the
1. Puducherry 1954 Supreme Court
2. Dadra and Nagar Haveli 1961 Choose the correct code.

3. Lakshadweep 1956 (1) 1 and 4 only (2) 2 and 3 only
4. Delhi 1969 (3) 1,2 and 4 only (4) 1, 2, 3 and 4

Choose the correct code.



6. Which of the following seas is Earth’s lowest
elevation on land and is located at the border of
(1) 1 only (2) 4 only Israel and Jordan?
(3) 1 and 4 only (4) 2 and 3 only (1) Red Sea (2) Black Sea
3. Which of the following statement is/are correct? (3) Dead Sea (4) Arabian Sea
1. Clement Attlee held the position of Prime 7. To which of the following Indian states does the
Minister of England at the time of India's tribe of ‘Nyishi’ belong?
independence. (1) Tamil Nadu (2) Bihar
2. Within the context of India's Freedom (3) Arunachal Pradesh (4) Chhattisgarh
Movement, James Ramsay MacDonald made 8. Which of the following is/are correctly matched?
the declaration of the Communal Award in Tribe
Locations
1932. 1. Masai Kenya
Choose the correct code.
2. Bedouin Arabia
(1) 1 only (2) 2 only 3. Hausa Northern Nigeria
(3) Both 1 and 2 (4) Neither 1 nor 2 4. Bushmen China
MOCK TEST Paper-7 45
Choose the correct code.
16. Which of the following reaction is an example of
(1) 1 and 2 only (2) 2 and 3 only exothermic reaction?
(3) 1, 2 and 3 only (4) 1, 2, 3 and 4 (1) Burning of natural gas
9. When the fiscal deficit is high, what happens to (2) Respiration
prices? (3) 
The decomposition of vegetable matter into
(1) Prices decrease compost
(2) There is no direct impact on prices (4) All Options are correct
(3) Prices remain stable 17. What is the chemical name of the deadly ‘cyanide’?
(4) Prices increase (1) Sulfuric acid (2) Hydrochloric acid
10. Which of the following is NOT a logic gate? (3) Nitric acid (4) Prussic acid
(1) AND (2) OR 18. Which of the following pair is/are correctly
(3) NAND (4) MAD matched?
11. All the arithmetic and logical operations in a Instruments Functions
computer are done by __________
1. Sonometer It is used to measure
(1) ALU (2) CU
blood pressure.
(3) Register (4) No option is correct
2. Hydrometer It is an instrument
12. Amino acids such as citrulline, ornithine, and
that measures that
gamma-aminobutyric acid are referred to as:
specific gravity
(1) non-protein amino acids (relative density)
(2) essential amino acids of liquids.
(3) protein amino acids 3. Sphygmomanometer The instrument
(4) acidic amino acids used to study the
13. Which of the following is/are correct? laws of vibrating
1. The pancreas releases enzymes that assist string.
in the process of digestion and also produce 4. Electrometer It is used to measure
hormones that play a role in regulating sugar electric charge or
metabolism. electrical potential
2. The main function of the liver is the production difference.
of bile.
Choose the correct code.

3. The small intestine is the principal organ for
the absorption of nutrients. (1) 1 and 3 only (2) 2 and 4 only
4. Absorption of water, some minerals, and drugs (3) 1 and 2 only (4) 1, 2, 3 and 4
takes place in the large intestine. 19. Which of the following is not a folk dance belonging
Choose the correct code.
to the union territory of Jammu and Kashmir?
(1) 2 and 3 only (2) 2 and 4 only (1) Dumhal (2) Hafiza
(3) 2, 3 and 4 only (4) 1,2, 3 and 4 (3) Rouf (4) Dangi
14. Which one among the following administers the 20. Who is the first Indian Gold Medal Winner in
International Development Association (IDA)?
individual category in Olympic Games?
(1) UNDP (2) UNIDO (3) IFAD (4) IBRD
(1) Abhinav Bindra (2) Sushil Kumar
15. Which of the following is/are not correctly
matched? (3) R. V. S. Rathore (4) Leander Paes
1. Padma Bhushan: Third-highest civilian honour 21. Which of the following successive discounts is
in India. better to a customer?
2. Bharat Ratna: The highest civilian award in (a) 20%, 15%, 10% (b) 25%, 12%, 8%
India. (1) (a) is better
3. Padma Vibhushan: It is the second-highest (2) (b) is better
civilian honour in India.
(3) (a) or (b) (both are same)
4. Padma Shri: It is the fourth-highest civilian
honour in India. (4) None of these
Choose the correct option.
22. If the mean of 5 observations x, x + 2, x + 4, x + 6
(1) 2 and 3 only (2) 1 and 4 only and x + 8 is 11, then value of x:
(3) 2 and 4 only (4) None of the above (1) 7 (2) 43 (3) 51 (4) 8
46 OSWAAL CUET (UG ) 10 Mock Test Papers GENERAL TEST

23. If a certain sum becomes two times in 5 years at If the data related to the production of cars of
compound interest, then in how many years, it will type E is represented by a pie chart, then the
become eight times? central angle of the sector representing the data of
(1) 10 (2) 20 (3) 15 (4) 30 production of cars in 2013 will be:
24. What is the least value of x such that 517x324 is (1) 102 (2) 84 (3) 70 (4) 80
divisible by 12? 34. The radius of a wheel is 21 cm. What is the distance
(1) 3 (2) 1 (3) 0 (4) 2 (in cm) travelled by the wheel in 10 revolutions?
(1) 660 (2) 1320 (3) 1980 (4) 2640
25. A bag contains 6 red, 4 blue, and 10 white balls. A
ball is picked from the bag at random What is the 35. In a triangle, if the measures of two sides are 5 cm
probability that it is neither white nor blue? and 8 cm then the third side can be:
(1) 20/6 (2) 3/10 (3) 2/5 (4) 1/2 (1) 3 cm (2) 4 cm (3) 14 cm (4) 2 cm
36. A man bought oranges at the rate of 8 for ` 34 and
26. The average weight of a class of 50 students is 48.6 sold them at the rate of 12 for ` 57. How many
kg. If the average weight of the 20 boys is 54 kg.
Then find the average weight (in kg) of the girls in oranges should be sold to earn a net profit of ` 45?
the class. (1) 90 (2) 100 (3) 135 (4) 150
(1) 40 (2) 46 (3) 45 (4) 42 37. A person bought 50 pens for ` 50 each. He sold 40
of them at a loss of 5%. He wants to gain 10% on
27. What is the least number which when divided by the whole. Then his gain percent on the remaining
15, 18 and 36 leaves the same remainder 9 in each
pens should be
case and is divisible by 11?
(1) 15% (2) 40% (3) 50% (4) 70%
(1) 1269 (2) 1089 (3) 1080 (4) 1071
38. Two vessels contain milk and water in the ratio 3 : 2
28. 18 persons working 8 hours a day can complete 3 and 7 : 3. Find the ratio in which the contents of the
units of works in 10 days. How many persons are two vessels have to be mixed to get a new mixture
required to complete 5 units of that work in 16 in which the ratio of milk and water is 2 : 1.
days working 6 hours a day?
(1) 2 : 1 (2) 1 : 2 (3) 4 : 1 (4) 1 : 4
(1) 25 (2) 15 (3) 20 (4) 9
39. The sum of three numbers is 280. If the ratio
between the first and second numbers is 2 : 3 and
29. If a + 2b + a − 2b = 3 , then a : b is equal to the ratio between second and third numbers is 4 :
a + 2b − a − 2b
5, then find the second number.
(1) 2 : 3 (2) 3 :4 (3) 3:2 (4) 4 : 3 (1) 80 (2) 90 (3) 86 (4) 96
30. ∆ABC is an equilateral triangle. O is the point of 40. In simple interest rate per annum a certain sum
intersection of altitudes AL, BM and CN. If OA = amounts to ` 5,182 in 2 years and ` 5,832 in 3 years.
16 cm, what is the semi-perimeter (in cm) of the The principal in rupees is
triangle ABC?
(1) ` 2882 (2) ` 5000 (3) ` 3882 (4) ` 4000
(1) 8 3 (2) 12 3 (3) 16 3 (4) 24 3
41. A + B means ‘B is the daughter of A’.
31. Twenty one times of a positive number is less that its A – B means ‘B is the sister of A’.
square by 100. The value of the positive number is
A × B means ‘B is the husband of A’.
(1) 25 (2) 26 (3) 42 (4) 41
A ÷ B means ‘A is the father of B’.
1
32. If 2 x  a  and a  0 , then the value of If P ÷ R × T + Q – S × U + Z,
a Then how is R related to Z?
x2 − 1 (1) Paternal grandfather
is
x − x2 − 1 (2) Maternal grandfather
1 (3) Paternal grandmother
(1) a + 1 (2)  a  1 (4) Maternal grandmother
2
1
(3)  a  1 (4) a – 1 42. Choose the Venn diagram that best illustrates the
2 relationship among the following classes:
33. The table shows the production of different types Crocodiles, Aquatic animals, Reptiles
of cars (in thousands). Study the table carefully
and answer the questions.
Years/ (1) (2)
2012 2013 2014 2015 2016
Cars
A 30 35 48 45 56
B 42 48 40 38 56
C 48 36 38 35 44 (3) (4)
D 51 24 30 46 54
E 20 42 40 35 43
MOCK TEST Paper-7 47
43. Two statements are given, followed by two 51. In a certain code language. ‘YOU TOOK MY PLACE
conclusions numbered I and II. Assuming the IN’ is written as ‘COU POON LC QVAKE IJ’. How
statements to be true, even if they seem to be at
will ‘COMPILE’ be written in that language?
variance with commonly known facts, decide
which of the conclusions logically follow(s) from (1) KOLQIME (2) KOLQIVE
statements. (3) KOLTYLE (4) YOLTILE
Statements:
I. No plant is a tree.
52. Which image will appear when the following
II. All ornaments are plants. transparent sheet will be folded.
Conclusions:
I. No ornament is a tree.
II. Some plants are ornaments.
(1) Both conclusions follow.
(2) Only conclusion I follows.
(3) Only conclusion II follows.
(4) Either conclusion I or II follows.
44. In the following diagram, the police represent (1) (2)
a circle, corrupt represents a triangle, the poet
represents a square, and married represents a
rectangle. The number representing unmarried
police officers who are not corrupt but are poets is

11 (3) (4)
1
10

5 53. In the following question, two statements are


2 13 4 6 given each followed by two conclusions I and
12 3 II. You have to consider the statements to be
8 7 true even if they seem to be at variance from
9 commonly known facts. You have to decide
which of the given conclusions, if any, follows
(1) 8 (2) 9 (3) 2 (4) 4 from the given statements.
45. Time appears in the mirror at 11 : 09. Then, what Statements:
time it will appear on the clock? (I)  Wages are determined by minimum wage
(1) 1 : 51 (2) 12 : 09 (3) 12 : 51 (4) 1 : 09 legislation in each country.
46. If ‘L’ means ‘×’, ‘N’ means ‘+’, ‘O’ means ‘÷’, and (II) Increase in wage rate is the sign of growth in
an economy.
‘M’ means ‘−’, then: 1000 O 5 N 66 M 33 = ?
Conclusions:
(1) 433 (2) 333 (3) 483 (4) 233
(I) Average wage rate signifies the wealth of a
47. Beverages’ is related to ‘Tea’, in the same way as country.
‘Medicine’ is related to (II) M inimum wage legislation does not account
(1) Analgesics (2) Disease wage imbalances.
(1) Only conclusion II follows.
(3) Doctor (4) Hospital
(2) Conclusion I and II both follow.
48. In a code language, if ‘fast and furious’ is written as (3) Neither I nor II follow.
‘co mo jo’, ‘do it fast’ is written as ‘cha mo ga’, and (4) Only conclusion I follows.
‘she did it’ is written as ‘ga la nop’, then what will
54. Find out which of the answer figures can be formed
be the code for the word ‘do’ in this language? from the pieces given in problem figure.
(1) ga (2) mo (3) nop (4) cha Question figure
49. In a row of 74 girls, Shweta is 27th from the left end.
Palak is 7th to the right of Shweta. What is Palak’s
position from the right end of the row?
(1) 40 (2) 41 (3) 42 (4) 44
50. Select the option in which the numbers do not
share the same relationship as that shared by the Answer figures
given triad of numbers.
(23, 115, 207)
(1) (2)
(1) (8, 35, 72) (2) (6, 30, 54)
(3) (16, 80, 144) (4) (21, 105, 189)
48 OSWAAL CUET (UG ) 10 Mock Test Papers GENERAL TEST

56. Pointing to a lady in the photograph, Amit said, “She


is the mother of the only grandson of my mother”.
(4) (4)
How is the lady related to Amit?
(1) Mother (2) Daughter
55. Which answer figure will complete the pattern in (3) Niece (4) Wife
the question figure? 57. Sunita travels 6 km towards North-East. Then travels
Question Figure:
9 km towards West. From there, she goes 6 km South-
West and finally goes 3 km towards East. How far is
she from her initial position?
(1) 3 km (2) 6 km (3) 9 km (4) 12 km
58. Select the pair of letters that will follow next in the
? given series.
AB, GH, MN, ST, ?

Answer Figures:
(1) YZ (2) VW (3) UV (4) XY
59. Pearl Tower is taller than Sky Tower but shorter
than Unity Tower. Unity Tower and Cyber Tower
are of the same height. Pearl Tower is shorter than
(1) (2)
Indus Tower. Amongst the buildings, which is the
second shortest?
(1) Pearl Tower (2) Sky Tower
(3) Indus Tower (4) Unity Tower
60. Select the correct option that will fill in the blank
(3) (4) and complete the series.

12, 12, 13, 26, 28, 84, ?
(1) 93 (2) 87 (3) 84 (4) 95


Answer Key

1. (3) 2. (2) 3. (3) 4. (2) 5. (3) 6. (3) 7. (3) 8. (3) 9. (2) 10. (4)
11. (1) 12. (1) 13. (4) 14. (4) 15. (4) 16. (4) 17. (4) 18. (2) 19. (4) 20. (1)
21. (2) 22. (1) 23. (3) 24. (4) 25. (2) 26. (3) 27. (2) 28. (1) 29. (4) 30. (4)
31. (1) 32. (3) 33. (2) 34. (2) 35. (2) 36. (1) 37. (4) 38. (2) 39. (4) 40. (3)
41. (4) 42. (2) 43. (1) 44. (1) 45. (3) 46. (4) 47. (1) 48. (4) 49. (2) 50. (1)
51. (2) 52. (3) 53. (4) 54. (2) 55. (2) 56. (4) 57. (2) 58. (1) 59. (1) 60. (2)
MOCK TEST Paper-1 49

MOCK
Test Paper 8
Maximum Marks : 250 Time : 60 Minutes

General Instructions :
(i) This paper consists of 60 MCQs, attempt any 50 out of 60
(ii) Correct answer or the most appropriate answer: Five marks (+5)
(iii) Any incorrect option marked will be given minus one mark (-1)
(iv) Unanswered/Marked for Review will be given no mark (0)
(v) If more than one option is found to be correct then Five marks (+5) will be awarded to only those who have
marked any of the correct options
(vi) If all options are found to be correct then Five marks (+5) will be awarded to all those who have attempted
the question.
(vii) Calculator / any electronic gadgets are not permitted.

1. Which of the following is not correctly matched? 1. Best Feature Film: Rocketry

Vedic River Name


Modern River Name 2. Best Popular Film Providing Wholesome
Entertainment: RRR
1. Shutudri Sutlej
3. Best Actor: Allu Arjun, Pushpa
2. Parushini Ravi
3. Askini Chenab 4. Best Actress: Alia Bhatt, Gangubai Kathiawadi
and Kriti Sanon, Mimi
4. Vitasta Beas
Choose the correct code.

Choose the correct code.

(1) 1 only (2) 2, 3, and 4 only
(1) 1 only (2) 2 and 4 only
(3) 1, 2 and 4 only (4) 1, 2, 3 and 4
(3) 4 only (4) 2, 3 and 4 only
2. Which of the following groups of countries is 5. What is the government’s goal under the “lakhpati
a member of the South Asian Association for didis” scheme?
Regional Cooperation (SAARC) and Bay of (1) To create two crore prosperous sisters in
Bengal Initiative for Multi-Sectoral Technical and villages.
Economic Cooperation (BIMSTEC)? (2) To provide skill training to women in villages.
(1) Bangladesh, Bhutan, India (3) To help women earn over ` 1 lakh per year.
(2) Bangladesh, Maldives, India (4) All of the above.
(3) Pakistan, Bhutan, India 6. ____________ expansion makes the Eiffel Tower
(4) Afghanistan - Bhutan - Maldives taller during summers.
3. Which of the following is correctly matched? (1) Thermal (2) Gradient
Constitutional Bodies
First Chairman (3) Gravitational (4) Chemical
1. Election Commission of India Sukumar Sen 7. Among all the millets, which of the following is the
2. Finance Commission of India K. C. Niyogi most abundantly cultivated in India?
3. Comptroller General of India V. Narhari Rao (1) Pearl millet (2) Sorghum
4. Attorney General of India M. C. Stelvad (3) Ragi (4) Foxtail millet
Choose the correct code.
8. Which of the following is the largest fish-exporting
(1) 1, 2 and 4 only (2) 2 and 4 only region of the world?

(3) 1, 3 and 4 only (4) 1, 2, 3 and 4 (1) The north-east Atlantic region

4. Which of the following pair is/are correctly (2) The north-east Pacific region
matched regarding the 69th National Film Awards (3) The north-west Pacific region
2023? (4) The south-east Asian region
50 OSWAAL CUET (UG ) 10 Mock Test Papers GENERAL TEST

9. Which of the following pair is/are correctly Choose the correct code.

matched? (1) 1-A 2-B 3-C 4-D (2) 1-B 2-A 3-C 4-D
Tribes Native State (3) 1-A 2-D 3-C 4-B (4) 1-C 2-A 3-B 4-D
1. Hatti tribe Himachal Pradesh 16. Marfati songs are traditional folk songs of
2. Narikoravan tribe Tamilnau ____________
3. Toda tribe Tamilnadu (1) Afghanistan (2) Bangladesh
4. Binjhia tribe Jharkhand
(3) Pakistan (4) Nepal
Choose the correct code.
17. Veteran freedom fighter, social reformer and
(1) 1 and 2 only (2) 2 and 4 only
feminist Savitribai Phule hailed from which of the
(3) 1, 2, and 4 only (4) 1, 2, 3 and 4 following states of India?
10. Which among the following is used to transmit
(1) Rajasthan (2) Gujarat
data in optical fibre?
(3) Maharashtra (4) Odisha
(1) Vibrations (2) Light
(3) Sound (4) Electrical current 18. In which of the following state Archaeological
Survey of India discover a 1300-year-old Buddhist
11. Which among the following is a male part of a
flower? Stupa?
(1) Stigma (2) Stamen (3) Style (4) Pistil (1) Bihar (2) Jharkhand
12. Which of the following is/are not correctly (3) Odisha (4) Madhya Pradesh
matched? 19. Which of the following substances is used as a
Dates Important Days substitute for chlorine in bleaching?
1. 10 January World Hindi Day (1) Ethanal
2. 12 January National Youth Day (2) Liquid carbon dioxide
3. 24 January National Girl Child Day (3) Hydrogen peroxide
4. 30 January Martyrs Day or Shaheed
(4) Tetra chloroethene
Diwas
20. Which of the following metals in pure form has the
Choose the correct code.
highest melting point?
(1) 1 and 2 only (2) 1, 2, 3 and 4
(1) Tungsten (2) Copper
(3) 2 and 4 only (4) None of the above
(3) Platinum (4) Gold
13. Who among the following is the first Indian to
swim across English Channel? 21. In what proportion must a grocer mix sugar at ` 12
(1) Aarti Saha (2) Mihir Sen per kg and ` 7 per kg so as to make a mixture worth
(3) P. K. Bannerji (4) Vikram Merchant ` 8 per kg?
14. Match the following (1) 7 : 12 (2) 1 : 4 (3) 2 : 3 (4) 12 : 7

Scientist Discovery 22. Please read the following carefully and answer the
questions: The marks of 7 students in a unit test are
1. Louis Pasteur A. Fermentation
given below 9, 10, 7, 6, 9, 3, 5 Median of the data is.
2. Edward Jenner B. Small Pox Vaccine
(1) 6 (2) 10 (3) 9 (4) 7
3. James Watson and C. DNA Structure
23. The single discount equal to three consecu­tive
Francis Crick
discounts of 10%, 12% and 5% is
4. Alexander Fleming D. Penicillin
(1) 26.27% (2) 24.76% (3) 9% (4) 11%
Choose the correct code.

24. If the six digit number 4x573y is divisible by 72 then
(1) 1-A 2-B 3-C 4-D (2) 1-B 2-A 3-C 4-D
the value of (x + y) is:
(3) 1-D 2-C 3-A 4-B (4) 1-C 2-A 3-B 4-D
(1) 9 (2) 4 (3) 8 (4) 6
15. Match the following:
25. A sum of ` 2000 amounts to ` 4000 in two years
Leaders Nicknames at compound interest. In how many years will the
1. Bal Gangadhar A. Tilak Father of same amount become ` 8000?
Indian Unrest (1) 2 (2) 4 (3) 6 (4) 8
2. Dr Rajendra Prasad B. Deshbandhu 26. A pair of dice is thrown. What is the probability of
3. Dadabhai Naoroji C. Grand old man of getting the sum of numbers on the two faces an
India odd-prime number?
4. Chittaranjan Das D. Desh Ratna (1) 7/18 (2) 1/6 (3) 1/2 (4) 5/12
MOCK TEST Paper-8 51

x3 3 37. Let ABC be an equilateral triangle and AX, BY, CZ


27. If 1   , then x equals be the altitudes. Then the right statement out of
100 5
the four given responses is
(1) 2 (2) 4 (3) 16 (4) (136)1/3 (1) AX = BY = CZ (2) AX ≠ BY = CZ
28. The table shows the production of different types (3) AX = BY ≠ CZ (4) AX ≠ BY ≠ CZ
of cars by a company (in thousands) in 5 years.
Study the table and answer the questions. 38. ABC is an equilateral triangle and CD is the internal
bisector of ∠C. If DC is produced to E such that AC
Car/ = CE, then ∠CAE is equal to
A B C D E
Year
(1) 45° (2) 75° (3) 30° (4) 15°
2014 52 54 48 46 64
2015 47 45 53 50 45
39. A can complete a piece of work in 18 days, B
in 20 days and C in 30 days. B and C together
2016 48 47 56 54 65 start the work and are forced to leave after
2017 43 50 57 67 63 2 days. The time taken by A alone to complete the
2018 38 40 54 68 70 remaining work is
(1) 10 days (2) 12 days

If the data related to the production of cars in 2018
is represented by pie chart then the central angle (3) 15 days (4) 16 days
of the sector representing the production of type C
cars will be: 40. If N = 7 − 3 , then what is the value of N + 1 ?
7+ 3 N
(1) 72° (2) 59° (3) 93° (4) 91°
29. A number is increased by 84, it becomes 107% of (1) 2 2 (2) 5 (3) 10 (4) 13
itself. What is the number? 41. Select the term that will come next in the following
(1) 600 (2) 900 (3) 1500 (4) 1200 series.
a, cd, fgh, jklm, opqrs, ........

30. The greatest number of four digits which when
(1) uvwxyz (2) vwxyza
divided by 3, 5, 7, 9 leave remainders 1, 3, 5, 7
respectively is: (3) ouvwsy (4) tuvwxy

(1) 9763 (2) 9764 (3) 9766 (4) 9765 42. Select the Venn diagram that best illustrates the
relationship between the following classes:
31. What is the sum of the first 17 terms of an arithmetic
Graduates, Teachers, Literates

progression if the first term is –20 and last term is
28?
(1) 68 (2) 156 (3) 142 (4) 242 (1) (2)
32. A person has to travel a distance of 30 km. He finds
5
that he has covered th of the distance in 3 hours
6 is his speed in kmph?
and 20 minutes. What
(1) 5.4 (2) 7.5 (3) 6.3 (4) 6 (3) (4)
33. A circular wire of length 168 cm is cut and bent in
the form of a rectangle whose sides are in the ratio 43. Two statements are given, followed by three
of 5 : 7. What is the length (in cm) of the diagonal conclusions numbered I, II and II. Assuming the
of the rectangle? statements to be true, even if they seem to be at
(1) 4127 (2) 3137 (3) 1813 (4) 3626 variance with commonly known facts, decide
which of the conclusions logically follow(s) from
34. A sold a horse to B for ` 4800 by losing 20%. B sells
statements.
it to C at a price which would have given A a profit
Statements:
of 15%. B’s gain is
I. No tree is a plum.
(1) ` 1800 (2) ` 1900 (3) ` 2000 (4) ` 2100
II. All pomegranates are trees.
35. A man sold an article at a gain of 5%. Had he sold it Conclusions:
for ` 240 more, he would have gained 8%. The cost
I. No pomegranate is a plum.
price of the article is
II. No plum is a tree.
(1) ` 6,000 (2) ` 10,000 III. Some trees are pomegranates.
(3) ` 12,000 (4) ` 8,000 (1) Only conclusions I and III follow.
36. The number to be added to each of the numbers 7, (2) Only conclusions II and III follow.
16, 43, 79 to make the numbers in proportion is (3) All the conclusions follow.
(1) 2 (2) 3 (3) 5 (4) 1 (4) Only conclusions I and II follow.
52 OSWAAL CUET (UG ) 10 Mock Test Papers GENERAL TEST

44. Five monkeys A, B, C, D and E are sitting on a 50. In a class, P has more marks than Q and R does
branch of a tree. C is sitting next to D. D is not not have the least marks. S has more marks than
sitting with E. E is on the left end of the branch. T and T has more marks than P, who among them
C is on second position from right. A is immediate will have the least marks?
right of B and B is on the right side of E. A and are
(1) P (2) Q (3) S (4) T
sitting together. Who is sitting at the centre?
(1) D (2) C (3) A (4) B 51. Read details carefully for questions
45. Which of the four changes given in the options Tennis
would make the following equation correct? Cricket 9
17
30 × 3 − 3 = 13
12 15 8
(1) × Interchanged with − 21
Hockey 6
(2) × changed to ÷; − changed to +
(3) × changed to +; − changed to ÷ How many people like Cricket and Tennis both?

(4) × Interchanged with −; 3 interchanged with 30 (1) 17 (2) 32 (3) 15 (4) 27
46. In the following question, two statements are 52. In a certain code language. ‘DRONE’ is written as
given each followed by two conclusions I and II.
‘SERFO’. How will ‘HOUSE’ be written as in that
You have to consider the statements to be true
language?
even if they seem to be at variance from commonly
known facts. You have to decide which of the (1) PIXFT (2) QJXGB
given conclusions, if any, follows from the given (3) PQWGS (4) PIVFT
statements.
53. Rohan walks a distance of 3 km towards North,
Statements: then turns to his left and walks for 2 km. He again

(I) Demonetization has evolved need for learning turns left and walks for 3 km. At this point he turns
electronic payment system (EPS). to his left and walks for 3 km. How far is he from

(II) EPS will enable people to go cashless and use the starting point?
plastic money for most of their transactions. (1) 5 km (2) 3 km (3) 2 km (4) 1 km
Conclusions:
54. Identify the correct option to complete the

(I) EPS is quick, safe, and easy way of transaction following series.
of money from one account to another.
5, 8, 13, 20, 29, ........., 53, 68, 85

Cashless transactions will further curb the
(II) 
(1) 40 (2) 39 (3) 35 (4) 36
black marketing of money in upcoming time.
(1) Only conclusion II follows 55. Which answer figure will complete the pattern in
the question figure?
(2) Conclusion I and II both follow
(3) Neither I nor II follow Question Figure:

(4) Only conclusion I follows
47. Select the correct option that will fill in the blank
and complete the series.

AA, BD, CI, DP, ......
(1) EY (2) FR (3) ER (4) FY
48. What is the meaningful/logical order of the words Answer Figures:

given below?
(a) Type (b) Save
(c) Open (d) Print (1) (2)
(e) Close
Choose the correct answer from the options given

below.
(1) (c), (a), (d), (b), (e) (2) (a), (c), (b), (d), (e)
(3) (c), (a), (b), (d), (e) (4) (a), (c), (d), (b), (e)
(3) (4)
49. In a certain code language, ‘FIGURE’ is written as
‘DJEVPF’. How will ‘DECODE’ be written as in
that language?
(1) BFAPBF (2) BFDNBF 56. Find out which of the answer figures can be formed
(3) BDDNBD (4) BDANBD from the pieces given in problem figure.
MOCK TEST Paper-8 53
Question figure
58. Rana walks 20 metre straight in North direction
and 20 m to the right. Then every times turning to
his left he walks 5, 25 and 25 metres respectively.
How far is he from his starting point now?
(1) 5 metre (2) 20 metre
(3) 25 metre (4) 30 metre
Answer figure

59. A + B means ‘B is the sister of A’.
A – B means ’A is the father of B’.

(1) (2)
A × B means ‘B is the son of A’.

A ÷ B means ‘B is the husband of A’.

If E ÷ F – H + J × G + D, then how is D related to

H?
(3) (4)
(1) Aunt (2) Father (3) Uncle (4) Niece
60. Select the option that is related to the third term in
57. Introducing Kavi, Veena Said, “She is the sister of the same way as the second term is related to the
the son of the wife of my husband”. How is Veena first term.
related to Kavi? India : Country :: Delhi : ..........

(1) Daughter (2) Sister (1) Continent (2) District
(3) Mother (4) Aunt (3) Capital (4) Red Fort


Answer Key

1. (3) 2. (1) 3. (1) 4. (4) 5. (4) 6. (1) 7. (1) 8. (1) 9. (4) 10. (2)
11. (2) 12. (4) 13. (2) 14. (1) 15. (3) 16. (2) 17. (3) 18. (3) 19. (3) 20. (1)
21. (2) 22. (4) 23. (2) 24. (3) 25. (2) 26. (1) 27. (2) 28. (1) 29. (4) 30. (1)
31. (1) 32. (2) 33. (4) 34. (4) 35. (4) 36. (3) 37. (1) 38. (4) 39. (3) 40. (2)
41. (1) 42. (1) 43. (3) 44. (3) 45. (2) 46. (3) 47. (1) 48. (4) 49. (1) 50. (2)
51. (1) 52. (1) 53. (4) 54. (1) 55. (1) 56. (2) 57. (3) 58. (1) 59. (4) 60. (3)
54 OSWAAL CUET (UG ) 10 Mock Test Papers GENERAL TEST

MOCK
Test Paper 9
Maximum Marks : 250 Time : 60 Minutes

General Instructions :
(i) This paper consists of 60 MCQs, attempt any 50 out of 60
(ii) Correct answer or the most appropriate answer: Five marks (+5)
(iii) Any incorrect option marked will be given minus one mark (-1)
(iv) Unanswered/Marked for Review will be given no mark (0)
(v) If more than one option is found to be correct then Five marks (+5) will be awarded to only those who have
marked any of the correct options
(vi) If all options are found to be correct then Five marks (+5) will be awarded to all those who have attempted
the question.
(vii) Calculator / any electronic gadgets are not permitted.

1. Which of the following ruler empire was spread 4. Which one of the following organizations
over almost whole of western India which included headquarter is not located in Paris?
Sindh, Gujarat, Konkan, Narmada Valley, Malwa (1) United Nations Educational Scientific and
and Kathiawar except the Nashik and Pune areas? Cultural Organisation (UNESCO)
(1) Rudradaman I (2) Kanishka (2) Organization for Economic Cooperation and
(3) Vasishka (4) Pushyamitra Sunga Development (OECD)
2. Match the following (3) International Council on Monuments and Sites
(ICOMOS)
Articles of Indian Provisions
Constitution (4) World Intellectual Property Organisation
(WIPO)
1. Article 52 A. There shall be a
president of India. 5. Which of the following is not correctly matched?
(1) Ukraine – The Bread Basket of Europe
2. Article 74 B. Council of Ministers
to aid and advise (2) Bhutan – Land of the Thunder Dragon
President. (3) North Korea – The Hermit Kingdom

3. Article 78 C. Prime Minister’s (4) Pakistan – Graveyard of Empires


responsibilities in 6. According to ___________, pressure is equal to the
terms of providing force divided by the area on which it acts.
information to the (1) Hooke’s law
President, etc.
(2) Stefan-Boltzmann law
4. Article 61 D. Procedure for (3) Newton’s law
impeachment of the
(4) None of the above
President.
7. An economic condition when there is one buyer
Choose the correct code.
and many sellers is called __________.
(1) 1-A 2-B 3-C 4-D (2) 1-B 2-A 3-C 4-D (1) Monopoly (2) Oligopoly
(3) 1-D 2-C 3-A 4-B (4) 1-C 2-A 3-B 4-D (3) Monopsony (4) Perfect Competition
3. Which of the following article is related to the 8. Lavasa, India’s inaugural privately developed
ordinance in Indian constitution? hill station, was acquired by Darwin Platform
(1) Article 123 Infrastructure for how many thousand crore
(2) Article 213 rupees?
(3) Both Article 123 and 213 (1) 1.8 thousand crore (2) 1.9 thousand crore
(4) None of the above (3) 2.0 thousand crore (4) 2.1 thousand crore
MOCK TEST Paper-9 55
9. Which of the following is managed by operating 20. Match the following
system? Martial Dance Related State
I. Memory 1. Silambam A. Maharashtra
II. Processor 2. Kalaripayattu B. Kerala
III. Input/Output devices 3. Thang-Ta C. Manipur
(1) Only I (2) I, II and II 4. Mardani Khel D. Tamil Nadu
(3) I and II (4) Only II Choose the correct code.
10. Where did the Dutch set up their trading hubs in (1) 1-A 2-B 3-C 4-D (2) 1-B 2-A 3-C 4-D
India among the options below? (3) 1-D 2-B 3-C 4-A (4) 1-C 2-A 3-B 4-D
(1) Nagappattinam, Chinsura, Machilipatnam 21. The L.C.M of two prime numbers x and y,
(2) Surat, Bharuch, Agra (x > y) is 161. The value of (3y – x):
(3) Cochin, Ahmedabad, Patna (1) –2 (2) –1 (3) 1 (4) 2
(4) All of the above 22. Successive discounts of 10% and 20% are equivalent
11. Chepu is a unique cultural symbol of ___________. to a single discount of:
(1) Bhutan (2) Myanmar (Burma) (1) 30% (2) 15% (3) 28% (4) 12%
(3) China (4) Nepal 23. Two racers run at the speeds of 100 metres/minute
12. Panama Canal connects which of the following and 120 metres/minute. If the second racer takes
water bodies? 10 minutes less than the first to complete the run,
how long is the race?
(1) Mediterranean Sea and Black Sea
(1) 1 km (2) 6 km (3) 4 km (4) 2 km
(2) Mediterranean Sea and Red Sea
(3) Atlantic Ocean and Pacific Ocean 24. Find the A.P. whose 3rd term is 18 and 12th term is
72.
(4) Mediterranean Sea and Arctic Ocean
(1) 6, 12, 18.... (2) 0,9, 18....
13. Which of the following is not correctly matched?
(3) 12, 15, 18.... (4) 4.5, 9, 18...
Place
Major Passes
(1) Jammu Kashmir Khardung pass 25. If the sum of the digits of a three digit number is
subtracted from that number then it will always be
(2) Sikkim Jelep La
divisible by
(3) Uttarakhand Muling La (1) 3 only (2) 2 only
(4) Himachal Pradesh Bomdi La
(3) Both 3 and 9 (4) All of 3,6 and 9
14. Identify the part of brain that controls the
maintenance of posture balance and equilibrium.
26. What is the sum of the first 13 terms of an arithmetic
progression if the first term is –10 and last term is
(1) Diencephalon (2) Brainstem 26?
(3) Cerebrum (4) Cerebellum
(1) 104 (2) 140 (3) 84 (4) 98
15. The structure which helps in the nutrition of the
Embryo is
27. A game consists of tossing a coin 3 times. Hanif
wins if all the tosses give the same result. What is
(1) Yolk sac (2) Amniotic membrane the probability that he loses the game?
(3) Crypts (4) Placenta
(1) 1/2 (2) 3/4 (3) 1/3 (4) 2/3
16. Who among the following is known as the Rocket
28. Kamal has some apples. He sold 40% more than he
Women of Chandrayaan 3?
ate. If he sold 70 apples, how many did he eat?
(1) Ritu Karidhal (2) Kalpana Kalahasti
(1) 18 (2) 42 (3) 50 (4) 90
(3) Nandini Harinath (4) Tessy Thomas
17. Who amongst the following invented ‘Chronometer’? 29. If 0.03  0.3  a  0.3  0.3  b , value of a is
b
(1) John Harrison (2) William Harvey
(1) 0.009 (2) 0.03 (3) 0.9 (4) 0.08
(3) Friese Greene (4) Robert Koch
18. Calcium sulphate dihydrate is commonly known 30. For an equilateral triangle, the ratio of the in-radius
and the ex-radius is
as
(1) 1 : 2 (2) 1 : 2 (3) 1 : 3 (4) 1 : 3
(1) Gypsum (2) Glass
(3) Asbestos (4) Limestone 31. The average marks obtained by a class of 60
students are 65. The average marks of half of the
19. Which of the following is NOT a noble gas?
students is found to be 85. The average marks of
(1) Neon (2) Hydrogen the remaining students is
(3) Helium (4) Argon
(1) 35 (2) 45 (3) 55 (4) 65
56 OSWAAL CUET (UG ) 10 Mock Test Papers GENERAL TEST

32. The table shows the production of different types 40. If 2 = 1.4142 , find the value of
of cars (in thousands). 1 1
2 2 2 
Years/ 2013 2014 2015 2016 2017 2  2 2 2
Cars
(1) 1.4144 (2) 2.8284 (3) 28.284 (4) 2.4142
A 35 40 48 50 36
41. Six friends, P, Q, R, S, T and U, are sitting in a closed
B 39 45 54 60 72 circle facing the centre such that each one is facing
C 52 25 32 54 45 another one. P is facing S. R is between P and Q, U
D 50 42 45 46 47 is between P and T who is sitting right of S. Who is
E 36 46 42 48 55 sitting to the immediate right of R?
(1) P (2) T (3) Q (4) S
The ratio of the total production of cars of type C

and E taken together in 2013 to the total production 42. Select the Venn diagram that best illustrates the
of cars of type D in 2014 and 2016 and type E in relationship between the following classes:
2017 taken together is: Villagers, Poor persons, Males

(1) 8 : 13 (2) 5 : 8 (3) 13 : 32 (4) 8 : 11
(1) (2)
33. In two blends of mixed tea, the ratios of Darjeeling
and Assam tea are 4 : 7 and 2 : 5. The ratio in which
these two blends should be mixed to get the ratio
of Darjeeling and Assam tea in the new mixture as
6 : 13 is (3) (4)
(1) 22 : 35 (2) 26 : 35 (3) 35 : 78 (4) 13 : 22
34. A sum of money invested at compound interest
amounts to ` 800 in 3 years and to ` 840 in 4 years. 43. Two statements are given, followed by two
The rate of interest per annum is: conclusions numbered I and II. Assuming the
statements to be true, even if they seem to be at
2
(1) 2 1 % (2) 4% (3) 5% (4) 6 % variance with commonly known facts, decide
2 3
which of the conclusions logically follow(s) from
35. If a = 64 and b = 289, then the value of statements.


Statements:

1

a + b  b a
2 is :
I. All farmers are doctors.
(1) 21/2 (2) 2 (3) 4 (4) –2 II. All doctors are magicians.
Conclusions:

36. The inner and outer radii of a circular track are,
respectively, 29 metre and 23 metre. The cost of I. All farmers are magicians.
levelling the track at `7 per sq. metre is. II. No farmer is a magician.
(1) `3,284 (2) `5,300 (3) `7,215 (4) `6,864 (1) Only conclusion II follow.
(2) Both the conclusions follow
37. A, B and C can do a piece of work in 30, 20 and 10
days respectively. A is assisted by B on one day and (3) Either conclusion I or II follows
by C on the next day, alternately. How long would (4) Only conclusion I follows.
the work take to finish? 44. A + B means ‘A is the wife of B’;
3 8 A – B means ‘B is the daughter of A’;
(1) 9 days (2) 4 days
8 8 A × B means ‘B is the brother of A’;
4 9 A ÷ B means ‘A is the father of B’;
(3) 8 days (4) 3 days
13 13 If P + R × T – Q + S ÷ U,
38. Alipta got some amount of money from her father. then how is P related to the mother of U?
In how many years will the ratio of the money and (1) Aunt
the interest obtained from it be 10:3 at the rate of (2) Paternal grandmother
6% simple interest per annum?
(3) Mother
(1) 7 years (2) 3 years (3) 5 years (4) 4 years (4) Maternal grandmother
39. Three numbers are in the ratio 5 : 7 : 12. If the sum 45. In the following question, two statements are
of the first and the third numbers is greater than given each followed by two conclusions I and II.
the second number by 50. The sum of the three You have to consider the statements to be true
numbers is even if they seem to be at variance from commonly
(1) 125 (2) 120 (3) 95 (4) 85 known facts. You have to decide which of the
MOCK TEST Paper-9 57
given conclusions, if any, follows from the given 51. A, B, C, D, E, F, G and H are sitting around a circular
statements. table facing towards centre. H is on the immediate
Statements:
left of A but is not the neighbor of D or E. F is on
(I)  Happiness is around everyone, we have to the immediate right of B. C is between E and F, G is
find it. the neighbor of E. Who is between A and B?
(II) Happiness is a state of mind. If we can’t find it, (1) D (2) G (3) F (4) H
we should create it.
52. What is the mirror image of SMILE?
Conclusions:

(1) SM LEI (2) SM I E L
(I) Creation of happiness itself makes an individual M
satisfied and happy because creativity is (3) S L EI (4) SM I L E
constructive.
53. Select the option that is related to the third term in
(II) It’s all about analyzing the situational factors to the same way as the second term is related to the
understand happiness.
first term and sixth term is related to fifth term.
(1) Only Conclusion II follows
72 : 14: : 87 : ? :: 96 : 54

(2) Conclusion I and II both follow
(1) 52 (2) 56 (3) 15 (4) 29
(3) Neither I nor II follow
(4) Only Conclusion I follows 54. Which answer figure will complete the pattern in
the question figure?
46. Ravi is walking in the East direction. After covering
a distance of one kilometre, he turns 45° left and Question Figure:

then 90° right. In which direction is he now?
(1) North-East (2) West
(3) North-West (4) North
47. Select the correct option that will fill in the blank
and complete the series.
AEI, EIO, IOU, ........

(1) OAU (2) OUA (3) IOA (4) MIL
48. Find out the two signs to be interchanged to make Answer Figures:

the following equation correct.
25 + 5 × 7 − 12 ÷ 3 = 26

(1) + and ÷ (2) + and × (2)
(1)
(3) − and ÷ (4) + and −
49. CDF : GHJ :: KLN : ?
(1) YED (2) EDU (3) OPS (4) OPR
50. Choose the correct mirror image:
(3)
(4)

55. Question figure

1. 2.
Answer figure

(2)
(1)

3. 4.

(3) (4)
(1) 1 (2) 2 (3) 3 (4) 4
58 OSWAAL CUET (UG ) 10 Mock Test Papers GENERAL TEST

56. Saksham introduced Nidhi to his friend, “She is the 59. A + B means ‘A is the father of B’.
daughter of the only son of my father’s wife.” How A – B means ‘A is the sister of B’.

is Saksham related to Nidhi?
A × B means ‘A is the brother of B’.

(1) Son (2) Cousin (3) Father (4) Brother
A ÷ B means ‘A is the mother of B’.

57. Directions: If you start running from a point
If, U + H × L – Q ÷ R – Y,

towards North and after covering 4 km, you turn
to yours left and run 5 km and then again turn to Then how is L related to Y?

your left and run 5 km and then turn to left again (1) Mother’s brother
and run another 6 km and before finishing you (2) Mother’s sister
take another left turn and run 1 km, then answer (3) Maternal grandmother
the following question based on this information. (4) Sister
Question: From the finishing point if you have to
60. Select the related word pair from the given
reach the point from where you started, in which alternatives.
direction will you have to run?
Afghanistan : Kabul :: ....... : .........

(1) West (2) East (3) South (4) North
(1) Iran : Dublin
58. Select the number that will come next in the
(2) Nepal : Monaco
following series.
(3) Pakistan : Islam Nagar
1, 5, 9, 17, 25, 37, 49, 65, 81, .........

(1) 99 (2) 110 (3) 100 (4) 101 (4) China : Beijing



Answer Key

1. (1) 2. (1) 3. (3) 4. (4) 5. (4) 6. (4) 7. (4) 8. (1) 9. (2) 10. (1)
11. (4) 12. (3) 13. (4) 14. (4) 15. (1) 16. (4) 17. (1) 18. (1) 19. (2) 20. (3)
21. (1) 22. (3) 23. (2) 24. (3) 25. (3) 26. (1) 27. (2) 28. (3) 29. (3) 30. (1)
31. (2) 32. (1) 33. (1) 34. (3) 35. (1) 36. (4) 37. (1) 38. (3) 39. (2) 40. (2)
41. (3) 42. (4) 43. (4) 44. (1) 45. (3) 46. (1) 47. (2) 48. (1) 49. (4) 50. (3)
51. (1) 52. (4) 53. (2) 54. (4) 55. (1) 56. (3) 57. (1) 58. (4) 59. (2) 60. (4)
MOCK TEST Paper-1 59

MOCK
Test Paper 10
Maximum Marks : 250 Time : 60 Minutes

General Instructions :
(i) This paper consists of 60 MCQs, attempt any 50 out of 60
(ii) Correct answer or the most appropriate answer: Five marks (+5)
(iii) Any incorrect option marked will be given minus one mark (-1)
(iv) Unanswered/Marked for Review will be given no mark (0)
(v) If more than one option is found to be correct then Five marks (+5) will be awarded to only those who have
marked any of the correct options
(vi) If all options are found to be correct then Five marks (+5) will be awarded to all those who have attempted
the question.
(vii) Calculator / any electronic gadgets are not permitted.

1. According to which inscription Samudragupta 6. Router is used in which of the following layer?
defeated 9 kings in the North, and 12 Kings in (1) Transport layer (2) Network layer
the South, and reduced all the Atavika states to (3) Physical layer (4) Application layer
vassalage?
7. Match the following:
(1) Iron Pillar (2) Sun Pillar
1. The Battle of Ghaghra A. 1529
(3) Vijaya Stambha (4) Allahabad Pillar
2. Battle of Khatauli B. 1518
2. Which of the following is correctly matched?
3. Battle of Gagron C. 1519
1. National Good Governance Day: 25 December
4. The third battle of Panipat D. 1761
2. Armed Forces Flag Day: 7 December
Choose the correct code.
3. Vijay Diwas: 16 December
4. Hindi Day: 14 September (1) 1-A 2-B 3-C 4-D (2) 1-B 2-A 3-C 4-D
Choose the correct code.
(3) 1-D 2-C 3-A 4-B (4) 1-C 2-A 3-B 4-D
(1) 1 and 2 only (2) 2 and 4 only 8. Which of the Mughal emperors is commemorated
(3) 1, 2 and 4 only (4) 1, 2, 3 and 4 with a mosque bearing his name in Fatehabad?
1. Akbar 2. Babur
3. Which of the following statement is/are correct
3. Humayun 4. Jahangir
1. Sucheta Kriplani was India’s first woman chief
minister. 9. Which of the following statement is correct?
2. Sarojini Naidu was the first woman Governor 1. The Rourkela Steel Plant was established in
of the state of Uttar Pradesh. 1959 within the Sundargarh district of Odisha
Choose the correct code.
through a partnership with Germany.
(1) 1 only (2) 2 only 2. The Bhilai Steel Plant, situated in the Durg
(3) Both 1 and 2 (4) Neither 1 nor 2 District of Chhattisgarh, initiated production in
1959 with collaborative efforts from Russia.
4. The first woman to preside over the UN General
Assembly: 3. The Durgapur Steel Plant in West Bengal
commenced its operations in 1962 with
(1) Rajkumari Amrit Kaur
the assistance of the United Kingdom’s
(2) Vijaylakshmi Pandit
government.
(3) Kamla Nehru
4. The Bokaro Steel Plant, founded in 1964 in
(4) Indira Gandhi
Bokaro, was established through collaboration
5. Which among the following is a light sensitive with Russia.
device used for converting images to their digital Choose the correct code.

form?
(1) 1 and 2 only (2) 2,3 and 4 only
(1) Printer (2) Monitor
(3) 1, 2 and 4 only (4) 1, 2, 3 and 4
(3) Scanner (4) RAM
60 OSWAAL CUET (UG ) 10 Mock Test Papers GENERAL TEST

10. Wind turbines convert _____________ energy into 20. ‘Mahuri’ is a traditional musical instrument
mechanical power. belonging to the state of:
(1) Chemical (2) Nuclear (1) Kerala (2) Odisha
(3) Gravitational (4) Kinetic (3) Himachal Pradesh (4) Arunachal Pradesh
11. Which of the following glands is present between 21. Allowing 20% and 15% successive discounts, the
the lungs? selling price of an article becomes ` 3,060; then the
(1) Pituitary (2) Hypothalamus marked price will be
(3) Thymus (4) Pineal (1) ` 4,000 (2) ` 4,400 (3) ` 5,000 (4) ` 4,500
12. Taiga is an example of which type of ecosystem? 22. The compound interest on ` 30,000 at 7% per
(1) Grassland ecosystem annum for a certain time is ` 4,347. The time is
(2) Forest ecosystem (1) 3 years (2) 4 years
(3) Tundra ecosystem (3) 2 years (4) 2.5 years
(4) Desert ecosystem 23. What is the value of (1 + 1/n) + (1 – 2/n) + (1 – 3/n)
13. Which of the following states share a boundary +……... up to n terms?
with Bhutan? (1) 1/n (2) n/2
1. West Bengal 2. Arunachal Pradesh (3) n(n – 1)/2 (4) (n – 1)/2
3. Sikkim 4. Assam 24. A man can row 6 km/h in still water. If the speed
Choose the correct code.
of the current is 2 km/h, it takes 3 hours more in
(1) 2, 3 and 4 only (2) 1 and 4 only upstream than in the downstream for the same
(3) 1, 2 and 4 only (4) 1, 2, 3 and 4 distance. The distance is
14. A worldwide financial messaging network which (1) 30 km (2) 24 km (3) 20 km (4) 32 km
exchange messages between banks and financial 25. If x2 – y2 = 80 and x – y = 8, then the average of x
institutions is known as_________ and y is
(1) CHAPS (2) SWIFT (3) NEFT (4) SFMS (1) 2 (2) 3 (3) 4 (4) 5
15. Who among the following is the first Indian Prime 26. The table shows the production of different types
Minister to receive the Grand Cross of the Order of of cars (in thousand). Study the table carefully and
Honor from Greece? answer the question:
(1) Indira Gandhi (2) Atal Bihari Vajpayee Years/
2014 2015 2016 2017 2018
(3) Narendra Modi (4) Dr. Manmohan Singh Cars
16. Match the following. A 64 56 57 63 70
B 48 54 63 64 72
List I (Unit) List II (Physical Quantity)
C 33 42 48 57 64
1. Pascal A. Pressure
D 25 45 40 55 35
2. Joule B. Work
E 40 48 52 61 60
3. Fahrenheit C. Temperature The ratio of the total production of type A cars in

4. Lightyear D. Distance 2015 and type B cars in 2014 taken together to the
Choose the correct code.
total production of type C cars in 2017 and type E
cars in 2018 taken together is:
(1) 1-A 2-B 3-C 4-D (2) 1-B 2-A 3-C 4-D
(1) 16 : 19 (2) 4 : 5 (3) 8 : 9 (4) 34 : 39
(3) 1-D 2-C 3-A 4-B (4) 1-C 2-A 3-B 4-D
17. Among the listed bacteria, which one exhibits 27. If A earns 25% more than B then how much percent
does B earns less than A:
penicillin resistance due to its absence of a cell
wall? (1) 15% (2) 20% (3) 25% (4) 30%
(1) Spirochetes (2) Mycoplasmas 28. In ∆ABC, the line parallel to BC intersects AB and
(3) Cyanobacteria (4) Bdellovibrios AC at P and Q respectively. If AB : AP = 5 : 3, then
18. Corundum is an ore of: AQ : QC is:
(1) Copper (2) Silver (1) 3 : 2 (2) 2 : 3 (3) 3 : 5 (4) 1 : 2
(3) Iron (4) Aluminum
29. The altitude of an equilateral triangle of side
19. Where is the headquarters of the Inter­ national
2
Olympic Committee located? cm is:
3
(1) Italy (2) Switzerland
4 4 4
(3) Belgium (4) France (1) m (2) cm (3) M (4) 1 cm
3 3 3
MOCK TEST Paper-10 61
30. The fifth term of the sequence for which 40. The ratio of boys and girls in a school is 27 : 23.
t1 = 1, t2 = 2 and tn+2 =tn + tn+1, is If the difference between the number of boys and
girls is 200, then find the number of boys.
(1) 5 (2) 10 (3) 6 (4) 8
(1) 1350 (2) 1250 (3) 1300 (4) 1200
31. The average weight of 15 oarsmen in a boat is
increased by 1.6 kg when one of the crew who 41. Find out the two signs to be interchanged to make
weighs 42 kg is replaced by a new man. Find the the following equation correct.
weight of the new man (in kg). 15 + 15 × 15 − 15 ÷ 15 = 15

(1) 67 (2) 65 (3) 66 (4) 43 (1) − and ÷ (2) + and −
32. A student was asked to divide a number by 6 and (3) × and ÷ (4) + and ×
add 12 to the quotient. He, however first added 12 42. Select the Venn diagram that best illustrates the
to the number and then divided it by 6. Getting relationship between the following classes:
112 as the answer. The correct answer should have
Indians, Females, Voters

been
(1) 124 (2) 122 (3) 118 (4) 114
33. By selling cloth at ` 9 per metre, a shopkeeper loses (1) (2)
10%. Find the rate at which it should be sold so as
to earn profit of 15%.
(1) ` 11.20 (2) ` 11.30 (3) ` 11.40 (4) ` 11.50
(3) (4)
34. The square root of 3 + 2 is
3− 2
43. Read the given statements and conclusions
(1) 3+ 2 (2) 3− 2
carefully. Assuming that the information given
(3) 2± 3 (4) 2− 3 in the statements is true, even if it appears to be at
35. A student takes 1.25 hours to travel from home to variance with commonly known facts, decide which
school at a speed of 4 km/h. By what percentage of the given conclusions logically follows from the
should he increase his speed to reduce the time statements.
by 25% to cover the same distance from school to Statements:
home? I. Some bottles are glasses.
(1) 33 1 % (2) 45% (3) 25% (4) 50% II. Some glasses are bowls.
3 Conclusions:
36. The radius of a wheel is 14 cm. What is the distance I. Some bowls are bottles.
(in cm) travelled by the wheel in 15 revolutions? II. Some glasses are bottles.
(1) 440 (2) 660 (3) 1320 (4) 1980 III. No bowl is a bottle.
37. PQRS is square whose side is 20 cm. By joining (1) All Conclusions I, II and III follow.
opposite vertices of PQRS are get four triangles. (2) Either Conclusion II or III follows.
What is the sum of the perimeters of the four (3) Only Conclusion II and either Conclusion I or
triangles? III follow.
(1) 40 2 cm (2) 80 2  80  (4) Only Conclusions II and III follow.
44. Question figure
(3)  40 2  40  (4)  40 2  80 

38. Ram and Shyam can complete a task in 6 2 days


3
and 15 days, respectively. They work together for
4 days, and then Ram leaves. In how many
days after Ram leaves, will Shyam complete the Answer figure

remaining task alone?
1
(1) 1 days (2) 2 days (3) 3 days (4) 4 days
2 (1) (2)
39. If a sum of money becomes ` 4000 in 2 years and
` 5500 in 4 years 6 months at the same rate of
simple interest per annum, then the rate of simple
interest is
(3) (4)
(1) 21 3 % (2) 21 2 % (3) 21 1 % (4) 21 5 %
7 7 7 7
62 OSWAAL CUET (UG ) 10 Mock Test Papers GENERAL TEST

45. A clock is started at noon. By 20 minutes part 6, the 50. WOLF : FLOW :: DRAW : ?
hand had has turned through: (1) WRAD (2) WARD (3) RWAD (4) DARW
(1) 175° (2) 180° (3) 185° (4) 190° 51. In a code language, ‘PLACARD’ is written as
46. Which answer figure will complete the pattern in ‘TPEYEVH’. How will ‘MONSTER’ be written in
the question figure? that language?
Question Figure:
(1) RTSOXIV (2) QSROXIV
(3) QSRRXIV (4) PSSOXJV
52. Five friends P, Q, R, S and T are sitting around a
circular table facing the center of the table. S is
sitting to the immediate right of P. T is sitting to the
immediate left of Q. P is between S and R. Who is
sitting at the second place to the left of T?
(1) Q (2) R (3) P (4) S
Answer Figures:
53. Select the option that is related to the third number
in the same way as the second number is related to
the first number.
(1) (2) 6 : 252 :: 5 : ?

(1) 225 (2) 150 (3) 175 (4) 125
54. Four villages A, B, C and D lie in a straight line. D is
(3) 10 km from B. A is exactly between D and C and C
(4)
from B is 2 km more than it is from D. How far is C
from B?
47. Introducing Kumar, Vikram said, “His mother is (1) 4 km (2) 6 km (3) 8 km (4) 2 km
the wife of my father’s son. Brother and sisters I 55. Which number will follow next in the given
have none.” How is Kumar related to Vikram? number series?
(1) Uncle (2) Son 3, 18, 48, 93, ?

(3) Brother (4) Father (1) 247 (2) 147 (3) 153 (4) 202
48. If today is Monday, After 62 it will be: 56. X, Y, Z, P, Q and R are sitting in a row facing north.
(1) Wednesday (2) Saturday X is the neighbour of Y and P. Q is the neighbour
of Z and R, P is to the right of X and P and Z are
(3) Sunday (4) Thursday
immediate neighbours. Which pair sits on the
49. In the following question, two statements are extreme ends?
given each followed by two conclusions I and II.
(1) Y, P (2) Y, R (3) X, P (4) P, Z
You have to consider the statements to be true
even if they seem to be at variance from commonly
57. Select the correct option that will fill in the blank
known facts. You have to decide which of the and complete the series.
given conclusions, if any, follows from the given J, N, R, V, .........

statements. (1) D (2) Z (3) V (4) G
Statements: 58. A + B means ‘A is the mother of B’;
(I) A
 ir purifier is the latest innovation of this A – B means ‘A is the brother of B’;

decade. A × B means ‘A is the father of B’;

(II) 
Marketers identified the need and grabbed A ÷ B means ‘A is the daughter of B’.

the opportunity of polluted atmosphere and
If, P – K × Y – J ÷ S + R,

created a new product for people.
then which of the following statements is NOT

Conclusions:
correct ?
(I) The reason for invention of air purifier is rising
(1) J is daughter of P.
levels of pollution.
(2) K is husband of S.
(II) The need is created in market about the product
and then it is established. (3) Y is son of S.
(1) Only Conclusion II follows. (4) P is paternal uncle of R.
(2) Conclusion I and II both follow. 59. Select the related word pair from the given
alternatives.
(3) Neither I nor II follow.
Month : Year :: .......... : ..........
(4) Only Conclusion I follows.
MOCK TEST Paper-10 63
(1) Litre : Liquid (2) Speed : Vehicle and also in line with B. In which direction of C is D
(3) Time : Distance (4) Minute : Hour located?
60. A is located to the West of B. C is located at North (1) South (2) South-East
in between A and B. D is exactly to the South of B (3) West (4) South-West


Answer Key

1. (4) 2. (4) 3. (3) 4. (2) 5. (3) 6. (2) 7. (1) 8. (3) 9. (4) 10. (4)
11. (3) 12. (2) 13. (4) 14. (2) 15. (4) 16. (1) 17. (2) 18. (4) 19. (2) 20. (2)
21. (4) 22. (3) 23. (4) 24. (2) 25. (4) 26. (3) 27. (2) 28. (1) 29. (4) 30. (4)
31. (3) 32. (2) 33. (4) 34. (1) 35. (1) 36. (3) 37. (2) 38. (2) 39. (1) 40. (1)
41. (1) 42. (1) 43. (3) 44. (4) 45. (3) 46. (4) 47. (2) 48. (1) 49. (4) 50. (2)
51. (2) 52. (3) 53. (2) 54. (2) 55. (3) 56. (2) 57. (2) 58. (1) 59. (4) 60. (2)
SOLUTIONS OF
Mock Test Paper 1
1. Option (3) is correct. and it was eventually abandoned after the
Explanation: The practice and institution of reunification of Vietnam in 1976.
Langar were originally initiated by Baba Farid, 38th Parallel: The 38th Parallel is the border
a Muslim belonging to the Chishti Sufi order. between North Korea and South Korea. It was
This tradition was already prevalent among drawn up by the United States and the Soviet
Sufis in the 12th and 13th centuries within the Union at the end of World War II. The 38th
Indian subcontinent. In Sikhism, Langar holds Parallel is also known as the Demilitarized
significant importance and was established Zone (DMZ), and it is one of the most heavily
when Guru Nanak Ji, the founder of Sikhism, fortified borders in the world.
provided food to hungry holy men. 22nd Parallel: The 22nd Parallel is the border
Guru Amardas Sahib Ji Introduced Anand between Egypt and Sudan. It was drawn up in
Karaj (Marriage Ceremony). 1899 by the British and the French, who were
Mata Khivi, the wife of the second Sikh the colonial powers in the region at the time.
Guru Angad Dev, played a crucial role in The 22nd Parallel is a natural boundary, as it
the development of Langar. She actively follows the course of the Nile River.
participated in serving at Gur ka Langar, the Durand Line: The Durand Line is the border
Guru's free kitchen, alongside the first five between Pakistan and Afghanistan. It was
Gurus. drawn up in 1893 by Sir Mortimer Durand, the
The third Guru, Amar Das, further refined British Foreign Secretary. The Durand Line is a
the concept of Langar by introducing "pangat controversial border, as it is not recognized by
sangat," where individuals of all ranks sit and Afghanistan.
eat together as equals in the congregation.
In present times, Langar remains an integral
5. Option (2) is correct.
part of every gurdwara and Sikh worship Explanation: Articles 245 to 255 in Part XI of the
service. Its provision, preparation, service, and Constitution deal with the legislative relations
cleanup are entirely voluntary, symbolizing between the Centre and the states.
the spirit of selfless service and equality within Constitution of India, being federal in
the Sikh community. structure, divides all powers (legislative,
2. Option (1) is correct. executive and financial) between the Centre
and the states.
Explanation: Fertile riverine alluvial soil is
best suited for producing sugarcane, rice and 6. Option (4) is correct.
plantain. Alluvial soil is rich in potassium. Explanation: On 21st January, 1972, all the
Red soil is ideal for crops like corn, red gram, three states (Manipur, Meghalaya and Tripura)
Bengal gram, green gram, groundnut and became full-fledged states under the North-
castor seed. Eastern Areas (Reorganisation) Act 1971.
3. Option (4) is correct. 7. Option (1) is correct.
Explanation: Andaman and Nicobar is a group Explanation: Iris is a potency in our eye that
of 572 islands out of which 37 are inhabited regulates the measure of the pupil. It widens
at the junction of the bay of Bengal and the and lessen because of which extent of the pupil
Andaman sea. changes. Iris controls the size of the pupil.
4. Option (3) is correct. 8. Option (4) is correct.
Explanation: 17th Parallel: The 17th Parallel Explanation: Treaty of Madras (1769): This
was the border between North Vietnam and treaty was signed between the British East
South Vietnam during the Vietnam War. It India Company and Hyder Ali of Mysore to
was drawn up by the Geneva Accords of 1954, end the First Mysore War. The treaty forced
which ended the First Indochina War. The Hyder Ali to cede some territory to the British
17th Parallel was never a permanent border, and to pay a large indemnity.
MOCK TEST Paper-1 65

Treaty of Wadgaon (1779): This treaty The World Trade Organization (WTO): The
was signed between the British East India World Trade Organization is headquartered
Company and the Maratha Empire to end in Geneva, Switzerland. It was established in
the Second Phase of the First Anglo-Maratha 1995 to promote free trade between member
War. The treaty forced the Marathas to cede countries.
some territory to the British and to pay a large
indemnity.
11. Option (4) is correct.
Treaty of Salbai (1782): This treaty was signed Explanation: Sir Syed Ahmad Khan found the
between the British East India Company and Muslim society to be educationally, socially
the Maratha Empire to end the First Anglo- and culturally backward. The movement came
Maratha War. The treaty restored most of the to be known as the Aligarh movement after sir
territory that the Marathas had ceded to the Syed established his school at Aligarh which
British in the Treaty of Wadgaon, but it also later became the center of the movement.
forced the Marathas to recognize the British as
the dominant power in India. 12. Option (2) is correct.
Treaty of Seringapatam (1792): This treaty Explanation: Linux is not a computer
was signed between the British East India language, it is a Unix-like computer operating
Company and Tipu Sultan of Mysore to end system assembled under the model of free
the Third Anglo-Mysore War. The treaty forced and open-source software development and
Tipu Sultan to cede much of his territory to distribution. Among all general- purpose-
the British, including the city of Seringapatam, operating systems, Linux has the largest
and to pay a large indemnity. installed base. Also, it is the leading operating
9. Option (3) is correct. system on servers and other big iron systems
(such as mainframe computers).
Explanation: First Five-Year Plan (1951-1956):
It is based on Harrod-Domar Model. It 13. Option (3) is correct.
mainly focused on Agriculture. Community Explanation: Maria Ressa was not honoured
Development Program is also introduced in with the Nobel Peace Prize in 2022. She was
this Plan. awarded the prize in 2021, along with Dmitry
Second Five-Year Plan (1956-1961): It is based Muratov, for their "courageous fight for
on Mahalanobis Model. It mainly focused on freedom of expression" in the Philippines and
Industry. Steel plants at Bhilai, Durgapur, and Russia, respectively.
Rourkela were established with the help of
The other three individuals or organizations
Russia, Britain (the U.K.), and West Germany
were honoured with the Nobel Peace Prize
respectively.
in 2022: Ales Bialiatski is a Belarusian human
Fourth Five-Year Plan (1969-1974): It is based
rights defender and the founder of the
on Ashok Rudra Alon Model. The smiling
Belarusian Helsinki Human Rights Center.
Buddha project was also introduced in
Russian human rights organisation Memorial
this Plan. 14 Commercial Banks were also
is a Russian human rights organization that
Nationalised in this Plan.
documents human rights abuses in Russia.
Fifth Five-Year Plan (1974-1979): It is based on
D.P. Dhar Model. The slogan "Garibi Hatao" Ukrainian human rights organisation Center
was also introduced in this plan. Regional for Civil Liberties is a Ukrainian human rights
Rural Bank was established In this Plan organization that promotes democracy and
(2 October 1975). human rights in Ukraine. The Nobel Peace
Prize 2022 was awarded to these individuals
10. Option (4) is correct. and organizations for their "efforts to
Explanation: UNICEF (United Nations safeguard freedom of speech in Belarus,
Children's Fund): UNICEF is headquartered Russia and Ukraine."
in New York City, United States. It was 14. Option (2) is correct.
established in 1946 by the United Nations
General Assembly to provide humanitarian Explanation: Surendranath Banerjee founded
and developmental assistance to children and the Indian National Association with Ananda
mothers in developing countries. Mohan Bose on 26 July 1876. Surendranath
The World Bank: The World Bank is Banerjee later joined the Indian National
headquartered in Washington, D.C., United Congress, he supported the Montagu-
States. It was established in 1945 to provide Chelmsford reforms in contrast to congress.
loans to developing countries for economic Later, he left congress and formed Indian
development projects. National Liberation Federation in 1919.
66 OSWAAL CUET (UG ) 10 Mock Test Papers GENERAL TEST

15. Option (3) is correct. 20. Option (1) is correct.


Explanation: The Indian Space Research Explanation: The Biraja Temple, the Rajarani
Organisation (ISRO) has planned to keep Temple and the Samaleswari Temple are all
the names of the Chandrayaan-2 lander and located in Odisha. Biraja Temple or Biraja
rover for their corresponding components Kshetra is one of the most famous Hindu
in the Chandrayaan-3 mission as well. The
temples located in the Jajpur District of Odisha,
Chandrayaan-3 lander will be named 'Vikram,'
India. Rajarani temple was built during the
paying tribute to Vikram Sarabhai, the pioneer
of the Indian space program. The rover will later stages of the Somavamshi rule in the
be called 'Pragyan.' According to available 11th century A.D. The Samaleswari temple is
information, the propulsion module will located in Sambalpur, Odisha.
transport the lander-rover configuration to a
21. Option (2) is correct.
lunar orbit at a distance of 100 kilometres from
the moon's surface. Upon a successful landing Explanation:
of the 'Vikram' module on the lunar surface, 789x531y is divisible by 72
it will release the 'Pragyan' rover, which will
So, 789x531y is divisible by 8 and 9.
then conduct an in-situ chemical analysis of
the lunar terrain during its mobility. Given number is divisible by 8.
31y is divisible by 8.
16. Option (2) is correct. ∴y=2
Explanation: Pedology is the study of soils Given number is divisible by 9
in their natural environment. There are two 7 + 8 + 9 + x + 5 + 3 + 1 + 2 = 36
main branches of soil science Pedology and ⇒ x + 35 = 36 ⇒ x =36 – 35 = 1
edaphology. Edaphology studies the way soils
∴ 5x – 3y = 5 × 1 – 3 × 2 = 5 – 6 = –1
influence plants, fungi and other living things
while Pedology deals with soil morphology, 22. Option (3) is correct.
pedogenesis and soil classification.
Explanation:
17. Option (1) is correct. Here, BC = Height of the unfinished tower
Explanation: Terylene is not a raw fiber. It AB = Height of the finished tower
is an artificial polyester fiber assembled by In DOAB,
C
petroleum-derived polymerizing ethylene AB
glycol and terephthalic acid. Terylene is the tan 30° =
OA x
trademark of the first polyester fabric ever
created for synthetic material. It was first made 1 h
= B
by Chemist JR Whinfield in 1941. 3 78
18. Option (4) is correct. 78 60°
h= m ...(i) h
3
Explanation: An insulator is a significance In DOAC, 30°
that accomplishes not permitting electricity to O 78 m A
hand via it. Glass is a fine insulator as it does AC
tan 60° =
not enable an outpour of electricity. OA
h+x
19. Option (3) is correct. 3=
78
Explanation: Indian Surf Team won the bronze 78 3 = h + x
medal at the Asian Surfing Championship, held
at Thulisdhoo Island in Maldives. 18 countries 78
78 3 = +x [From equation (i)]
from Asia took part in this championship. 3
The Surfing Federation of India sent a team 78
comprising 4 senior Surfers in the men’s x = 78 3 −
3
category and 2 Surfers in the Under 18 category.
5 out of the 6 Surfers were from Tamilnadu. 78 3
= 78 3 − ×
Srikanth from Kovalam, Tamil Nadu made it to 3 3
the semi-finals in the Men's Category.
78 3
The President of the Surfing Federation of = 78 3 −
India Arun Vasu said that the Surfing team 3
finished overall 3rd with a bronze medal.  1 2
= 78 3  1 −  = 78 3 ×
Indian Surfers will now get ready for the first-  3 3
ever World Surfing League (WSL) event in
x = 52 3 m
India to be held in Mahabalipuram in Tamil
Nadu from August 14 to 20 this year. Hence, the tower will be raised 52 3 m.
MOCK TEST Paper-1 67
23. Option (2) is correct. 29. Option (3) is correct.
Explanation: Explanation:
Speed of boat in still water
Average = A1 x1 + A2 + x 2 + ... + An xn
1
x1 + x 2 + ... + xn = (Downstream rate + Upstream rate)
2
Where, A1, A2, A3 … An are frequencies.
1 12
= (8 + 4) kmph = kmph = 6 kmph
∴ Required mean 2 2
1 ×1 + 2 ×2 + 3 ×3 + 4 ×4 + 5 ×5 + 6 ×6 + 7 ×7 30. Option (2) is correct.
=
1+ 2 + 3+ 4 + 5+ 6 + 7
Explanation: Let time taken by B and C be x
1  4  9  16  25  36  49 140 days
=  5
28 28 So, time taken by A = 3x days
Part of work done by A, B and C in 1 day
24. Option (1) is correct. = 
1 1 31 4
 
x 3x 3x 3x
Explanation: S.P. of machine = 80% of ` 3840
As per the question,
 3840  80  4 1
=`   = ` 3072 ∴ = or 3x = 4 × 24
 100  3x 24
4 × 24
or x = = 32 days
25. Option (3) is correct. 3
Thus, time taken by A = 3x = 3 × 32 = 96 days
Explanation: Maximum length of each price
31. Option (3) is correct
= HCF of 1.5 m and 1.2 m = 0.3 m
Explanation: p, q, r are in G.P.
12 15 1 q r
So, =
12 p q
3 12  4 or q2 = pr
12
x or q = pr

∴ HCF of 1.5 and 1.2 metre = 0.3 metre. 32. Option (1) is correct.
Hence, the greatest length of each pipe is 0.3m Explanation:
A

26. Option (3) is correct


Q R

Explanation:
1
 100 B P C
S.I .  100 100
Rate =   12% p.a. According to the question,
Principal  Time 1
1 AQ = AR
12
Q AB = AC (Given)
27. Option (4) is correct. So, BQ = RC
Now, BQ = BP,
Explanation: As per question, CP = CR
The number 84 must not be a multiple of sum ∴ BP = PC
of the terms of ratio. 33. Option (4) is correct.
For ratio 3 : 2. Explanation:
Sum of the terms of ratio = 3 + 2 = 5 which Given that,
is not a factor of 84 ∆ABC is an isosceles triangle.
AB = AC,
28. Option (1) is correct. XY || BC
Explanation: As we know that, and ∠A = 30°
A
(m – n)2 = m2 + n2 – 2 mn
30˚
162 = 400 – 2 mn
2 mn = 400 – 256 X Y

2 mn = 144
mn = 72 B C
68 OSWAAL CUET (UG ) 10 Mock Test Papers GENERAL TEST

180  30
38. Option (4) is correct.
Now, ∠ABC = ∠ACB =
2 Explanation:
150 Given that,
  75
2 Diagonals of a rhombus = 16 cm and 30 cm
Q XY || BC D
∴ ∠AXY = ∠ABC = 75°
∴ ∠BXY = 180° – 75° = 105°
34. Option (4) is correct.
Explanation: Time from 10 p.m., to 6 a.m. = O C
A
8 hours
∴ Required time (15 + 35)% of 8 hours
8  50 
=   hours
 100 
B
= 4 hours

35. Option (1) is correct. Now, AC = 16 cm


∴ OA = 8 cm
Explanation: By using the alligation rule, we BD = 30 cm
get
Milk in Vessel A Milk in Vessel B ∴ OB = 15 cm
∠ AOB = 90°
4 2
litre litre ∴ In ∆OAB
7 5
1
AB = OA 2 + OB 2
2
1 2 4 1  ( 8 )2  (15)2
− −
2 5 7 2
= 64 + 225
5 −4 1 8 −7 1
= = = =
10 10 14 14 = 289
1 1
So, the required ratio = : = 17 cm
10 14
∴ Perimeter of rhombus = 4 × l
= 14 : 10 = 7 : 5 = 4 × 17
36. Option (4) is correct. = 68 cm

Explanation: Given: x = 3 − 2 2 39. Option (1) is correct.


1 1 1 32 2 Explanation: Distance covered by the train
   
x 32 2 32 2 32 2 that started at 11 AM in half an hour
32 2 1
  32 2  110   55 km
98 2
2
Relative speed
Now,  x  1   x  1  2
 
 x x = (140 – 110) km/hr
= 3 −2 2 + 3 + 2 2 + 2 = 8 = 30 km/hr
1
∴ x+ = 8 = 2 ×2 ×2 = 2 2 Time Taken in meeting from 11:30 AM
x
55 11
37. Option (3) is correct. = = hours.
30 6
Explanation: Distance covered by the train that started at
We know that, 11:30 a.m.
radius of the incircle (x) of an equilateral triangle  11 
=  140 ×  km
side  6
=
2 3 770
= km
2 3 3
=
2 3 2
⇒ 256 km
= 1 cm. 3
MOCK TEST Paper-1 69
40. Option (4) is correct. 44. Option (2) is correct.
Explanation: Rate = 10% per annum = 5% per Explanation:
half year
Time = T years 12 S
2T
 R 
 
A  P1  2  9 3 E W
 100 
2T
 5 
926.10  800  1   6 N
 100 
2T As per the question, at 12 a.m., the minute
926.10  1 
  1 hand towards South. So, at 3 a.m. also the
800  20 
minute hand would at 12 and so it would be
2T
9261  21  point at South.
 
8000  20 
3
 21   21 
2T
45. Option (4) is correct.
    
 20   20 
Explanation:
On comparing the powers on both sides, we
get As,
17 24 45
3 ⇒ T 3=
2T = = 1
years 1 years
2 2
+7 +21
41. Option (2) is correct. Same as,
Explanation: As Dentist comes under the 12 19 40
category of Doctor, in the same way Chemistry
comes under the category of Science. +7 +21

42. Option (3) is correct. 46. Option (1) is correct.


Explanation: According to the questions, Explanation:

I
like chocolates 9 5 8 (i) 2 12 15 3 11
B L O C K
As,
We bought chocolates 1 5 3(ii) +1 +1 +1 +1 +1
3 13 16 4 12
We like them 8 1 6 (iii) Similarly,
From equation (i), (ii) and (iii) 19 21 16 18 5 13 5
I = 9, bought = 3 and them = 6 S U P R E M E
∴ I bought them = 936 +1 +1 +1 +1 +1 +1 +1

43. Option (1) is correct. 20 22 17 19 6 14 6


Explanation: Given, total students = 42
Swati’s rank from bottom = 19 47. Option (4) is correct.
So, Purshottam’s rank from bottom Explanation: Correct relation between the
= 19 – 6 = 13 given classes
We know that,
Total students = Position from top + Position Relatives
from bottom – 1
⇒ 42 = Position from top + 13 – 1 Uncle Rich
\ Position from top = 42 – 12 = 30
Hence, Purshottam’s rank from the top = 30th
70 OSWAAL CUET (UG ) 10 Mock Test Papers GENERAL TEST

48. Option (4) is correct. 54. Option (3) is correct.


Explanation: According to the statements, Explanation:

Diagonals Decagons
Angles Cubes
X
Polygons Cones

I. () II. () III. () IV ()


Therefore, Sujit is the brother-in-law of
Hence, conclusions I and III follow.
Manideep.
49. Option (2) is correct.
55. Option (4) is correct.
Explanation:
Explanation:
3 16 29 42 55
Given : 8 ÷ 2 + 4 = 10
+13 +13 +13 +13
8 + 4 ÷ 2 = 10
50. Option (4) is correct.  + changed
÷
to
changed
Explanation: First arrange the position of 2 4
to
A, B, C and D. Then, fix the position of the
remaining friends.
56. Option (3) is correct.
E
Explanation:
F D
E 50 m F
N
50 m
A B 70 m W E
D
C
S

51. Option (1) is correct. H 50 m G


∴ Required distance = DH = FG – ED
Explanation: According to the given
statements, only conclusion II follows. = 70 – 50 = 20 m
Conclusion I does not follow because there So, she is 20 m far at H from point D, in South
is no information about the importance of direction.
retailers and distributors.
57. Option (2) is correct.
52. Option (1) is correct.
Explanation:
Explanation: Television signal to be telecast
Mother
Similarly,
Radio signal to be broadcast.
Sister
Diwakar
53. Option (4) is correct. Lady

Explanation: Given, Mohini > Nita; Son


Mohini ≥ Sarita = Malini and Mohini < Hema Grandson
So, Hema > Mohini ≥ Sarita = Malini
Hence, second tallest cannot be determined. The lady is Diwakar ‘s sister.
MOCK TEST Paper-1 71
58. Option (2) is correct. 59. Option (1) is correct.
Explanation:
Explanation: 10 km
School N

1 4 10 km W E

5 S
10 km Home
2 3
Hence, he is 10 km in North at school from his
   home.
60. Option (1) is correct.
4 5 Explanation:
1 2 Y C L

3
+2 M Q Z +2

+2
Hence, by joining all the parts we can see the
correct figure. +2 A E N +2

O S +1
B


SOLUTIONS OF
Mock Test Paper 2
1. Option (4) is correct. and each of them holds equal authority.
Explanation: Somnath Temple is a Hindu While English and French serve as the official
temple dedicated to Lord Shiva. It is located working languages of the United Nations
in Prabhas Patan, Gujarat, India. The temple is Secretariat.
one of the twelve Jyotirlingas or holiest shrines 3. Option (4) is correct.
of Shiva. The Somnath Temple was destroyed
and rebuilt several times over the centuries, Explanation: The Battle of Chausa was fought
most recently in 1951. in 1539 between the forces of Babur and the
Kamakhya Temple is a Hindu temple Delhi Sultanate. Babur emerged victorious,
dedicated to the goddess Shakti or divine and this battle marked the beginning of the
energy. It is located in Guwahati, Assam, India. Mughal Empire in India.
The temple is one of the five Shakti Peethas or The Battle of Saraighat was fought in 1671
the holiest shrines of Shakti. The Kamakhya between the Mughal Empire and the Ahom
Temple is known for its unique architecture, Kingdom. The Ahoms were victorious, and
which features a lingam (phallic symbol) this battle helped to prevent the Mughal
inside a cave.
Empire from expanding further into Assam.
Ramnathaswamy Temple is a Hindu temple
The Battle of Saragarhi was fought in 1897
dedicated to Lord Rama. It is located in
between the British Indian Army and the
Rameswaram, Tamil Nadu, India. The temple
is one of the five dhams, or holiest pilgrimage Afghan tribesmen. The British Indian Army
sites for Hindus. The Ramanathaswamy was victorious, but they suffered heavy losses.
Temple is known for its imposing gopurams This battle is considered to be one of the most
(gateway towers) and its 1212 pillars. heroic battles in British Indian history.
Mahabodhi Temple is a Buddhist temple The First Battle of Panipat was fought in 1526
dedicated to the Buddha. It is located in Bodh between the forces of Babur and the Delhi
Gaya, Bihar, India. The temple is the site Sultanate. Babur emerged victorious, and this
where the Buddha attained enlightenment. battle marked the beginning of the Mughal
The Mahabodhi Temple is one of the most Empire in India.
important pilgrimage sites for Buddhists
around the world. 4. Option (1) is correct.
2. Option (2) is correct. Explanation: Trade-Related Investment
Measures (TRIMS) prohibit quantitative
Explanation: The United Nations utilizes six restrictions on imports by foreign investors,
official languages, namely Arabic, Chinese, making statement 1 correct. However, the
English, French, Russian, and Spanish. TRIMS agreement applies only to investment
Ensuring accurate interpretation and measures related to trade in goods, not services,
translation of these languages, both spoken
as stated in Article 1 of the TRIMS agreement,
and written is crucial to the organization's
making statement 2 incorrect. Regarding
operations as it facilitates clear and concise
existing TRIMS that are inconsistent with the
communication on global issues. The
agreement, members are required to notify the
General Assembly (Article 51 of the Rules of
WTO Council for Trade in Goods. TRIMS are
Procedure), the Economic and Social Council,
and the Security Council all incorporate these designed to restrict the preferences given to
languages during their meetings (Article 41 of domestic firms, allowing international firms to
its Rules of Procedure). Representatives from operate more easily in foreign markets. They
each country have the option to speak in any do not directly regulate foreign investment;
of the six official languages or any language instead, foreign investment is governed by
with interpretation available in one of the FEMA (Foreign Exchange Management Act)
six official languages. Official documents are and DIPP (Department of Industrial Policy
also distributed in all six official languages, and Promotion) guidelines and regulations.
MOCK TEST Paper-2 73
5. Option (3) is correct. and octopuses. The number of fossil species
Explanation: The last Nawab of Awadh was is estimated between 60,000 and 1,00,000
Wajid Ali Shah. He was the eleventh and additional species. It is the second largest
last King of Awadh and held his title for only phylum of invertebrate animals after the
9 years from 1847 to 1856. His kingdom was Arthropoda.
annexed by the British and he was sent to exile 10. Option (1) is correct.
in Kolkata. The Nawab is famously known
to be a patron of the arts and revived Kathak Explanation: Unicellular organisms reproduce
dance to its present glory. through asexual means of reproduction.
Asexual mode of reproduction involves cell
6. Option (4) is correct. division in which a single parent cells divides
Explanation: An individual with into two or more daughter cells. Cell division
hypermetropia can see distant entities usually occurs as part of a large cell cycle.
Different asexual modes of reproduction
absolutely but cannot see nearby objects
includes binary fission, multiple fission,
distinctly. Hypermetropia is even known as
fragmentation, budding etc.
farsightedness. Such a person has to keep
reading material much beyond 25 cm from 11. Option (2) is correct.
the eye for satisfying reading. This deficiency
can be rectified by utilizing a convex lens of Explanation: Jnanpith Award: The Jnanpith
reasonable power. Award is one of the most prestigious literary
awards in India. It is awarded for outstanding
7. Option (4) is correct. contributions to Indian literature. Instituted in
Explanation: Mumbai Port, situated on the 1961 by the Bharatiya Jnanpith organization,
western coast of India, is the country's largest the award recognizes exceptional works in
port in terms of size and shipping traffic. It any of the 22 official languages of India. The
benefits from a natural harbour with water recipient is presented with a cash prize, a
citation, and a bronze replica of the goddess
depths of 10-12 meters, allowing smooth
Saraswati, the Hindu goddess of knowledge
docking and passage for large cargo ships.
and wisdom.
Kochi, often referred to as the "Queen of the
Pritzker Architecture Prize: The Pritzker
Arabian Sea," held significant importance as
Architecture Prize is considered one of the
a spice trading centre on India's west coast highest honours in the field of architecture.
since the 14th century. It maintained trade Established in 1979 by the Pritzker family
relations with Arab merchants even during of Chicago, United States, the award aims
the pre-Islamic era. The Kingdom of Cochin to honour living architects who have made
later became a protectorate of the Portuguese significant contributions to the world of
in 1503. architecture. The laureate receives a bronze
Vishakhapatnam Harbour, located in Andhra medallion and a cash prize, and the prize
Pradesh, serves as an example of a landlocked ceremony takes place at various locations
harbour. around the world.
Chennai, situated on the eastern coast, holds Grammy Award: The Grammy Award is
the distinction of being the oldest port in the bestowed by the Recording Academy in the
region. United States. It is one of the most prestigious
awards in the music industry, recognizing
8. Option (1) is correct. outstanding achievements in various musical
genres. The Grammy Awards ceremony is an
Explanation: A plotter is a computer hardware
annual event where artists, producers, and
device much like a printer that is used for
other music professionals are honoured across
printing vector graphics. Therefore, it is an
a wide range of categories, including Best
output device. MICR, OMR and Barcode
Album, Best Song, and Best New Artist.
Reader are examples of input devices (any
Abel Prize: The Abel Prize is an international
hardware device that sends data to a computer
award presented for exceptional achievements
allowing user to interact with & control the
in the field of mathematics. Named after the
computers).
Norwegian mathematician Niels Henrik
9. Option (2) is correct. Abel, the prize was established in 2001 by
the Norwegian government. It is awarded
Explanation: The largest invertebrate is annually by the Norwegian Academy of
Mollusc. With over 80,000 species Mollusc Science and Letters. The laureate receives a
forms the second largest phylum of animal on substantial cash award and a commemorative
Earth and range from tiny snails to giant squid medal.
74 OSWAAL CUET (UG ) 10 Mock Test Papers GENERAL TEST

12. Option (2) is correct. Four primary impurities are carbon monoxide
Explanation: In India the ‘Van Mahotsav’ Day (CO), hydrocarbons, particulate matter—PM,
is observed on 1st July. It is an annual tree- and nitrogen oxides (NOx).
planting festival in India, celebrated in the first 18. Option (3) is correct.
week of July. This movement was initiated in
the year 1950 by India’s then Union Minister Explanation:
for Agriculture, Kulapati Dr. KM Munshi. Sports Terms associated with the sport
Cricket Dead Ball, Bouncer, Wide ball,
13. Option (2) is correct. Bye, Carron Ball, Duck, No Ball.
Explanation: Kalpana Chawla was the first Volleyball Ace, Attacker, Joust, Pipe, Roof,
Indian American astronaut to fly into space in Shank, Triple block.
November 1997 with a crew of six aboard the Basketball Dribble, Exceed, Free throw, Set
Columbia space shuttle STS-87. Chawla was shot, Chest pass, Lay-up, Slam
the first and second Indian woman to fly into dunk
space after astronaut Rakesh Sharma, who Football Dribble, Throw-in, Bicycle kick,
flew with the Soyuz T-11 in 1984. She died Back heel, Dummy run, Penalty,
during her second space flight aboard the Toe Poke, Wingers.
space shuttle Columbia in 2003.
19. Option (3) is correct.
14. Option (2) is correct.
Explanation: The vaccination for “small pox”
Explanation: Sarkaria commission was was invented by Edward Jenner in 1796. Before
appointed by the central government in 1983 the introduction of a vaccine, the mortality
to examine issues related to Centre-state of the severe form of smallpox-variola major
relations. was very high. Edward Jenner acted upon his
observation that milkmaids who caught the
15. Option (2) is correct.
cowpox virus did not catch smallpox.
Explanation: The Balwant Rai Mehta
Committee, constituted on 16th January 1957 20. Option (2) is correct.
by the Government of India, had the primary Explanation: The city of Lucknow is famous
objective of examining the functioning of the for the Chikankari Embroidery style. It is very
Community Development Programme and intricate and delicate thread work done on the
the National Extension Service. Chaired by muslin cloth originally. Nowadays the work is
Balwant Rai Mehta, the committee proposed being used to decorate a wide variety of fabric.
various recommendations, including the
establishment of a 3-tier Panchayati Raj 21. Option (3) is correct.
system. According to their recommendations, Explanation:
the 3-tier system is comprised of the following
Principal × Time × Rate
levels: S.I. =
100
Gram Panchayat at the village level.
18 12 
Panchayat Samiti at the block level. = ` 4000  
Zila Parishad at the district level.  12 100 
These tiers were intended to bring about = `720
effective local governance and empower
communities by decentralizing administrative
22. Option (3) is correct.
responsibilities. Explanation: We know that: (3)1 =3, (3)2 =9,
(3)3 =27, (3)4 = 81
16. Option (3) is correct.
Last digit of 34 = 1
Explanation: Suppose the soil is too acidic, and This implies 321 = (34)5 × 31
to modify the soil’s pH. To create it less acidic, Last digit of 321 = 1 × 3 =3
both quick lime and slaked lime is counted to
So, units digit in the expansion of (3)21 = 3
the soil, also learned as the liming of the soil.
Lime is alkaline, and it balances the acidity of Thus Remainder when 321 is dividing by 5 = 3
the soil and creates a neutral soil. 23. Option (1) is correct.
17. Option (4) is connect. Explanation: Single equivalent discount for
20% and 10%
Explanation: Diesel motors donate to
environmental pollution generated by exhaust  20  10 
  20  10    28%
emissions. It is liable for several fitness troubles.  100 
MOCK TEST Paper-2 75

Single equivalent discount for 28% and 10% 29. Option (1) is correct.
 28  10  Explanation: Multiple of 3 in between 1 to 20
  28  10    35.2%
 100  = 3, 6, 9, 12, 15, 18
So, S.P. of piano So, probability that the ticket drawn bears
15000  100  35.2  6
= = ` 9,720 multiple of 3 =
100 20
3
24. Option (3) is correct. =
10
Explanation: Suppose the numbers be 3x, 4x
and 5x. 30. Option (1) is correct.
∴ 5x + 3x = 4x + 52.
Explanation:
52
or 4x = 52 or x = = 13 or x = 13 1 + 0.6 + 0.06 + 0.006 + 0.0006 + ...
4
= 1.666 ... = 1.6
So, the smallest number = 3x = 3 × 13 = 39
6 2
= 1 =1
25. Option (3) is correct. 9 3
Explanation: Given, distance are same 31. Option (3) is correct.
 2xy 
Average speed =   km/h Explanation:
xy Given that,
2  30  60 
= 
E
 km/h C
 30  60  96°

2  30  60 3600
= 
90 90 A D B
= 40 km/h AD = DC = BC,
26. Option (3) is correct. and ∠BCE = 96°,
Now Suppose
Explanation: No. of books in each stack of
equal height HCF of 84, 90 and 120. ∠ACD = a = ∠DAC
Now, 84 = 2 × 2 × 3 × 7 ∴ ∠CDB = 2a = ∠CBD
90 = 2 × 3 × 3 × 5 We know that, the angles of the base of an
120 = 2 × 2 × 2 × 3 × 5 isosceles triangles are equal.
∴ HCF of 84, 90 and 120 = 2 × 3 = 6 ∴ ∠ACB = 180° – 96 = 84°
or ∠ACD + ∠DCB = 84°
27. Option (3) is correct.
or a + 180° – 4a = 84°
Explanation: Given:
or 180° – 3a = 84°
2   2 p  2   7  13  17  4   p  1
or 3a = 180° – 84° = 96°
7
96°
⇒ 3p + 44 = 56 or a = = 32°
⇒ 3p = 12 3
⇒p=4 or ∠DBC = 2a
So, ascending order of numbers, 2, 3, 4, 7, 10, = 2 × 32° = 64°
13, 17
Hence, Median = 7 (Middle value of series) 32. Option (3) is correct.
28. Option (3) is correct. Explanation: Given: (10.15)2 = 103.0225
Explanation: A finish the work in 40 days. ⇒ (1.015)2 = 1.030225
B finish the work in 60 days. ⇒ (101.5)2 = 10302.25
then time taken to do work together ∴ 1.030225 + 10302.25
xy
= days =
xy (1.015)2 + (101.5)2

40  60 2400 = 1.015 + 101.5


=   24 days
40  60 100 = 102.515
76 OSWAAL CUET (UG ) 10 Mock Test Papers GENERAL TEST

33. Option (2) is correct. 42 − 38 76 − 66


= =
Explanation: Weight to height ratio is 19 × 7 11 × 19
calculated below:
Weight 59 4 10
Apeksha’s – = = 32.7 = =
Height 1.8 19 × 7 11 × 19

Apsara’s – 51 = 29.65 So, the required ratio


1.72 4 10 4 10
= : = :
Aradhana’s – 62 = 39.49 19 × 7 11 × 19 7 11
1.57 = 44 : 70 = 22 : 35
Archana’s – 74 = 44.05 37. Option (1) is correct.
1.68
So, Apsara has the least weight to height Explanation: More distance covered in one
ratio. hour without stoppage = 72 – 60 = 12 km

34. Option (2) is correct. 12


So required Time = × 60
72
Explanation: Distance covered by wheel in 1
one   60 = 10 minutes
6
Revolution = Circumference of wheel
11000 11 38. Option (3) is correct.
= = metre
5000 5
Explanation:
11 1100
= ×100 cm = cm T
5 5  R 
A  P1  
= 220 cm  100 
Now, 2πr = 220 3 3
 7.5   75 
or 2 × 22 × r = 220  64000  1    64000  1  1000 
7  100   
220 × 7
or r=  43 
3

2 × 22  64000  
1540  40 
r=
44 64000  43  43  43 64  43  43  43
 
= 35 cm 40  40  40 444
35. Option (1) is correct. 5088448

64
Explanation: We know that,
a = ` 79507
In-radius =
2 3 ∴ C.I. = ` (79507 – 64000)

where a = side of equilateral triangle = ` 15,507

Circum-radius =
a 39. Option (2) is correct.
3
a a Explanation: Given: P = 7 + 4 3 and PQ = 1
Now, required ratio = :
2 3 3
1 1 74 3
or required ratio = 1 : 2   
P 7  4 3 (7  4 3 )(7  4 3 )
36. Option (1) is correct.
On rationalising the denominator, we get
Explanation: By using the rule of alligation,
we get 7 −4 3
= = 7 −4 3
Mixture I Mixture II 49 − 48
Darjeeling tea Darjeeling tea
1
Now, Q = = 7 −4 3
4 2 P
11 7
6 1 1 74 3
  
19 Q 7  4 3 (7  4 3 )(7  4 3 )
6 2 4 6
− −
19 7 11 19 74 3
MOCK TEST Paper-2 77

1 1
43. Option (2) is correct.
   ( 7  4 3 )2  ( 7  4 3 )2
2 2 Explanation:
P Q
For consonants = Position value of alphabet
 2[(7 )2  ( 4 3 )2 ] For vowels =
= 2(49 + 48) = 2 × 97 = 194
A Reverse

Order
 (27 – 1 = 26) × 2 = 52
40. Option (2) is correct. Reverse
E 
Order
 (27 – 5 = 22) × 2 = 44
Explanation: Given: 20% of rice is lost in Reverse
I   (27 – 9 = 18) × 2 = 36
transportation. Order

4 64 Reverse
O   (27 – 15 = 12) × 2 = 24
So, the remaining rice  16   quintals Order
5 5
To gain 25%, Reverse
U   (27 – 21 = 6) × 2 = 12
Order
5632  125 
Required S.P. = `   = ` 7040 As, A R O U N D 52 18 24 12 14 4
 100 

So, the required rate = `  7040  per quintal


 64 
 
 5 
= `  7040  5  per quintal and F I X 6 36 24
 64 
 
= ` 550 per quintal
Similarly
41. Option (4) is correct.
P L A S T I C 16 12 52 19 20 36 3
Explanation:
E 20 m F
final point

44. Option (2) is correct.


50 m
B Home Explanation:
A
T N
Son
M G

Mother Son
90 m 120 m Sister
R K

It is clear from the graph, T is not the paternal


grandfather of M.

C 30 m D
45. Option (1) is correct.
So, required distance AF = 120 – 90 = 30 m
Explanation:
Thus, she is 30 m in North from her house.
As,
42. Option (4) is correct. 109 114 139

Explanation: Since we have 365 days in a year, +5 +25


so from going to 18th Feb 1998, we will come
across 52 weeks and one day, therefore 18th Same as,
Feb 1998 will be Wednesday. Similarly, 18th
313 318 343
February 1999 will be a Thursday, because
there is no leap year involved between the
+5 +25
year 1997 and year 1999.
78 OSWAAL CUET (UG ) 10 Mock Test Papers GENERAL TEST

46. Option (4) is correct. 50. Option (4) is correct.


Explanation: Explanation: Fix Raju’s position in the extreme
Reverse left and arrange according to the given
F R E N C H 
Order
 21 9 22 13 24 19 conditions.
Now, 21 + 9 + 22 + 13 + 24 + 19 = 108 R K S P D
⇒ 108 + 6 (total alphabets in the word)
= 114
and 51. Option (4) is correct.
Reverse Explanation:
L O S S 
Order
 15 12 8 8
32 * 8 * 14 * 7 *16
Now, 15 + 12 + 8 + 8 = 43 ⇒ 43 + 4 = 47
Similarly, – ÷ =
Reverse
∴ 32 – × 14 ÷ 7 = 16
C O U R S E   24 12 6 9 8 22
Order ⇒ 32 – × 2 = 16
Now, 24 + 12 + 6 + 9 + 8 + 22 = 81 ⇒ 32 – 16 = 16
⇒ 81 + 6 = 87
Note: Reverse positional value = 27 – Original 52. Option (1) is correct.
positional value. Explanation:
3 km
47. Option (1) is correct. B final point P

Explanation: P Q F 6 km E
Correct relation between the given classes 18 km N
final point
Reptiles Q 10 km W E

Q
s

A
ou

S
Snakes
on

Starting 9 km D
is

point
Po

\ Required distance
48. Option (3) is correct. = PQ = BA – ED
Explanation: According to the statements, = 18 – 10 = 8 km
Hence, Q is now at F, 8 km in South from
Stalls
position of P.
X
offices Bank
53. Option (1) is correct.
Explanation: ‘Hum’ is the sound made by bees.
I. () II. () III. () IV ()
Similarly,
Hence, only conclusion III follows.
‘Hiss’ is the sound made by snakes.

49. Option (3) is correct. 54. Option (4) is correct.


Explanation: Explanation: Since 2 and 4 is common in both
P R L dice, therefore 3 is opposite to 1.
+1 –2 +1 55. Option (3) is correct.
Q P M Explanation:
brother
+1 –2 +1 D B L
R N N Daughter Son
+1 –2 +1 sister N
P T
S L O
+1 –2 +1
brother
From the figure, it is clear that N is the brother
T J P
of T.
MOCK TEST Paper-2 79
56. Option (3) is correct. 59. Option (2) is correct.
Explanation: Both conclusions are incorrect Explanation: So,
as no information about activeness in
given and also number of schools are not
decreasing.

57. Option (3) is correct.


Explanation:
Grandmother will complete the figure.

60. Option (2) is correct.


Only child
daughter Explanation:
50 km
E D
Chetan Girl N
brother
110 km W E
Chetan is the brother of that girl.
S
58. Option (3) is correct. B C
50 km
Explanation: 40 km
A Starting point
4 3
2 From figure,
3 1 1 4 EB = DC = 110 km
2 ∴ Required distance AE
= AB + BE = 40 + 110 = 150 km

Hence, by joining all the parts we can see the Hence, bird is 150 km far at North from starting
correct figure.
point.




SOLUTIONS OF
Question Paper 3
1. Option (4) is correct. The nucleus is a double-membraned
Explanation: Ribosomes are small organelles eukaryotic cell organelle that contains genetic
that are found in the cytoplasm of cells. They material. The nucleus is found in both plant
are responsible for making proteins. Proteins and animal cells. It is the control centre of
are essential for all living things and they are the cell and it contains the DNA, which is the
involved in a wide range of functions, such as: genetic material of the cell. The DNA stores
• Building and repairing tissues the instructions for making proteins and other
• Producing enzymes molecules that the cell needs to function.
• Transporting substances in and out of cells The cell membrane is the outermost covering
• Fighting infection of all cells. It is a selectively permeable
membrane that allows certain molecules to
Golgi apparatus is a large organelle that is
found in the cytoplasm of cells. It is responsible enter and exit the cell. The cell membrane
for packaging and transporting proteins and is found in both plant and animal cells. It is
other substances around the cell. The Golgi responsible for maintaining the cell's internal
apparatus also plays a role in the production environment and protecting the cell from its
of lysosomes, which are organelles that break surroundings.
down waste products. 3. Option (3) is correct.
Mitochondria are small organelles that
Explanation: National Bank for Agriculture
are found in the cytoplasm of cells. They
and Rural Development (NABARD) is a
are responsible for producing energy for
development bank set up by the Government
the cell. Mitochondria are often called the
"powerhouses of the cell" because they of India in 1982. It is headquartered in Mumbai,
produce most of the cell's energy. Maharashtra. NABARD provides financial
assistance to banks, financial institutions, and
So, the statements that Ribosomes make
cooperatives engaged in agricultural and rural
protein, the Golgi apparatus is involved in
development activities.
transporting substances in and out of the cell,
and Mitochondria produce energy for the cell National Housing Bank (NHB) is a
are all correct. government-owned financial institution that
was established in 1988. It is headquartered
2. Option (2) is correct. in Delhi. NHB provides financial assistance
Explanation: Plastids are found in plant to individuals and institutions for housing
cells, not animal cells. They are responsible purposes.
for photosynthesis, which is the process by Insurance Regulatory and Development
which plants make their own food. There Authority (IRDAI) is a statutory body that
are three main types of plastids: chloroplasts, was established in 2000. It is headquartered in
chromoplasts, and leucoplasts. Chloroplasts Hyderabad, Telangana. IRDAI regulates and
are the most common type of plastid and
supervises the insurance industry in India.
they are responsible for photosynthesis.
Chromoplasts give plants their colourful Small Industries Development Bank of India
pigments, such as the red colour of tomatoes (SIDBI) is a financial institution that was
and the orange colour of carrots. Leucoplasts established in 1990. It is headquartered in
are colourless plastids that store starch and Lucknow, Uttar Pradesh. SIDBI provides
other nutrients. financial assistance to small and medium
Lysosomes are known as the "suicide bags" of enterprises (SMEs).
the cell because they contain enzymes that can 4. Option (3) is correct.
break down cellular components. Lysosomes
are found in both plant and animal cells. They Explanation: Wodeyars ruled Mysore under
play a role in cell defence, autophagy, and a subsidiary alliance with the British after
apoptosis. the defeat of Tipu Sultan at the Battle of
MOCK TEST Paper-3 81

Seringpatnam. The Wodeyar dynasty ruled is located in the Kanyakumari district of Tamil
Mysore from 1399–1947. It is believed to be Nadu. Kanyakumari is a town that is located on a
the only royal family that ruled for more than small island at the tip of India.
500 years over the Kingdom. The Battle of The statement that the Easternmost: Kibithu in
Seringapatan took place between Apr–May Arunachal Pradesh, Westernmost: is Ghuar Mota
1799. Tipu Sultan was killed in the War.
in the Kutch region of Gujarat, Northernmost:
5. Option (4) is correct. Indira Col, and Southernmost (Mainland):
Kanyakumari are all correct.
Explanation: Kuroshio Current is a warm
current of the Pacific Ocean. It originates in 8. Option (3) is correct.
the East China Sea and flows northeast along
Explanation: In Microsoft Excel, the MIN
the coast of Japan. The Kuroshio Current is
function networks the smallest value among
one of the strongest currents in the world and
the values passed an arguments. For example,
it plays a major role in the climate of East Asia.
the fastest time in a race, the smallest sales
Humboldt Current is a cold current of the
number, or the lowest temperature. Arguments
Pacific Ocean. It originates in the Peru-Chile
can be provided, for example-
Trench and flows northward along the coast
of Chile and Peru. The Humboldt Current is = MIN (15, 60)
responsible for the cold climate of the west = MIN (B1, B2, B3)
coast of South America. = MIN (A1: A10)
Labrador Current is a cold current of the
Atlantic Ocean. It originates in the Labrador 9. Option (2) is correct.
Sea and flows southward along the coast of Explanation: A virtual image is formed by
Newfoundland and Labrador. The Labrador
a convex mirror. The image constructed
Current is responsible for the cold climate of
is usually reduced because of the mirror
the east coast of Canada.
curvature. A convex mirror is comprehended
Canaries Current is a cold current of the as a diverging mirror. It is a curved mirror
Atlantic Ocean. It originates in the Canary with a reflective surface that bulges towards
Islands and flows southward along the coast
the light origin.
of Africa. The Canaries Current is responsible
for the cool climate of the west coast of Africa. 10. Option (4) is correct.
6. Option (3) is correct.
Explanation: The Battle of Chamkaur was
Explanation: Siraj-ud-Daulah became the fought between the forces of the Mughal
Nawab of Bengal after the death of Alivardi Empire under Aurangzeb and the forces of
Khan in 1756. He was the last self-governing the Sikh Guru Gobind Singh in 1704. The
Nawab of Bengal. Murshid Ali Khan was the Sikhs were defeated and Guru Gobind Singh
first Nawab of Bengal. Mir Jafar served as the was killed.
first Nawab of Bengal under British Influence. The Battle of Swally was fought between
7. Option (3) is correct. the Portuguese and the British East India
Company in 1612. The British were victorious
Explanation: Easternmost: Kibithu in Arunachal
and this victory helped them to establish a
Pradesh is the easternmost point of India. It is
foothold in India.
located in the Lohit district of Arunachal Pradesh.
The Battle of Plassey was fought between the
Kibithu is a small village that is located on the
forces of the Mughal Empire under Nawab
border with China.
Siraj ud-Daulah and the forces of the British
Westernmost: Ghuar Mota in the Kutch region East India Company under Robert Clive in
of Gujarat is the westernmost point of India. It 1757. The British were victorious and this
is located in the Bhuj district of Gujarat. Ghuar victory gave them control of Bengal.
Mota is a small village that is located on the border
The Battle of Buxar was fought between the
with Pakistan.
forces of the Mughal Empire under Shah
Northernmost: Indira Col is the northernmost Alam II, the Nawab of Awadh Shuja ud-Daula,
point of India. It is located in the Ladakh region and the Nawab of Bengal Mir Qasim on one
of Jammu and Kashmir. Indira Col is a mountain side and the forces of the British East India
pass that is located on the border with China. Company under Hector Munro on the other
Southernmost (Mainland): Kanyakumari is side in 1764. The British were victorious and
the southernmost point of India's mainland. It this victory gave them control of most of India.
82 OSWAAL CUET (UG ) 10 Mock Test Papers GENERAL TEST

11. Option (4) is correct. 16. Option (4) is correct.


Explanation: Carbon dioxide is the gas that Explanation: Rehuke Khim or Cowrie shawl
donates most to the sensations of global is the most important cloth meant for the rich
warming. Carbon dioxide is developed as a man of the Yimchunger tribe in Nagaland. In
byproduct of cellular respiration and burning Naga society, cowrie is a symbol of marital
of fossil fuels. It is also generated by human status.
industries, which increases global warming.
17. Option (4) is correct.
12. Option (2) is correct.
Explanation: The recipient of the Lokmanya
Explanation: The electron was found by J. Tilak Award 2023 is Prime Minister Narendra
J. Thomson. Thomson’s investigations with Modi.
cathode-ray pipes revealed that all atoms The Lokmanya Tilak Award is a prestigious
possess small negatively charged subatomic
award given by the Tilak Smarak Mandir Trust
particles on electrons.
to individuals who have made significant
13. Option (4) is correct. contributions to the fields of politics, social
Explanation: Dr Thani Al Zeyoudi hails from work, and public service. The award is named
United Arab Emirates (UAE). He is the Minister after Lokmanya Tilak, a prominent leader of
of State for Foreign Trade of the UAE and the the Indian independence movement.
Minister of State for Artificial Intelligence. He Prime Minister Modi was awarded the
was chosen as the chair of the 13th Ministerial Lokmanya Tilak Award for his outstanding
Conference of the World Trade Organization contribution to the development of India.
(WTO) in 2023. He was recognized for his leadership in the
Dr Al Zeyoudi is a graduate of the University areas of economic growth, social welfare, and
of Cambridge and the Massachusetts Institute national security.
of Technology. He holds a PhD in Computer
Science from MIT. He is a leading expert The award was presented to Prime Minister
in artificial intelligence and has published Modi by the Governor of Maharashtra, Bhagat
extensively on the topic. He is also a member Singh Koshyari, at a ceremony in Mumbai on
of the World Economic Forum's Global Future August 1, 2023. In his acceptance speech, Prime
Council on Artificial Intelligence. Minister Modi paid tribute to Lokmanya Tilak
Dr Al Zeyoudi is a strong advocate for free and said that he was inspired by his vision of a
trade and open markets. He believes that strong and prosperous India.
trade can help to boost economic growth and
development. He is also a strong supporter of
18. Option (3) is correct.
the WTO and believes that the organization Explanation: The Malimath Committee
can play a vital role in promoting free trade on Reforms of Criminal Justice System
and global economic cooperation. was constituted by the Ministry of Home
Dr Al Zeyoudi's appointment as the chair of Affairs under the Chairmanship of Justice
the 13th Ministerial Conference of the WTO is V.S. Malimath, former Chief Justice of the
a significant achievement for the UAE. It is a
Karnataka and Kerala High Courts. It has been
recognition of the UAE's growing role in the
constituted to recommend the admissibility
global economy and its commitment to free
trade. of confessions made before a police officer as
evidence in a court of law.
14. Option (1) is correct.
19. Option (4) is correct.
Explanation: Sir C.V. Raman was an Indian
physicist who won the 1930 Nobel Prize in Explanation: All of these articles are
physics for his pioneering work in the field of fundamental rights guaranteed to all citizens
light scattering. He found that some refracted of India by the Constitution.
light changes its wavelength when light passes Article 15 prohibits discrimination on the
through a transparent material. grounds of religion, race, caste, sex, or place
of birth. This means that no one can be
15. Option (2) is correct. discriminated against on the basis of their
religion, race, caste, sex, or place of birth.
Explanation: J.J Thomson was an English
This article also prohibits the state from
physicist. He received the Nobel Prize in
discriminating against any citizen on any of
physics for the discovery and identification
of electron. He won Nobel Prize in 1906. these grounds.
Electrons are the first subatomic particle to be Article 19 guarantees certain rights regarding
discovered. freedom of speech, assembly, association,
MOCK TEST Paper-3 83

movement, residence, and profession. 23. Option (2) is correct.


This means that all citizens have the right
Explanation: Any number is divisible by 88 if
to freedom of speech, to assemble peacefully,
it is divisible by 8 and 11.
to form associations, to move freely
So, the given number is divisible by 8 if 8y2 is
throughout the country, to reside in any part
divisible by 8.
of the country, and to practice any profession
they choose. When y = 3 then 832 ÷ 8 = 104
Article 23 prohibits traffic in human beings For divisibility by 11,
and forced labour. This means that no one can Sum of digits at odd places – sum of digit at
be forced to work against their will. This article even places = 0
also prohibits the employment of children ⟹ (1 + 3 + x + 5 + 3) – (3 + 0 + 5 + 8 + 2)
below the age of 14 years in any factory or =0
mine. ⟹ x + 12 – 18 = 0
Article 25 guarantees freedom of conscience ⟹x=6
and free profession, practice, and propagation ∴x+y=6+3=9
of religion. This means that all citizens have
the right to freedom of conscience and the 24. Option (3) is correct.
right to profess, practice, and propagate their
religion. This article also prohibits the state Explanation:
from interfering with the free exercise of Given numbers = 12, 15, 7, 8 and x +13
religion. Where x = 2
So, last number = 15
20. Option (4) is correct. Now mean = (sum of all numbers) / 5
= 57 / 5 = 11.4
Explanation: Bull’s eye is used in the game of
shooting sport. In this game participants uses 25. Option (2) is correct.
handguns at paper targets at fixed distances
and time limits. Focus of this game on accuracy Explanation: Given: 20% of rice is lost in
and precision. This game is mostly played in transportation.
USA and Canada. 4 64
So, the remaining rice  16   quintals
5 5
To gain 25%,
21. Option (2) is correct.
5632  125 
Explanation: Required S.P. = `   = ` 7040
 100 
We know that,

LCM of fractions = LCM of numerators  7040 


So, the required rate = `  per quintal
HCF of denominators 64 
 
2 4 5 LCM of 2, 4, 5  5 
∴ LCM of , , =
3 9 6 HCF of 3, 9, 6 = `  7040  5  per quintal
20  64 
 
=
3
= ` 550 per quintal
22. Option (4) is correct.
26. Option (4) is correct.
Explanation:
35 Explanation: Given, a, 1, b are in A.P.
Downstream rate of boat (x) = = 7 km/h
5 ab
So, 1 
35 2
Upstream rate of boat (y) = = 5 km/h
7
or a + b = 2 ...(i)
1
speed of boat in still water = ( x + y ) and 1, a, b are in G.P.
2
a2 = b ...(ii)
1
= (7 + 5) Put this value of b in eq. (i)
2
a + a2 = 2
1
=  12  6 km/h or a2 +a–2=0
2
84 OSWAAL CUET (UG ) 10 Mock Test Papers GENERAL TEST

or a2 + 2a – a – 2 = 0 AB
or a(a + 2) – 1(a + 2) = 0 So, sin 60° = AC
or (a – 1) (a + 2) = 0
3 120 3
From Eq. (ii), we get ⇒ = [sin 60° = ]
2 AC 2
if a = –2 then b = 4
if a = 1 then b = 1 ⇒ AC = (120 × 2 ) / 3 = 80 3 m
But given a ≠ b then b = 4. ∴ The length (in m) of the string is 80 3 m

27. Option (2) is correct. 29. Option (4) is correct.


Explanation: Explanation:
Given that, DABC is an isosceles right angled Number of boys = 40 + 35 + 15 + 35 + 40 + 20
triangle. = 185
A Number of girls = 10 + 25 + 10 + 35 + 30 + 20
= 130
E ∴ Required ratio = 185:130
D = 37:26

B
30. Option (1) is correct.
C

∠C = 90° Explanation: Given: x = 5 + 2 6


Now, 1 1 52 6
AC2 + CB2 = AB2   
x 5  2 6 ( 5  2 6 )( 5  2 6 )
or 2 BC2 = (AD + DB)2
or 2 BC2 = AD2 + DB2 + 2 AD DB  ...(i) On rationalising the denominator, we get
Now, ∆CEB and ∆CED are right angles.
5 −2 6
then, = = 5 −2 6
25 − 24
CD2 = CE2 + ED2
2
and BC2 = CE2 + BE2 Now,  x  1   x  1  2
 
BC2 – CD2 = (CE2 + BE2) – (CE2 + ED2)  x x
= CE2 + BE2 – CE2 – ED2
 52 6 52 6 2
⇒ BC2 – CD2 = BE2 – ED2 2
= (BE + DE) (BE – DE)  1 
 x    12
= (AE + DE) (BE – DE)  x
= AD · BD ...(ii) 1
⇒ x  12  2 3
From eq (i) and (ii) x
AD2 + DB2 = 2CD2
28. Option (4) is correct. 31. Option (2) is correct.
Explanation: Explanation: Distance covered in one hour
Height of the kite = 120 m without stoppage = 80 – 72
The angle is made by the string = 60° = 8 km
A
8
Required time = × 60 min
80
120 m
= 6 minutes
So, train stops for 6 minutes per hour
60°
C B 32. Option (1) is correct.
As per the given information the figure is Explanation: We know that,
drawn.
Period of stoppage per hour
The distance of kite from the ground (AB) =
120 m and ∠ACB = 60°  Difference in speeds 
  60  minutes
The length of the string = AC  faster speed 
 80  64 
=  60  minutes
 80 
 16 
   3  minutes
 4 
MOCK TEST Paper-3in speeds
 Difference  85
  60  minutes
 faster speed 
36. Option (2) is correct.
 80  64 
=  60  minutes
 80  Explanation:
 16  Principal × Time × Rate
   3  minutes S.I. =
 4  100
= 12 minutes S.I. 100 R  100
Principal =   ` 50
33. Option (4) is correct. Time  Rate 2R

Explanation: Let, capacity of every container 37. Option (3) is correct.


= x litre
3x Explanation: According to question,
In the I container, milk = litres
4 B’s work = 1st, 3rd, 5th, 7th, 9th and 11th days
x
Water = litres = 6 days
4
5x
In the 2nd container, milk = 7 litres 1
A work for 5 days
2x 3
Water = 7 litres
1 16 1
Now, work done by A =  
On mixing both, we get 48 3 9
3x 5x 1 8
quantity of milk = + Remaining work = 1  
4 7 9 9
None
21x + 20 x 41x 8
= = litres ∴ B complete work in 6 days
28 28 9
x 2x 7 x + 8x 6×9
quantity of water = + = litres ∴ B complete 1 work in days
4 7 28 8
15x
= litres ∴ Required time for B to finish 4 times of
28
6×9
41x 15x work = ×4
So, the required ratio = : 8
28 28 54 × 4
= 41x : 15x = 41 : 15 =
8
34. Option (3) is correct. = 27 days

Explanation: Single equivalent discount 38. Option (1) is correct.


 xy 
x  y  % Explanation: Let time taken by (B + C) in
 100 
doing one work = x days
 50  25 
  50  25  % So, time taken by A = x days
 100 
1 1
= (75 – 12.5)% = 62.5% Now B‘s 1 day work = -
36 x (i)
∴ C.P. of article = (100 – 62.5)% of 2375
 2375  37.5  According to question, we have
=`   = ` 890.625
 100  1 1
B’s 1 day work = -
Actual Cost = ` 890.625 + 165 = ` 1055.625 x 60 (ii)
To gain 62.5%, required S.P. From eqs. (i) and (ii), we get
 1055.625  162.5  1 1 1 1
=`   = ` 1715.39   
 100  x 60 36 x

35. Option (4) is correct. 2 1 1 35 8


or    
x 60 36 180 180
Explanation: Number of balls in bags x and y
respectively 2a and 3a. 180  2 360
or x    45
∴ 3a – 5 = 2a + 5 8 8
or a = 5 + 5 = 10 1 1
∴ B’s 1 day work = -
So, the total number of balls = 5a = 5 × 10 = 50 36 45

50 54 1
So, balls in each bag = = 25 = 
2 180 180
86 OSWAAL CUET (UG ) 10 Mock Test Papers GENERAL TEST

Now time taken by B = 180 days 42. Option (1) is correct.


1 1 Explanation: The year 2001 was an ordinary
(A + C)’s 10 days work = 10   
 45 60  year and in an ordinary year 1st day = Last
day
43
= 10  
(remember) 1st January = 31st December
 180  As, given that, 1st January = Monday
10  7 7 Hence, 31st December = Monday
= 
180 18 43. Option (2) is correct.
∴ Remaining work will be Explanation:
7 18  7 11 Reverse
= 1   As, R E A S O N  Order
N O S A E R
18 18 18
N = 14 ⇒ 1 + 4 = 5, O = 4, S = 19
180 ⇒ 1 + 9 = 10
B’s work = 11 × = 110 days A = 1, E = 2, R = 18 ⇒ 1 + 8 = 9
18
[∵ Codes for vowels A E I O U
39. Option (2) is correct. ] 1 2 3 4 5
Explanation:
So, REASON = 5410129
As per question, we have
Reverse
Length of the rubber band = 3d + 2πr and HEALTH 
Order
 YHTLAEH
= (3 × 10 + 10π) cm Y = 25 ⇒ 2 + 5 = 7, H = 8 ⇒ 8 × 2 = 16
= ( 30 + 10π) cm T = 20 ⇒ 2 + 0 = 2, L = 12 ⇒ 1 + 2 = 3
A = 1, E = 2, H = 8 ⇒ 8 × 2 = 16
So, HEALTHY = 716231216
40. Option (1) is correct.
Similarly,
Explanation: Reverse
DIVULGE 
Order
 EGLUVID
 R 
T
 E = 2, G = 7, ⇒ 7 × 2 = 14, L = 12 ⇒ 1 + 2 = 3
C.I  P  1  R  T  1T

C.I  P  1  100 R   1 U = 5, V = 22, ⇒ 2 + 2 = 4, I = 3


C.I  P  1  100   1 D=4⇒4×2=8
 100 
 10 4 

 4000  1  10  44  1 Hence, DIVULGE = 21435438
 4000  1  100 10   1
 4000  1  100   1 44. Option (3) is correct.
 11 1004  
 4000  11  44  1 Explanation:
 4000  10 11   1 T
 4000  10   1
1011  11
11   11  Mother
 4000  11  11  11  11  1 brother
10  10  10 11
10  1 S R
 4000  11  11  11 
 4000  10  10  10  10  1
10  1010000
 14641 10  10  Father
 4000  14641  10000  brother
 4000  14641 10000 10000  P Q
 4000  10000 
 10000
4641  Therefore, it is clear that P is the grandson of T.
 4000  4641
 4000  10000 4641
 4000  10000 45. Option (2) is correct.
10000
Explanation:
= 4000 × 0.4641
= ` 1856.4

41. Option (4) is correct.


Explanation:

will complete the given figure.


MOCK TEST Paper-3 87
46. Option (4) is correct. 49. Option (1) is correct.
Explanation: Explanation:
As,
F O R E N S I C

6 12 18 22 14 19 18 3

[∵ For vowels = Positional value in reverse i


order and for consonant = Positional value]
and D O R M

4 12 18 13

Similarly,

C A R A M E L 50. Option (1) is correct.


Explanation: First fix the position of D, then,
fix the position of other friends.
3 26 18 26 13 22 12
D
47. Option (2) is correct. B E

Explanation:
F C
Correct relationship between the given classes

Parents G H
A
Rich
Persons Farmers
51. Option (3) is correct.
Explanation:
48. Option (2) is correct. Starting N
End point
Explanation: point 40 m
30 m W E
According to the statements,
30 m S
Copies 20 m

Envelopes Cards Postcards It is clear from the figure that from the starting
point Lokesh is now in West direction.

I. () II. () III. ()

Hence, conclusions I and III follow


88 OSWAAL CUET (UG ) 10 Mock Test Papers GENERAL TEST

52. Option (2) is correct. 55. Option (1) is correct.

Explanation: Explanation: In the mirror image left becomes


right and right becomes left.
Father

56. Option (4) is correct.


Father Explanation:
Given : 54 * 6 * 5 * 14

Ritika Son (Man)


÷ + =
∴ 54 ÷ 6 + 5 = 14
Sister ⇒ 9 + 5 = 14

53. Option (2) is correct. 57. Option (3) is correct.


Explanation: Explanation:

,
58. Option (3) is correct.
Explanation:
50 49 47 44 40 35 29 22
54. Option (2) is correct.
Explanation: –1 –2 –3 –4 –5 –6 –7

Given,
59. Option (4) is correct.
Explanation:
12 Aman So,

Baman 18

So, Maximum boys in the row = 12 + 6 + 18 will complete the figure.


= 36
60. Option (2) is correct.
Explanation: Both conclusions are correct
according to given statements.


SOLUTIONS OF
Question Paper 4
1. Option (4) is correct. humanitarian assistance to children in need,
Explanation: Ahom Dynasty: The Ahom promotes child rights, and supports early
Dynasty ruled the Ahom Kingdom in the childhood development.
region of Assam, which is now a state in The United Nations Security Council (UNSC)
northeastern India. The dynasty lasted for is headquartered in New York City, USA.
several centuries, with its origins dating back to The UNSC is one of the six principal organs
the 13th century. The Ahom rulers established of the UNO. It is responsible for maintaining
a strong administration and a unique culture international peace and security. The UNSC
in Assam. They successfully resisted Mughal has five permanent members (China, France,
invasions and maintained their sovereignty Russia, the United Kingdom, and the United
for a significant period of time. States) and 10 rotating members.
Parmar Dynasty: The Parmar dynasty ruled in The United Nations Educational, Scientific
the Malwa region of central India. The dynasty and Cultural Organization (UNESCO) is
is known for its rule during the medieval headquartered in Paris, France. UNESCO
period, particularly in the 9th to 14th centuries. is a specialized agency of the UNO that
The Parmar rulers played a significant role in promotes education, science, culture, and
the political landscape of central India and communication. It works to build peace
contributed to the cultural and architectural in these areas and to foster international
development of the region. cooperation.
Gurjar Pratihara Dynasty: The Gurjar
Pratihara dynasty ruled in the northwestern 3. Option (1) is correct.
region of India, with its capital at Ujjain. They
were prominent during the medieval period Explanation: The “Treaty of Salbai” was signed
and played a major role in the history of the on 17 May 1782 by representatives of the
Indian subcontinent. The dynasty is known for Maratha Empire and the British East India
its expansion, military prowess, and patronage Company after long negotiations to settle
of art and culture. the outcome of the first Anglo-Maratha War.
Salbai is situated in Gwalior District, Madhya
Chandel Dynasty: The Chandel Dynasty
Pradesh.
ruled the region around Khajuraho, known
for its famous group of temples with intricate 4. Option (3) is correct.
sculptures. The dynasty existed during the
Explanation: Lord Canning: As the Governor-
medieval period and was centred in present-
General of India from 1856 to 1862, Lord
day Madhya Pradesh, India. The Khajuraho
Canning held a prominent role. During his
temples are renowned for their stunning
tenure, the pivotal Government of India Act, of
architecture and artistic representations. 1858 was enacted. This legislation established
2. Option (3) is correct. that the position of Viceroy would be held by
the same individual serving as the Governor-
Explanation: The United Nations Organisation
General of India. In addition to his role as
(UNO) is headquartered in New York City, Governor-General, Lord Canning assumed
USA. The UNO is the main international the position of the inaugural Viceroy of India.
organization for global cooperation. It was Notably, he oversaw the withdrawal of the
founded in 1945 after World War II to prevent "Doctrine of Lapse" policy during his time.
future wars and to promote peace and security. Lord Ripon: Serving as India's Viceroy
The UNO has 193 member states. from 1880 to 1884, Lord Ripon's term was
The United Nations Children's Emergency marked by significant events. Noteworthy
Fund (UNICEF) is also headquartered in actions during this period include the repeal
New York City, USA. UNICEF is a specialized of the Vernacular Press Act in 1882, which
agency of the UNO that works to improve the aimed to provide greater freedom to the
lives of children around the world. It provides press. Additionally, a resolution introduced
90 OSWAAL CUET (UG ) 10 Mock Test Papers GENERAL TEST

in 1882 marked the initiation of local self- Mahe is the smallest district in India by area. It
governance in India. The year 1882 also saw is located in the Union Territory of Puducherry.
the establishment of the Hunter Commission, Mahe is a coastal district and is surrounded
which focused on educational reforms. Lord by the state of Kerala. The capital of Mahe is
Ripon's involvement with the compromised Mahe.
state of the Ilbert Bill is also of importance. Kutch is the largest district in India by area.
Apart from his governance, Lord Ripon's It is located in the state of Gujarat. Kutch is a
notable contribution includes his renowned desert district and is known for its salt flats.
The capital of Kutch is Bhuj.
work "The Imperial Gazetteer of India," a
project he embarked upon in 1869. Leh is the second largest district in India by
area. It is located in the Union Territory of
Warren Hastings: Warren Hastings, the
Ladakh. Leh is a mountainous district and
inaugural Governor-General of Bengal, played is known for its Buddhist monasteries. The
a significant role in reshaping administrative capital of Leh is Leh.
structures. During the period of the Dual
System, the East India Company held Diwani 6. Option (1) is correct.
rights, encompassing revenue collection,
while administrative control rested with the Explanation: Rice: China is the world's largest
Nizam or Indian chiefs. Hastings took action producer of rice, followed by India and
to dismantle the Dual System, originally Indonesia.
established by Robert Clive. Notably, he Corn: The United States is the world's largest
reduced Nawab's annual allowance from producer of corn, followed by China and
Brazil.
Rs. 32 lakhs to Rs. 16 lakhs, bringing about a
financial adjustment in the region. Jute: India is the world's largest producer of
jute, followed by Bangladesh and China.
Lord Dufferin, who held the office of Viceroy
Wheat: China is the world's largest producer
of India from 1884 to 1888, was in power
of wheat, followed by India and the United
during the occurrence of the Third Anglo-
States.
Burmese War between 1885 and 1886. In a
parallel instance, the Second Anglo-Burmese 7. Option (4) is correct.
War transpired during the administration
Explanation: Fiscal deficit: The fiscal deficit
of Lord Dalhousie, who was in office from
is the difference between the government's
1848 to 1856. The outcome of the Second total expenditure and its total revenue,
Anglo-Burmese War was the British gaining excluding borrowings. It is a measure of the
control over Lower Burma. In contrast, the government's borrowing requirement for the
Third Anglo-Burmese War resulted in the year.
British capturing the entirety of Burma and Revenue deficit: The revenue deficit is the
subsequently integrating it as a province of difference between the government's total
India. The transformation of Burma into a revenue and its total revenue expenditure. It
distinct crown colony took place in the year is a measure of the government's inability to
1935. meet its revenue expenditure from its own
revenue sources.
5. Option (3) is correct. Primary deficit: The primary deficit is the
difference between the fiscal deficit and
Explanation: Rajasthan is the largest Indian the interest payments. It is a measure of the
state in terms of area. It is located in the government's borrowing requirement for the
northwestern part of India and borders year, excluding interest payments.
Pakistan to the west. Rajasthan is known for
its desert landscapes, forts, and palaces. The 8. Option (3) is correct.
capital of Rajasthan is Jaipur.
Madhya Pradesh is the second-largest Indian Explanation: Microphone is an input device
state in terms of area. It is located in the used to input sound that is then stored
central part of India and borders Rajasthan in a digital form. Whereas, a touch pad
to the northwest, Uttar Pradesh to the north, is a locator device for controlling input
Chhattisgarh to the east, Maharashtra to the positioning on a computer display screen.
south, and Gujarat to the southwest. Madhya It is used as an alternative to the mouse.
Pradesh is known for its forests, wildlife, and Monitors, commonly called as VDU (also
temples. The capital of Madhya Pradesh is known a Visual Display Unit), are the main
Bhopal. output device of a computer.
MOCK TEST Paper-4 91
9. Option (2) is correct. I Am and I Will: We can all make a difference.
Explanation: Trochodendron is a genus We can be advocates for cancer care, and we
of flowering plant with one living species can take steps to reduce our own risk of cancer.
fewer than 200 species of angiosperms Not Beyond Us: Closing the care gap is not
have a primitive feature of vesselless wood beyond us. It is a challenge, but it is one that
we can overcome if we work together.
with two exceptions - Trochodendron and
tetracentron all vesselless angiosperms occurs 13. Option (2) is correct.
in magnolidae.
Explanation: The Laser (light amplification by
10. Option (3) is correct. stimulated emission of radiation) was invented
by T.H. Maiman (Theodore Harold Maiman) in
Explanation: A thump of either a single hind
1960 at Hughes Research Laboratories. Maiman’s
leg or both together shows the rabbit’s fear
functional laser used a solid-state flashlamp-
or displeasure over something. In any case,
pumped synthetic ruby crystal to produce red
it shows the identification of some danger in
laser light, at 694 nanometers wavelength.
the vicinity by the rabbit. The rabbit feels the
danger and warns the rest of his warren. 14. Option (1) is correct.
11. Option (4) is correct. Explanation: A coup is a sudden overthrow
of a government illegally. A strike is a work
Explanation: The Bharat Ratna is the highest
stoppage caused by a huge number of
civilian award in India. It was instituted on
employees refusing to work. Coup or an
January 2, 1954, by the President of India,
overthrow, is the illegal and overt seizure of a
Rajendra Prasad.
state by the military or other elites within the
The first recipients of the Bharat Ratna were state apparatus.
C. Rajagopalachari, Sarvepalli Radhakrishnan,
and CV Raman. They were awarded the 15. Option (4) is correct.
award in 1954. Explanation: According to Article 361 of the
Khan Abdul Ghaffar Khan and Nelson Constitution of India, a criminal proceeding
Mandela were the first two non-Indians to be cannot be instituted in a court against the
awarded the Bharat Ratna. They were awarded Governor during his term of office. The
the award in 1987 and 1990 respectively. President or Governor, while in the office is
Mother Teresa was a naturalized Indian not personally answerable to any court for the
citizen. She was awarded the Bharat Ratna in exercise and execution of the powers.
1980. 16. Option (4) is correct.
12. Option (2) is correct. Explanation: Acetic acid is a significant
Explanation: The theme of World Cancer Day component of vinegar. Its concentration
2023 is "Close the Care Gap". This theme was specifies the strength of vinegar. It is an
chosen to highlight the inequities in cancer uncolored liquid with a solid eroding pungent
care that exist around the world. People in low- aroma. As vinegar and an acidity regulator,
and middle-income countries are more likely acetic acid is widely utilized in the food
to die from cancer than people in high-income industry.
countries. This is due to a number of factors, 17. Option (4) is correct.
including lack of access to early detection and
treatment, as well as poor quality of care. Explanation: Methane is the hydrocarbon
having the lowest molecular mass with the
The campaign "Close the Care Gap" aims to
chemical formula CH4. It is called Marsh gas
raise awareness of these inequities and to call
because it is released in vast amounts from
for action to address them. The campaign has
marshes where anaerobic bacteria decompose
four key messages:
matter. Alessandro Volta discovered it in
We can, I can: We all have a role to play in
1776. It is the main constituent of biogas and
closing the care gap. We can raise awareness, Compressed Natural Gas (CNG).
donate to cancer charities, and support cancer
research. 18. Option (2) is correct.
Close the care gap: We need to work together Explanation: Aga Khan Cup is a hockey
to close the care gap. This means governments, tournament held in Mumbai. It is one of the
healthcare providers, and individuals all oldest hockey tournaments in India, having
working together to improve access to cancer been started in 1896. The cup is named after
care. the Aga Khan, the spiritual leader of the Ismaili
Muslims.
92 OSWAAL CUET (UG ) 10 Mock Test Papers GENERAL TEST

Begum Hazrat Mahal Cup is a football 21. Option (2) is correct.


tournament held in India. It is named after Explanation:
Begum Hazrat Mahal, the queen of Awadh who 25 25
fought against the British in the Indian Rebellion Given:  % of 1600  % of 800
4 2
of 1857.
1600  25 800  25
Ramanuja Trophy is a table tennis tournament  
400 200
held in India. It is named after Ramanuja, an
= 100 + 100 = 200
11th-century Hindu philosopher and theologian.
Walker Cup is a golf tournament held between 22. Option (4) is correct.
the United States and Great Britain and Ireland. It
Explanation: Given: S.P. = `1000, Loss = 10%
is the oldest international team golf competition
in the world, having been started in 1922. 100
∴ Cost price of sewing machine  1080 
90
19. Option (3) is correct. = ` 1200
Explanation: Sriharikota is a bare island off the 1200 × 110
So, S.P. for a profit of 10% =
coast of Andhra Pradesh in Southern India. It 100
= ` 1320
has India’s only satellite launch center at the
Satish Dhawan Space Center (SHAR) and is used 23. Option (4) is correct.
by the ISRO to launch satellites with multistage
rockets such as polar satellite launchers and Explanation: Let the length of flag = l
geostationary satellite launchers. Originally In the triangle DBC,
called the Sriharikota High Range and later 10
tan 30° =
the Sriharikota Launch Range. The center was BC
renamed in 2002 following the death of former 1 10
⇒ =
ISRO President Satish Dhawan. 3 BC
20. Option (2) is correct. ⇒ BC = 10 3

Explanation: Flamingo Festival is celebrated Now in DABC,


in Andhra Pradesh, near Pulicat Lake. It is a 10 + l
tan 45° =
three-day festival that is organized during the BC
winter season when thousands of migratory ⇒ 10 3 = 10 + l
flamingos visit the region. The festival includes
⇒ l = 10 (1.732 − 1)
cultural events, boat rides, and bird watching.
= 7.32 m
Pongal is a harvest festival celebrated in Tamil
Nadu, Karnataka, and Andhra Pradesh. It is a 24. Option (3) is correct.
four-day festival that is celebrated in January
or February. The first day of Pongal is called Explanation:
Bhogi Pongal, when people clean their homes Given numbers = 11, 15, 13, 120, 10, 8, 11, 7,
15, 11, 13, 7, 11
and burn old clothes and materials. The second
Ascending order of given numbers = 7, 7, 8,
day is called Surya Pongal, when people offer
10, 11, 11, 11, 11, 12, 13, 13, 15, 15
prayers to the sun god. The third day is called
Mean of numbers = Sum of 13 Numbers =
Mattu Pongal, when people worship cattle. 144
The fourth day is called Kanum Pongal, when Median of numbers = 11 (Mid number of given
people visit their relatives and friends. data)
Kambala Festival is celebrated in Karnataka, Mode of numbers = 11 (Most frequent number
especially in the Dakshina Kannada district. It in given data)
is a buffalo race that is held during the winter So, mean > mode = Median
season. The race is held in a muddy field, and
the buffaloes are decorated with colourful
25. Option (1) is correct.
clothes and bells. Explanation: Single equivalent discount
Theyyam Festival is celebrated in Kerala. It is a  10  5 
ritualistic dance that is performed by Theyyam   10  5   %  14.5%
 100 
artists. The dancers wear elaborate costumes
and masks, and they represent different gods ∴ Amount to be paid = (100 – 14.5)% of 110
and spirits. The Theyyam Festival is held in 110 × 85.5
= = ` 94.05 = ` 94
temples and public places. 100
MOCK TEST Paper-4 93
26. Option (1) is correct. 31 1
or  or 3x = 4 × 9
Explanation: Given: S.I. = `60, Time = 5 years 3x 9
S.I. × 100 49
Principal = or x =  12 days
Time × Rate 3
60  100 6000 30. Option (2) is correct.
=   ` 200
56 30
Explanation: We know that,
27. Option (4) is correct. M1D1 = M2D2
Explanation: Let speed of motorboat in still or 24 × 12 = 36 × D2
water be x kmph
24  12
Speed of stream = 3 km/h or D2 =  8 days
∴ Downstream rate = (x + 3) kmph 36
Upstream rate = (x – 3) kmph 31. Option (3) is correct.
According to question, we have
12 12 60 Explanation:
  x 3 5
x  3 x  3 60
1 1  On squaring both sides, we get
or 12   1
 x  3 x  3  x  3  5  2 15
or x  8   2 15
or x  3  x  3  1
( x  3)( x  3) 12 On squaring again both sides, we get
x2 – 16x + 64 = 60
or x2 – 9 = 24x
or x2 – 16x + 4 = 0
or x2 – 24x – 9 = 0
∴ x2 – 16x + 6 = 2
( 24 )  ( 24 )2  4  ( 9 )
or x = 32. Option (2) is correct.
2
by quadratic formula Explanation:
24  576  36 24  612 66049  257  257 = 257
= 
2 2 So, the unit place in the square root of 66049
24  36  17 24  6 17 is 7
= 
2 2 33. Option (2) is correct.
Q x cannot be negative
Explanation:
6( 4  7 ) Given that, ∆ABC is an isosceles triangle.
∴x=  3( 4  17 ) km/h
2 AB = BC
28. Option (4) is correct. Point D is the mid point of side BC and
PD is internal bisector of ∠ADB.
Explanation: HCF of 391 and 323. According to the question,
A
323 391 1
323
68 323 4 P Q
272
51 68 1 B D C
51 ∠ADB = ∠ADC = 90°
17 51 3 and ∠PDA = 45°
51 Q PQ || BC
x
∴ ∠ADQ = 45°
∴ Number of classes = 17 ∴ PDQ = 45° + 45° = 90°
29. Option (4) is correct 34. Option (3) is correct.
Explanation: Let A complete work in x days
Explanation: Students who score 30 or more
∴ time taken by B to complete the work than 30 marks but less than 40 marks =
= 3x days 44 – 25 = 19.
According to question, we have Students who score 20 or more but less than
1 1 1 30 marks = 61 – 44 = 17.
  ∴ Required answer = 19 + 17 = 36.
x 3x 9
94 OSWAAL CUET (UG ) 10 Mock Test Papers GENERAL TEST

35. Option (3) is correct. 39. Option (3) is correct.


Explanation: (x + y)2 = 4xy Explanation: Let the 2principal be ` P
 R 2 
or x2 + y2 + 2xy – 4xy = 0 C.I. P  1  R   1
C.I. P  1  100   1
or (x - y)2 = 0 or x = y  100  
or x : y = 1 : 1  5 2 
2

328  P  1  5   1
328  P  1  100   1
 100  
36. Option (3) is correct.
 21  2 
2

 328  P  21   1
Explanation: Given that,  328  P  20   1
 20  
Side of equilateral triangle = 12 cm  
 441 
Now, we know that,  328  P  441  1 
 328  P  400  1 
3  400 
Height of equilateral triangle = × side
2  441  400 
 328  P  441  400 
3  328  P  400 
= × 12  400 
2
41P
= 6 3 cm  328  41P
 328  400
400
328  400
 P  328  400
37. Option (1) is correct.  P 41
41
Explanation: Given: Average of L, M and N = 131200
 P  131200
93 kg  P  41
41
P = ` 3200
L + M + N = 3 × 93 = 279 kg ….(i)
L+ M 40. Option (2) is correct.
= 89
2 Explanation: In 100 kg of alloy the quantity of
⇒ L + M = 89 × 2 = 178 kg ….(ii) nickel
2
M+N =  100  20
and = 96.5 10
2 Suppose x kg of nickel be mixed.
⇒ M + N = 96.5 × 2 = 193 kg ….(iii) 20 + x 3
So, = =
Adding eqn (ii) and (iii), 100 + x 11
L + M + N + M = 371 or 220 + 11x = 300 + 3x
or 11x – 3x = 300 – 220
⇒ M = 371 – (L + M + N)
80
⇒ M = 371 – 279 or 8x = 80 or x = = 10 kg
8
⇒ M = 92 kg
41. Option (1) is correct.
38. Option (1) is correct. Explanation:
Explanation: Let the distance between A and Correct relation between the given classes,
B = x km Literates

Then speed of train


Hard-
Graduates working
x 4x
  km / h
45 3
x 4x
 60  km / h
45 3
x
When speed is48reduced
4 by 5 km/h
then 60   42. Option (4) is correct.
4x 60 5
x 5 48 4 Explanation: According to the statements,
then 3  
4x
3x  5 4 60 5 Glasses Utensils
 3 
4 x  15 5
3x 4 Bowls Cups Plates
 
4 x  15 5
⇒ 16x – 60 = 15x I. () II. ()
⇒ x = 60 km Hence, neither conclusion I nor II follows.
MOCK TEST Paper-4 95
43. Option (1) is correct. 49. Option (4) is correct.
Explanation: In the mirror image left becomes Explanation:
right and right becomes left. According to the question,

44. Option (2) is correct. who are you 4 3 2 (i)


Explanation: As ‘Turmeric’ is ‘Spices’, same as 4
they is you 8 5 (ii)
‘Dates’ are ‘Dry Fruits’.
they are dangerous 2 9 5 (iii)
45. Option (2) is correct.
From equation (i), (ii), and (iii), dangerous = 9.
Explanation:
As, R A J Positional
  18 1 10
50. Option (1) is correct.
Value
Explanation:
Now, (18 + 1 + 10 = 29) × 3
O R U X A D
(total alphabets in the word)
= 87 +3 +3 +3 +3 +3
Positional
and G I T A 
Value
 7 9 20 1 51. Option (4) is correct.
Now, (7 + 9 + 20 + 1 = 37) × 4 = 148 Explanation: Assertion: Leakages in household
Similarly, gas cylinders can be detected. It is true.
Positional Household gas cylinders have LPG in them
V A R U N 
Value
 22 1 18 21 14
but it is odorless.
Now, (22 + 1 + 18 + 21 + 14 = 76) × 5 = 380 For leak detection in household cylinders,
ethyl mercaptan is added to it as it has a smell
46. Option (2) is correct. of rotten cabbages which help in the detection
of leaks. Reason: LPG has a strong smell. It is
Explanation: false as it is odorless.
According to the information,
52. Option (2) is correct.
Explanation:
Given :

So, maximum cars in the row = 6 + 1 + 9 + 1 +


8 + 1 = 26 53. Option (4) is correct.
Explanation:
47. Option (2) is correct.
4
Explanation:
Devansh Dileep Maaya
Father Son
Sister
Virat Preeti Manoj
∴ Maaya is the mother-in-law of Preeti. Hence, required distance dotted line XY, i.e
(17 + 7) = 24m, and Y is in east direction with
respect to X.
48. Option (3) is correct.
54. Option (1) is correct.
Explanation:
Explanation:

T R P N L J H

–2 –2 –2 –2 –2 –2
96 OSWAAL CUET (UG ) 10 Mock Test Papers GENERAL TEST

55. Option (1) is correct. 58. Option (4) is correct.


Explanation: So, Explanation:
2 8 32 128 512 2048

×4 ×4 ×4 ×4 ×4

59. Option (3) is correct.


Explanation:
Staring point N
E 10 km B
will complete the figure. A
3 km W E
3 km
D C S
6 km
56. Option (3) is correct.
Given, DC = BE = 6 km
Explanation: Both the conclusions have ∴ Required distance AE = AB – BE
information which does not follow from given
= 10 – 6 = 4 km
statements.
Hence, he is at E, 4 km to the East from starting
Hence, neither I nor II follows. point A.

57. Option (3) is correct. 60. Option (1) is correct.


Explanation: Only grandson of Anup’s mother
Explanation: ‘Roar’ is the sound made by lion. means the son of Anup.
Similarly, Mother of Anup’s son means the wife of Anup.
‘Trumpet’ is the sound made by elephant. Only daughter of Anup’s wife is also the
daughter of Anup.


SOLUTIONS OF
Question Paper 5
1. Option (4) is correct. Protons were discovered by Ernest Rutherford
Explanation: Bimbisara (544 - 492 BC) - He in 1917. Rutherford was a New Zealand-born
founded the Haryanka dynasty. He's also physicist who was studying the scattering
known as Shrenika. He lived during the time of alpha particles by gold foil. He found that
of Lord Buddha. Bimbisara had three wives: a small fraction of the alpha particles were
Kosaladevi, Chellana, and Khema. When deflected through large angles, which he
the ruler of Avanti, Pradyota, suffered from could only explain if the atom had a small,
Jaundice, he sent the royal physician Jivaka to dense nucleus that was positively charged.
help. Rutherford called the particles in the nucleus
Ajatashatru (492 - 460 BC) - He seized the "protons."
throne by killing his father. Ajatashatru had
3. Option (1) is correct.
other names: Kunika and Ashokachanda.
He defeated his maternal uncle, Prasanjit, Explanation: Foehn: The Foehn is a warm,
and married Prasanjit's daughter, Vajira. dry wind that occurs in the Alps. It is caused
The famous weapons Mahasilakantaka and by the adiabatic heating of air as it descends
Rathamusala were associated with him. from the mountains. The Foehn is known for
He organized the first Buddhist council in its warm, dry weather and its ability to melt
Rajgriha after Buddha's death. snow quickly.
Udayin (460 - 444 BC) - Udayin was the son Sirocco: The Sirocco is a hot, dry wind
of Ajatashatru. He designated Pataliputra as that blows from the Sahara Desert to the
the capital of Magadha and built a fort at the Mediterranean Sea. It is known for its dusty,
confluence of the Ganges and Son rivers in hot weather and its ability to cause sandstorms.
Patna.
Chinook: The Chinook is a warm, dry wind
Kalashoka (394 - 366 BC) - Sishunaga Dynasty that occurs in the Rocky Mountains. It is
Kalashoka was also known as Kakavarna. The
caused by the adiabatic heating of air as it
second Buddhist council was held in Vaishali
descends from the mountains. The Chinook
during his reign.
is known for its warm, dry weather and its
ability to melt snow quickly.
2. Option (2) is correct. Mistral: The Mistral is a cold, dry wind that
Explanation: X-rays were discovered by blows from the north along the western coast
Wilhelm Conrad Roentgen in 1895. Roentgen of France. It is known for its strong winds and
was a German physicist who was studying its ability to cause damage.
the properties of cathode rays (streams of 4. Option (1) is correct.
electrons) when he noticed that a fluorescent
screen nearby was glowing even though it was Explanation: Delhi and Noida share the status
not connected to his electrical equipment. He of twin cities.
realized that he had discovered a new kind of Kolkata and Howrah are recognized as twin
radiation that could pass through solid objects. cities. Kolkata is situated on the western bank
Roentgen named his discovery "X-rays" of the Hooghly River.
because he did not know what they were. Durgapur and Asansol are often referred to as
Electrons were discovered by JJ Thomson in twin cities.
1897. Thomson was an English physicist who Hyderabad and Secundrabad bear the
was studying cathode rays when he found distinction of being twin cities.
that they were negatively charged particles.
The capital of Telangana is Hyderabad.
He called these particles "corpuscles," but
Cuttack and Bhubaneswar in Odisha are also
they are now known as electrons. Thomson's
designated as twin cities.
discovery of the electron was a breakthrough
in physics, and it helped to lay the foundation Pune and Pimpri-Chinchwad in Maharashtra
for our understanding of atomic structure. are commonly referred to as twin cities.
98 OSWAAL CUET (UG ) 10 Mock Test Papers GENERAL TEST

Hubli and Dharwad in Karnataka share the 8. Option (4) is correct.


status of twin cities. Explanation: National Electronic Funds
Durg and Bhilai in Chattisgarh are also Transfer (NEFT) is a nationwide centralized
acknowledged as twin cities. payment system owned and operated by the
5. Option (1) is correct. Reserve Bank of India (RBI). It allows you
to transfer funds from one bank account to
Explanation: The “FOCAL LENGTH” improves another within India.
when the human eye lens evolves light. When
IFSC or Indian Financial System Code is an
the eye muscles are comforted, the lens-thin
11 digits alpha-numeric code that uniquely
and the focal size grow to make feasible the
distant objects to see. When we look at objects identifies a bank branch participating in the
nearer to the eye, the ciliary muscles contract NEFT system. It is used to facilitate electronic
and the lens becomes thick. fund transfers between banks.
MICR Code or Magnetic ink character
6. Option (4) is correct. recognition(MICR) is a 9 digits code printed
on the cheque which enables identification
Explanation: The 42nd Amendment to
the Indian Constitution, enacted in 1976, of the cheques and identifies the bank and
established Administrative Tribunals in branch participating in an Electronic Cheque
India. The original constitution did not Clearing system. It is used to process cheques
include provisions regarding tribunals, but electronically.
the 42nd Amendment introduced a new Swift Code (Society for Worldwide
part, XIV-A, and Article 323A. Article 323A Interbank Financial Telecommunication) is
grants the Parliament the authority to create an internationally recognized system for
administrative tribunals. These tribunals identifying banks and financial institutions
handle disputes and complaints related to globally. It is used to facilitate cross-border
the recruitment and terms of employment payments.
for individuals in public service positions
associated with the Union and the States. 9. Option (3) is correct.
The 24th Amendment to the Indian
Constitution, which followed, mandated Explanation: Read Only Memory (ROM) is
that the president must provide assent to a read only, it cannot be changed, it is permanent
constitutional amendment bill. Subsequently, and non-volatile, meaning it also holds its
the 44th Amendment removed the right to memory even when power is removed. It is
property from the list of fundamental rights. a type of medium for storage that stores data
The First Amendment, passed in 1951 by the permanently on personal computers (PCs)
Provisional Parliament, occurred right after the and other electronic devices.
Constitutional Assembly completed drafting
the Constitution. This amendment modified
10. Option (2) is correct.
articles 15, 19, 85, 87, 174, 176, 341, 342, 372, Explanation: Full-duplex data transmission is
and 376. It allowed for the preservation of used to transmit data in both directions on a
laws related to estate acquisition, introduced signal carrier at the same time. For example, on
the Ninth Schedule to shield land reforms a local area network with a technology that has
and included rules from judicial review, and full-duplex transmission one workstation can
inserted Articles 31A and 31B after Article 31. be receiving data while another workstation is
7. Option (3) is correct. sending data on the line.

Explanation: “A record 13.5 crore people 11. Option (3) is correct.


moved out of multidimensional poverty
between 2015-16 and 2019-21 as per NITI Explanation: Cloves are the aromatic dried
Aayog’s Report ‘National Multidimensional flower buds of a tree in the family Myrtaceae,
Poverty Index: A Progress Review 2023’” is Syzygium Aromaticum. They are native to
correct. The report found that the percentage the Maluku Islands in Indonesia and are
of people living in multidimensional poverty commonly used as a spice. The flower buds are
in India declined from 24.85% in 2015-16 to at first of a pale color and gradually become
14.96% in 2019-21. This means that a total of 13.5 green, after which they develop into a bright
crore people moved out of multidimensional red, when they develop into a bright red they
poverty during this period. are harvested by farmer.
MOCK TEST Paper-5 99
12. Option (2) is correct. World Health Organisation (WHO): The
Explanation: The People's Republic of China, WHO headquarters is located at Avenue
situated in East Asia and bordering India and Appia 20 in Geneva, Switzerland. The WHO
was established in 1948 to promote health
other countries, holds the title of the world's
and well-being for all people. The WHO is
most populous nation and ranks as the fourth-
the leading international authority on public
largest in terms of land area. Its head of state is
health.
the President. China lies to the north of India,
International Labour Organisation (ILO): The
and Indian states that share borders with
ILO headquarters is located in International
China include Ladakh, Himachal Pradesh, Labour Office in Route des Morillons 1 in
Sikkim, Arunachal Pradesh, and Uttarakhand. Geneva, Switzerland. The ILO was established
Afghanistan, located in Central Asia, is in 1919 to promote social justice and decent
officially known as the Islamic Republic of work for all. The ILO is the only tripartite
Afghanistan. It spans a total land area of United Nations agency, meaning that it
652,230 square kilometres and is characterized brings together governments, employers, and
by its predominantly Islamic population. The workers to make decisions.
country's economy relies significantly on Organisation for the Prohibition of Chemical
agriculture. In nearby countries like India, Weapons (OPCW): The OPCW headquarters
Afghanistan engages in industrial activities is located in The Hague, Netherlands. The
such as the production of cotton textiles, OPCW was established in 1997 to prohibit the
woollens, woven carpets, and handicrafts. development, production, stockpiling, and
Afghanistan shares a border with Pakistan- use of chemical weapons. The OPCW is the
occupied Kashmir. implementing body of the Chemical Weapons
Convention, which is an international treaty
Bhutan, situated to the north of India and
that was adopted in 1993.
officially recognized as The Kingdom of
Bhutan, shares borders with the Indian states 14. Option (4) is correct.
of Arunachal Pradesh, Sikkim, Assam, and
Explanation: The Arjuna Award is awarded by
West Bengal. It is a landlocked country with the Ministry of Youth Sports of the government
an area of 38,394 square kilometres and its of India in recognition of the excellence of
capital city is Thimphu. Bhutan is governed national sports. The award was established in
by a constitutional monarchy and its 1961 and is presented with a statue and a scroll
economy is primarily driven by hydropower, of Arjuna for 500,000 rupees.
agriculture, forestry, and tourism. Among
India's neighbouring countries, Bhutan is the 15. Option (3) is correct.
smallest. Explanation: ‘Agha Khan Cup’ is associated
Bangladesh, officially named the People's with hockey tournament. It is a very famous
Republic of Bangladesh, is positioned in tournament played in 1896. It is one of the
South Asia and shares one of the world's most oldest hockey tournaments of India.
longest borders with India. The capital city
16. Option (3) is correct.
is Dhaka, and Indian states such as Assam,
Mizoram, Tripura, Meghalaya, and West Explanation: Radar (radio detection and
Bengal share their borders with Bangladesh. ranging) is an object detection system that uses
With a population of 161 million, Bangladesh radio waves to determine the distance, height,
ranks as the eighth-most populous country speed or direction of an object. It can be used
globally and is known for its high population to identify aircraft, spacecraft, ships guided
density. Bangladesh was initially part of missiles and engines, vehicles, metrological
Pakistan, known as East Pakistan, but gained structures and terrain.
independence after the 1971 Liberation War. 17. Option (1) is correct.
13. Option (4) is correct. Explanation: Malic acid is seen in apples. It is
a dicarboxylic acid that exists in grapes, wines,
Explanation: United Nations Conference on
and rhubarb. It imparts a tart flavor to a part of
Trade and Development (UNCTAD): The the meals. Formic acid is seen in ants.
UNCTAD headquarters is located in the Palais
des Nations in Geneva, Switzerland. The 18. Option (4) is correct.
Palais des Nations is a complex of buildings Explanation: Ethyl acetate odor. This uncolored
that was initially built as the headquarters sweet aroma is utilized in decaffeinating tea
of the League of Nations. The UNCTAD was and coffee. Acetate has a vigorous liquid that
established in 1964 to promote international has distinctive glue also used as nail polish
trade and development. removers.
100 OSWAAL CUET (UG ) 10 Mock Test Papers GENERAL TEST

19. Option (2) is correct. 24. Option (1) is correct.


Explanation: ‘World Ozone Day’ is celebrated Explanation:
on 16 September every year since 1995. This 10
day reminds the signing of the Montreal The length of the shadow of tree is m.
3
Protocol on substances that depletes the Height of tree is 30 m.
Ozone layer. The theme of World Ozone day A
2021 was Montreal Protocol – keeping us, our
food and vaccines cool.
20. Option (1) is correct.
Explanation: Bhavai is a folk dance of Gujarat 30
and Rajasthan also. The dancers perform
on pieces of glass and sword in the dance.
First Bhavai dancer was Mrs. Krishna Vyas
θ
Chhangani, who born in Jodhpur.
B C
21. Option (3) is correct. 10 3

Let us assume that AB be the height of the tree


Explanation: Given S.P. of 20 apples = `100
and BC be the length of the shadow.
Gain = 20%
Let the angle of elevation of the sun be θ.
C.P. of 20 apples,  120c.p.   100 In the given triangle,
 100 
 
1000 AB
C.P. = tan θ =
12 BC
No. of apples that can be bought in `100 30
⇒ tan θ =
20 10
= 1000 × 100 3
12 3
⇒ tan θ =
= 24. 3
22. Option (3) is correct. ⇒ tan θ = 3
Explanation: Let the roots of quadratic ⇒ tan θ = tan 60°
equation are a and b. So, θ = 60°
Given that: The angle (in degrees) of elevation of the sun is
Arithmetic mean of quadratic equation = 8 60°.
α+β 25. Option (1) is correct.
∴ =8
2
or α+β =16 Explanation: According to the question,
Geometric mean of quadratic equation = 5 A’s 4 days work = B’s 5 days work
A:B = 4:5
αβ = 5
and B : C = 6 : 7
αβ = 25
Now A : B : C = (4 × 6) : (5 × 6) : (5 × 7)
Using, quadratic equation
= 24 : 30 : 35
⇒ x2 – (a + b) x + ab = 0
Time taken by A = 7 days
⇒ x2 – 16x + 25 = 0
35 245 5
So, option (3) is correct. ∴ Time taken by C = 7   10 days
24 24 24
23. Option (1) is correct.
26. Option (2) is correct.
Explanation: Suppose the numbers be x, 2x
and 4x. Explanation: 67127y76x2 is divisible by 88.
As per question, So, the given number is divisible by 8 and 11,
x2 + (2x)2 + (4x)2 = 1029 As 632 is divisible by 8 so, x = 3
or x2 + 4x2 + 16x2 = 1029 (6 + 1 + 7 + 7 + 3) – (7 + 2 + y + 6 + 2) = 0
or 21x2 = 1029 ⇒ 24 – (17 + y) = 0
1029 ⇒ 24 – 17 – y = 0
or x2 = = 49 or x2 = 49 or x = 7
21 ⇒7–y=0
So, the required difference = 4x - x = 3x ⇒y=7
= 3 × 7 = 21 ∴ 7x – 2y = 7 × 3 – 2 × 7 = 21 – 14 = 7
MOCK TEST Paper-5 101
27. Option (4) is correct. 29. Option (3) is correct.
Explanation: According to Rule Explanation:
Let the speed of the second train be x m/sec Given that,
AD ⊥ BC,
Speed of first train A

150
= = 10 m / s
15
Relative speed of trains
= (x + 10) m/sec B D C

Total distance covered Suppose, AB = 2x units


= (150 + 150) = 300 metre or BD = DC = x units
So, time taken = 300 ∴ AB : BD
x + 10 =2:1
⇒ 300
 12
x  10 30. Option (2) is correct.
⇒ 12x + 120 = 300 Explanation: Let the number of trees in the garden
be x. Then,
⇒ 12x = 300 – 120 = 180
60
180 Coconut trees = x
x  15m / sec 100
12
15  18 25 60 15
 km / hr Mango trees =  x x
5 100 100 20
= 54 km/hr 20 15 3x
Apple trees =  x
100 100 100
28. Option (1) is correct.
According to the question,
Explanation: LCM of 15, 18, 20 and 27. Apple trees = 1500
2 15, 18, 20, 27 3x
⇒ = 1500
2 15, 9, 10, 27 100
3 15, 9, 5, 27 ⇒ x = 500 × 100
⇒ x = 50,000.
3 5, 3, 5, 9
3 5, 3, 5, 3 31. Option (3) is correct.
5 5, 1, 5, 1 Explanation:
1, 1, 1, 1 GDP in the beginning of 2015

∴ LCM of 15, 18, 20 and 27  104 


= 3 × trillion dollar = 3.12
= 2 × 2 × 3 × 3 × 3 × 5 = 540  100 
According to the question, = 3.12 trillion dollar
∴ x = 540k + 10,
Where k is a position integer 32. Option (1) is correct.
= 31 × 17k + 13k + 10 Explanation:
Here, 13k + 10 must be divisible by 31. x2 – 3x + 1 = 0
For k = 4, or x2 + 1 = 3x
13 × 4 + 10 = 62, which is divisible by 31 On dividing both sides by x, we get
∴ x = 540 × 4 + 10 = 2170 1
Now, x  3
x
1 1
4 2170 46 x 2  x   2
x x
4 16
 2 1   1
86 570 x  2 x  
 x   x
6 516 2
 1  1
x   2 x  
∴ 47 × 47 = 2209  x  x
So, required number = 2209 – 2170 = 39 = 9 – 2 + 3 = 10
102 OSWAAL CUET (UG ) 10 Mock Test Papers GENERAL TEST

33. Option (2) is correct. Now, we know that,


Explanation: In the original mixture, Area of triangle
Radius of Incircle =
1 Semi perimeter
Water = × 240 = 60 cc 15
4 or 3 =
3  S
Glycerine =   240   180 cc 15
4  or S = =5
3
Suppose x cc of water be mixed
∴ Perimeter = 2S
60 + x 2
So, = = 2 × 5 = 10 cm
180 3
or 180 + 3x = 360 or 3x = 360 – 180 = 180
38. Option (3) is correct.

or x =
180
= 60cc Explanation: Given:
1
=
( 2 −1 )
3 2 +1 ( 2 +1 )( 2 −1)
34. Option (3) is correct. Rationalising the denominator, we get,
Explanation: 2 −1
= = 2 −1
( 2)
2
Sum of 0, 1, 2, 3, ... 110 −1
1
110(110 + 1) Also, = 3− 2
= 2+ 3
2
= 6105 1
= 4− 3
 n( n  1)  4+ 3
Q1  2  3  ...  n  2  1
  and = 9− 8
∴ Unit digit = 5 8+ 9
1 1 1 1
35. Option (2) is correct.    ..........
1 2 2 3 3 4 8 9
Explanation: According to the questions,  2  1  3  2  4  3.......... 5
X + Y + Z = 3 × 74 = 222 kg ….(i)
 4 6 5 7 6 8 7 9 8
X+Y = 2 × 68 = 136 kg ….(ii)
 9 1  3 1  2
Y+Z = 2 × 78 =156 kg …. (iii)
Adding eqn (ii) and (iii), 39. Option (2) is correct.
X + Y + Z + Y = 292 Explanation: Single equivalent discount
⇒ Y = 292 – 222  xy 
⇒ Y = 70 kg x  y  %
 100 
36. Option (2) is correct.  30  20 
  30  20   %   50  6  %  44%
Explanation:  100 
Length of wire = circumference of circle
40. Option (2) is correct.
= 2πr
22 44 Explanation:
=2× × 84 = × 84
7 7
A P  1  rrr11   1  rrr22 
= 528 cm A
A PP  11  1 1
 100   1  100 2 

 100 100   100 100 
∴ Perimeter of square = 528 cm
10  
 1  10 12 
12
10000 10   11  12 
528 10000  11 
10000  100
100   1 
 100
100 
So, length of side =  100   100 
4
110 112 112
 10000
 10000   110  112
110 
= 132 cm  10000  100
100  100
100 100 100
37. Option (4) is correct. 11 2828
 10000
 10000   11  28
11 
 10000  10
10  25
Explanation: 10 2525
3080000
Given that  3080000
 3080000
 250
250
Area of triangle = 15 sq. cm 250
Incircle radius = 3 cm = ` 12,320
MOCK TEST Paper-5 103
41. Option (1) is correct. 47. Option (4) is correct.
Explanation: Only daughter of Nitin's mother- Explanation: (a) Amphibian ….(4)
in-law is Nitin's wife. Son of Sanchi's mother's (b) Amorphous ….(3)
father is Nitin. Therefore, Nitin is brother of (c) Amphidextrous ….(5)
Sanchi's mother. (d) Ambiguous ….(1)
42. Option (2) is correct. (e) Ambivalent ….(2)
Hence, (d), (e), (b), (a), (c) is the correct answer.
Explanation: Correct relation between the
given classes
48. Option (2) is correct.
Doctors
Explanation:
As,
11 165 209
Salaried ×15
Parents
Persons
×19
Same as,
43. Option (1) is correct. 12 180 228
Explanation: ×15
×19
According to the statements,

Chains Rings
49. Option (1) is correct.
Lockets
Explanation:
Medals Players
Doctors
A
B C D
I. () II. ()

Hence, only conclusion II follows.


Singers
44. Option (3) is correct.
50. Option (4) is correct.
Explanation:
Explanation:
F

A D

will complete the given figure.


B C
45. Option (3) is correct.
Explanation: As ‘Jackal’ sound is called ‘Howl’, E
in the same way the ‘Rain’ sound is called Fix the position of E and F first. Here, clearly
‘Patter’. from the figure, F and B are the neighbours
46. Option (2) is correct. of A.

Explanation: 51. Option (2) is correct.


Given
Sunil Sunil Explanation:
20 28 S 7 km N
R NW
NE
2 km
Deepak 36 W E
Deepak SW
T 4 km U SE
Hence, total boys in the row = position from S
left + Position from right – 1 Clearly, he is at point U in South-West direction
= 28 + 36 – 1 = 63 from R.
104 OSWAAL CUET (UG ) 10 Mock Test Papers GENERAL TEST

52. Option (4) is correct. 57. Option (2) is correct.


Explanation: Explanation:
1 Given,
2 Maruti Honda
2 20 30
4 ⇒ 1 3
3
4
Middle Position
Hence, by joining all the parts we can see Hence, total cars in the row = 30 + 30 – 1 = 59

the correct figure. 58. Option (3) is correct.


53. Option (4) is correct. Explanation:
Eshwar
Explanation: Conclusion I have the facts that
can be proved right from statement I, but Father
conclusion II does not relate to any of the Arsh Bimla
statements.
Father son
Hence, only conclusion I follows. brother
Shivam Dhruv
54. Option (3) is correct.
Therefore, from the figure, it is clear that Bimla
Explanation: is the daughter-in-law of Eshwar.
Given : 525 Q 25 R 42 S 4 = 189
59. Option (2) is correct.
÷ + × Explanation:
∴ 525 ÷ 25 + 42 × 4 B M O
= 21 + 168 +3 +2 +2
= 189
E O Q

55. Option (1) is correct. +3 +2 +2

H Q S
Explanation: So,
+3 +2 +2

K S U
+3 +2 +2

N U W

60. Option (1) is correct.


will complete the figure. Explanation: Given,
56. Option (1) is correct. 1 2 3 4 5 6 7 8 9
Explanation: 〈 ∗ 〉 ∆ ◊

Teeth are used to cut.


Similarly, So, 〈 ◊=9 1 4 8 6
Legs are used to walk.

SOLUTIONS OF
Question Paper 6
1. Option (4) is correct. Altostratus clouds, a type of stratus cloud, are
Explanation: The Nanda dynasty was mid-level clouds that have a grey or blue-grey
founded by Mahapadma Nanda, who ruled appearance, often covering the entire sky.
from around 467 BC. The Nanda dynasty was Nimbus: Nimbus clouds are characterized
the first imperial dynasty of ancient India and by their black or dark grey colouration. They
played a significant role in shaping the political develop at middle altitudes or very close to the
landscape of the region during that time. Earth's surface. These clouds are highly dense
Dhanananda is traditionally considered to and opaque, sometimes appearing so low that
be the last ruler of the Nanda dynasty. He they appear to touch the ground. They lack
a distinct shape, existing as thick masses of
was overthrown by the famous Mauryan
vapour.
emperor Chandragupta Maurya, marking
the transition from the Nanda dynasty to the Cumulonimbus: Cumulonimbus clouds,
Maurya dynasty. commonly known as "thunderheads," are
noticeable on warm summer days and have
Alexander the Great did invade the Indian
the potential to bring about strong winds, hail,
subcontinent in 326 BC. His invasion reached
and rain. They often occur along cold fronts,
as far as the Punjab region in northwestern
where cool air is pushed beneath warmer air.
India. It's believed that the ruler of the Nanda
As evening approaches, cumulonimbus clouds
dynasty at that time was indeed Dhanananda,
typically diminish in size as the moisture in the
making this statement accurate. air evaporates. Gradually, they transform into
2. Option (1) is correct. stratocumulus clouds, which seldom produce
rain.
Explanation: Jallianwala Massacre: This tragic
event took place on April 13, 1919, in Amritsar, 4. Option (2) is correct.
Punjab, during the British colonial period. It Explanation: The Pakistan Resolution was
occurred before the other events listed. a landmark document in the history of the
Formation of the Swaraj Party: The Swaraj Indian independence movement. It called for
Party was formed in 1923. This party was a the creation of a separate Muslim-majority
political group in India that aimed to attain state in British India. The resolution was
self-governance (Swaraj) within the British passed by the All India Muslim League at its
Empire. 27th annual session held in Lahore on 22–24
Gandhi-Irwin Pact: Also known as the Delhi March 1940.
Pact, this agreement was signed on March 5, The resolution was moved by Mohammad
1931, between Mahatma Gandhi and the then Ali Jinnah, the leader of the All-India Muslim
Viceroy of India, Lord Irwin, during the Indian League. Jinnah argued that the Muslims of
independence movement. British India were a separate nation and that
Lucknow Pact: The Lucknow Pact was an they deserved their own state. He said that the
agreement between the Indian National Muslims of India could not live in peace and
harmony with the Hindus and that the only
Congress and the Muslim League, reached in
way to ensure their safety and security was to
1916. It aimed to present a united Indian front
create a separate Muslim state.
to the British government on constitutional
The Pakistan Resolution was met with mixed
reforms while also addressing some Muslim
reactions. The Hindus were opposed to the
demands.
resolution, and they argued that it would
3. Option (4) is correct. divide India and weaken it. The British were
also opposed to the resolution, and they did
Explanation: Stratus: These clouds, as
not want to see India divided. However, the
indicated by their name, form layers that cover
Muslims of India were overwhelmingly in
extensive portions of the sky. They typically
favour of the resolution, and they saw it as
result from either heat loss or the mixing of air their only hope for a secure and prosperous
masses with varying temperatures. future.
106 OSWAAL CUET (UG ) 10 Mock Test Papers GENERAL TEST

The Pakistan Resolution ultimately led to the The Nand Baba Milk Mission is a major
creation of the independent state of Pakistan in initiative of the Uttar Pradesh government
1947. The partition of India was a bloody and to boost the dairy sector in the state. The
chaotic process, but it ultimately fulfilled the mission is expected to benefit millions of
dream of the Muslims of India for a separate milk producers in Uttar Pradesh and help to
Muslim state. improve their livelihoods.
8. Option (2) is correct.
5. Option (3) is correct.
Explanation: The Maharashtra government
Explanation: The city of Madurai is located has recently introduced a new financial
in Tamil Nadu on the bank of river Vaigai. It initiative designed to offer assistance to
has found mentions in the text of Kautilya and farmers within the state. The program,
Megasthenes dating back to 3rd century B.C. named the "Namo Shetkari Mahasanman
The ancient city has served as the capital of the Yojana," received approval during a Cabinet
Kingdom of Pandyas. meeting presided over by Chief Minister
Eknath Shinde. Under this scheme, farmers
6. Option (2) is correct. in Maharashtra will receive an annual
payment of ` 6,000. This financial support is
Explanation: The Danube river in Europe in addition to the existing ` 6,000 that farmers
is the river that gives through the ultimate already receive annually in instalments from
number of countries in its pilgrimage. It passes the central government's “Pradhan Mantri
through several countries, Bulgaria, Croatia, Kisan Samman Nidhi Yojana.” The objective
Moldova, Ukraine, Slovakia. It begins in of this initiative is to provide an extra boost
Southern Germany and steers into the Black to farmers' income and enhance their overall
Sea via Romania. financial stability.

7. Option (1) is correct. 9. Option (2) is correct.


Explanation: The Nand Baba Milk Mission Explanation: The first ever motion to remove
a Supreme Court Justice was signed in the year
is a flagship scheme of the Uttar Pradesh
1991, by 108 members of the Parliament. It was
government launched in June 2023 with a
against Justice V. Ramaswami, who was accused
budget of `1,000 crore. The mission aims to of wrong doing related to his tenure as Chief
increase milk production in the state by 20% Justice of the Haryana and Punjab High Court.
in the next five years and provide a fair price
to milk producers.
10. Option (3) is correct.
The mission will focus on the following areas: Explanation: The first stable product of
• Improving animal husbandry: The photosynthesis is phosphoglyceric acid (a
mission will provide financial assistance to three-carbon compound). Almost immediately,
two of these join to form a glucose molecule.
farmers to improve their animal husbandry
Some herbaceous plant such as sugarcane,
practices. This will include providing
corn and sorghum, form as the first products
training to farmers on modern animal
of photosynthesis not three-carbon but four-
husbandry techniques, providing subsidies carbon compounds – oxalacetic, malic and
for the purchase of high-quality breeds of aspartic acids.
cattle, and providing veterinary care to
cattle. 11. Option (2) is correct.
• Setting up dairy processing units: The Explanation: Trypsin is a protease enzyme
mission will set up dairy processing units in that breaks down proteins. It is most active in
all districts of Uttar Pradesh. This will help an alkaline environment, with an optimum
to increase the value addition of milk and pH of 7.8 to 8.0. Amylase is an enzyme that
provide employment opportunities to the breaks down carbohydrates. It is most active
youth. in an alkaline environment, with an optimum
pH of 7.0.
• Creating a market for milk: The mission
Therefore, trypsin and amylase are functional
will work to create a market for milk in
in the alkaline environment.
Uttar Pradesh. This will include promoting
The other enzyme pairs, trypsin and pepsin,
the sale of milk through dairy cooperative
and chymotrypsin and pepsin are both
societies and providing financial assistance
functional in an acidic environment. Pepsin
to milk producers to set up their own milk is a protease enzyme that is most active in
vending units.
MOCK TEST Paper-6 107

an acidic environment, with an optimum pH 16. Option (1) is correct.


of 1.5 to 2.5. Explanation: The capability of a visual mechanism
or style of flick to split or differentiate slight or
12. Option (4) is correct. closely bordering images of the human eye with
Explanation: As of August 12, 2023, 18 nations 20/20 eyesight is commonly assumed to be about
‘one arc minute’ or ‘60 arc seconds’.
have officially attained recognition from the
World Health Organization (WHO) that they 17. Option (4) is correct.
have successfully eliminated trachoma as a
Explanation: Ernest Rutherford’s interest was
public health concern. The 18th nation is Iraq. to understand how the electrons are positioned
This achievement also marks Iraq as the 50th within an atom. For this, Rutherford organized
country to eliminate at least one Neglected an investigation in which fast-moving alpha
Tropical Disease (NTD) on a global scale. (a)-particles were constructed to drop on the
gold foil.
13. Option (2) is correct.
18. Option (2) is correct.
Explanation: Aryabhatta, was the first
Explanation: Liquid hydrogen has been used
satellite launched by Indian Space Research as a fuel in rockets and space shuttles as it
Organization on 19 April, 1975. Later, on June 7, supplies a characteristic impulse almost 30%–
1979 an Earth observation satellite Bhaskara-I 40% higher than further rocket fuels. Liquid
was launched for India. India launched its oxygen and liquid hydrogen are used as the
own satellite for the first time on July 18, 1980. fuel in the high-efficiency main motors of the
space shuttle.
It was the Rohini-1 Satellite Launch Vehicle
(SLV) rocket from the Sriharikota island 19. Option (4) is correct.
launch site. Explanation: 2 February – World Wetlands
Day: This day is dedicated to raising awareness
14. Option (1) is correct.
about the importance of wetlands for the
Explanation: Pakhawaj is not a vocal form of environment and human well-being. Wetlands
Hindustani Classical music. It is a two-headed play a crucial role in water purification, flood
drum which is barrel shaped, also known as control, carbon storage, and providing habitat
mridang. Tarana, Draupad, Khyal and Dhamer for various species.
are major vocal forms of Hindustani Classical 22 March – World Water Day: World Water Day
music. Hindustani Classical music originated focuses on the importance of freshwater and
in 12th century C.E. advocating for the sustainable management of
water resources. It aims to address issues such
15. Option (4) is correct. as water scarcity, sanitation, and water-related
challenges faced by communities worldwide.
Explanation: A moment of significance 7 April – World Health Day: World Health
unfolded in the realm of archery for India as Day is observed to promote global health
Aditi Gopichand Swami secured a remarkable awareness and highlight a specific health
achievement by earning the first individual theme. It is organized by the World Health
gold medal for India at the World Archery Organization (WHO) and is meant to draw
Championships 2023. This remarkable victory attention to important health issues.
was realized through a gripping competition 22 April – World Earth Day: Earth Day
in the final of the women's compound event in is celebrated to raise awareness about
Berlin, Germany, where she prevailed against environmental issues and promote actions to
Andrea Becerra of Mexico. protect the planet. It emphasizes the need for
sustainable practices and encourages people
Further contributing to this historic occasion, to take part in environmental conservation
the 2023 Berlin World Archery Championships efforts.
witnessed the Indian women's compound
team attaining gold medal glory in the final 20. Option (2) is correct.
by surpassing Mexico. This triumphant team Explanation: Sir Frank Whittle and Dr. Hans
consisted of Jyothi Surekha Vennam, Aditi von Ohain are considered as co-inventor of
Gopichand Swami, and Parneet Kaur, thus the Jet Engine. Hans von Ohain is recognized
establishing themselves as the first Indian as the designer of the first operational turbojet
engine. Frank Whittle was the first to register
women's compound team to claim the world
a patent for the turbojet engine in 1936. Hans
title. von Ohain’s jet was the first to fly in 1939 and
Frank Whittle’s in 1941.
108 OSWAAL CUET (UG ) 10 Mock Test Papers GENERAL TEST

21. Option (2) is correct. 27. Option (2) is correct.


Explanation: If x is subtracted from each Explanation: Let S be the sum of infinite terms
quantity, the new mean is x less than the of GP, having first term a and common ratio r.
earlier mean. a
So, the required new mean = 18 – 2 = 16 So, S1 = =4 (1)
1−r
22. Option (3) is correct. According to the question,
Explanation: Given that: a3
S2 = = 192 (2)
Average score of 50 students = 79.5 1 − r3
\ Total score of 50 students = 79.5 ×50 Putting value of a in equation (2) from (1).
= 3975
According to the question, 64 (1 − r )3
⇒ = 192
3975 − ( 94 + 69 ) + ( 49 + 89 ) 1 − r3
Correct means = ⇒ 2 r 2 + 5r + 2 = 0
50
= 79 1
⇒ r = −2 or
2
23. Option (3) is correct. – 2 is not possible here.
Explanation: The given number is divisible 1
by 72, if it is divisible by 8 and 9 So, r = −
2
So, the number is divisible by 8 if 5x6 is
divisible by 8, 28. Option (4) is correct.
When x = 7, Then 576 ÷ 8 = 72
The number is divisible by 9 if sum of its Explanation: The least number = LCM 4, 5, 8,
digits is divisible by 9. 10 and 12.
So, y + 56 is divisible by 9. 2 4, 5, 8, 10, 12
For y = 7, y + 56 = 63 which is divisible by 9.
2 2, 5, 4, 5, 6
∴ xy  7  7  7
2 1, 5, 2, 5, 3
24. Option (4) is correct. 3 1, 5, 1, 5, 3
Explanation: According to the question, 5 1, 5, 1, 5, 1
Speed of current 1, 1, 1, 1, 1
1
= (Downstream rate – Upstream rate) ∴ LCM of 4, 5, 8, 10 and 12
2
= 2 × 2 × 2 × 3 × 5 = 120
1
= (14 – 8) kmph = 3 kmph So, the least number = 120
2

25. Option (3) is correct. 29. Option (2) is correct.


Explanation: Suppose the numbers be 2x and Explanation:
3x respectively. Given,
As per question, 9 x = 12 + 147
2x + 8 3
= or 9 x  3  2  2  3  7  7
3x + 8 4
or 9x + 24 = 8x + 32 or 9 x = 2 3 + 7 3 = 9 3
or 9x – 8x = 32 – 24 = 8 or x = 3
or x =8
So, the sum of numbers = 2x + 3x = 5x 30. Option (4) is correct.
= 5 × 8 = 40
Explanation:
26. Option (1) is correct. Given that, the base and altitude of an
Explanation: Journey on foot = x km isosceles triangle are 10 cm and 12 cm
Journey on cycle = (80 – x) km respectively.
x 80  x A
So  7
8 16
2 x  80  x
or 7
16 12
or x + 80 = 16 × 7 = 112
or x = 112 – 80 B D C
x = 32 km 10
MOCK TEST Paper-6 109

Now, 32. Option (4) is correct.


AC  AD 2  DC 2 Explanation:
Given: 24010000 = 4900
12    5 
2 2
 and 4900 = 70
 144  25 ∴ 4
24010000 = 70

= =
169 13 cm.
33. Option (1) is correct.
Explanation:
31. Option (1) is correct. The pattern is
3×2+1=6+1=7
Explanation: 7 × 2 + 2 = 14 + 2 = 16
Given that, 16 × 2 + 3 = 32 + 3 = 35
Perimeter of an isosceles right triangle is 35 × 2 + 4 = 70 + 4 = 74 ≠ 70
8  2  1 cm.  74 × 2 + 5 = 140 + 5 = 153
Now, ABC is an isosceles right triangle. 34. Option (1) is correct.
C Explanation: Suppose 0.73 = a and 0.27 = b

Given :
(0.73)3 + (0.27 )3
x (0.73) + (0.27 )2 − (0.73) × (0.27 )
2

a
a3 + b 3 ( a + b )( a 2 − ab + b 2 )
= =
a − ab + b
2 2
a 2 − ab + b 2

A a B = a + b = 0.73 + 0.27 = 1

Using Pythagoras theorem,


35. Option (1) is correct.
Explanation: Let the marked price of the trip
AC2 = AB2 + BC2
= ` x.
or x2 = a2 + a2 = 2a2 and its cost price be ` 100.
or x = a 2  ...(i) S.P. = 141% of C.P. = ` 141
And, Perimeter = 8  2 1  According to the question,
75x
or x + a+ a = 8  2 1  75% of M.P. = ` 141 
141 × 100
100
 141

⇒x= = ` 188
or x + 2a = 8  2  1 75
After a discount of 26%
or a 2  2 a  8  2  1  (From eq. (i)) Required S.P. = (100 – 26)% of ` 188
 188  74 
or a  2  2   8  2  1 =`  
 100 
= ` 139.12
8  2  1
or a  So, required profit % = 39.12
2 2
36. Option (1) is correct.
or a 
8   2 1 2 2 Explanation: According to the question, we
2  2  2  2  have
Width of the road
8 2 2  2  2  2  = r2 – r1
or a  Given that the difference between
2
22  2 circumference is 66 metre. B
∴ 2πr2 – 2πr1 = 66
8 2
or a   [Q a2 – b2 = (a – b) (a + b)]
42
r2
or a = 4 2 A
r1

∴ Length of hypotenuse = 4 2 × 2
= 8 cm.
110 OSWAAL CUET (UG ) 10 Mock Test Papers GENERAL TEST

or 2π (r2 – r1) = 66 41. Option (3) is correct.


66 66  7 462 Explanation:
or r2 – r1 =  
2 2  22 44 ×2 – 1 ×2 – 1 ×2 – 1 ×2 – 1

= 10.5 metres
1 3 5 11 21 43 85 171 341

37. Option (4) is correct. ×2 + 1 ×2 + 1 ×2 + 1 ×2 + 1

Explanation: Given that, Area of square = 24


sq. units 42. Option (3) is correct.
We know that,
Explanation:
Side of square = Area
Correct relation between the given classes,
= 24 = 2 6 units
 
So, perimeter of square = 4  2 6 units Entrepre- Engineers
= 8 6 units neurs

38. Option (1) is correct.


Explanation: C.P. of I toy = ` x Women
C.P. of 2nd toy = ` y
x  112 504  100
So,  504 ⇒ x  = ` 450
100 112 43. Option (1) is correct.
96 504  100
Again, y   504  y  Explanation:
100 96
According to the statements,
= ` 525
Total C.P. = ` (450 + 525) = ` 975 Newspaper
The total S.P. = 2 × 504 = ` 1008
Gain = 1008 – 975 = ` 33 Photographs
33  100 44 5
∴ Required, profit %   3 % Computers
975 13 13

39. Option (2) is correct.


I. () II. () III. ()
Explanation:
Hence, all of the conclusions follows.
Work done by both son in 1 hour =

=
1 1 21 1 44. Option (3) is correct.
  
3 6 6 2
Explanation:
1
∴ Work done by father in an hour = M h z
2
+1 –1 –1
∴ Required time = 2 hours.
N g y
40. Option (1) is correct.
+1 –1 –1
Explanation: Let the required time be t years
 PRT  O f x
According to the question,
SI  100 
  +1 –1 –1
6000  4  5 8000  3  t
⇒  P e w
100 100
+1 –1 –1
120000 24000  t
⇒ 
100 100 Q d v
120000 Hence, Ludo is between the Carrom and
⇒t= = 5 years
24000 Monopoly games.
MOCK TEST Paper-6 111
45. Option (1) is correct. 51. Option (1) is correct.
Explanation: Explanation: So,

1 2 3 4 5
2
3
4
5 will complete the given figure.

Row: 5 + 4 + 3 + 2 + 1=15.
52. Option (3) is correct.
Column: 5+ 4 + 3 + 2 + 1 = 15
= 15 × 15 = 225 Explanation: Dice is single so it is standard
Dice. And we know the sum of the opposite
46. Option (1) is correct. side of standard Dice is 7.
Explanation: As living place of ‘Bee’ is ‘Hive’, So, 3 is opposite to 4.
in the same way living place of ‘Hare’ is called
‘Burrow’. 53. Option (1) is correct.
47. Option (3) is correct. Explanation:
Explanation: We know that, orange is a citrus R M B
fruit. But here orange has been called guava. –2 +2 –2
So, guava will be a citrus food.
P O Z
48. Option (4) is correct.
–2 +2 –2
Explanation: According to the given
N Q X
information,
Sunny/ Vicky < Vicky/Sunny < Hitesh < –2 +2 –2
Bharat < Nitin L S V
Hence, Nitin has the maximum height among
–2 +2 –2
them.
49. Option (3) is correct. J U T
Explanation:
As, 54. Option (1) is correct.
269 278 296
Explanation:
+9 +18
Same as,
3 4 5
313 322 340

+9 +18 ⇒ 1 4 3
2
50. Option (1) is correct. 5 2
1
Explanation:
According to the question, Hence, by joining all the parts we can see the
there ter correct figure.
you are der jer (i)

we ner (ii)
55. Option (3) is correct.
stay here yer mer
664 + 4 ÷ 34 × 28 = ?
we are late ser ner der (iii)
↓  ↓   ↓
I stay there yer fer jer (iv) ÷ + –
From equation (i), (ii), (iii), and (iv), you = ter, ⇒ 664 ÷ 4 + 34 – 28
stay = yer. ⇒ 166 + 6
Hence, you stay late = ter yer ser. ⇒ 172
112 OSWAAL CUET (UG ) 10 Mock Test Papers GENERAL TEST

56. Option (3) is correct. 59. Option (3) is correct.


Explanation: Neither conclusion I nor Explanation:
conclusion II can be proved correct from the J
statement I and statement II. mother

Hence, neither I nor II follows. H R


brother mother er
oth
br
N T U
57. Option (1) is correct. te
r
sis mother
Explanation: Celsius is a unit of P
temperature. From the figure, it is clear that N is the aunt of
Similarly, P and not the sister.
Kilometer is a unit of distance.
60. Option (1) is correct.
Explanation: Omelette …..(3)
58. Option (2) is correct.
(b) Hen …..(1)
Explanation: (c) roll …..(4)
According to the given information, (d) serve …..(5)
T<P<S<Q<R (e) egg …..(2)
Hence, R has the maximum height in the class. Hence, b, e, a, c, d.


SOLUTIONS OF
Question Paper 7
1. Option (3) is correct. Ladakh: Established on 31st October 2019 as
Explanation: Fa Hein, who lived during per the Jammu and Kashmir Reorganisation
405-411 A.D., was a Buddhist monk hailing Act of 2019.
from China. He undertook a journey to Lakshadweep: Formed on 1st November 1956
India during the reign of Chandragupta II under the States Reorganisation Act of 1956.
(Vikramaditya). During his travels in India, Puducherry: Attained union territory status
he made a significant visit to Lumbini, the on 1st November 1954, and was renamed
birthplace of Buddha. Fa Hein documented his Puducherry in 2007.
observations and experiences in a travelogue
titled "Records of Buddhist Kingdom." 3. Option (3) is correct.
Another prominent Buddhist monk from Explanation: Statement 1 is correct. Clement
China was Hiuen Tsang, active between 630- Attlee was the Prime Minister of the United
645 AD. Hiuen Tsang's expedition to India took Kingdom from 1945 to 1951, during which time
place during the rule of Harsha Vardhana. He India gained independence on August 15, 1947.
authored a work known as "The Records of Statement 2 is also correct. James Ramsay
Western World," also referred to as Si-Yu-Ki. MacDonald was the Prime Minister of the
After being hosted by the Kamarupa ruler, United Kingdom from 1924 to 1924 and
Bhaskaravarman, Hiuen Tsang gained an again from 1929 to 1935. In 1932, he made the
audience with King Harshavardhana. declaration of the Communal Award, which
I-Tsing, who travelled from 671-695 AD, was a plan to give separate electorates to
explored India during Harshavardhana's different religious communities in India.
reign. Originating from China, I-Tsing was
drawn to India due to his affiliation with 4. Option (2) is correct.
Buddhism. He extensively chronicled the Explanation: Holding together federation or
lives of Indian monks, producing several cooperative federalism if the system in which
biographical accounts in his writings. power is divided between constituent states
2. Option (2) is correct. and national government. In this system,
the Central government has more power in
Explanation: Collection of Indian Union
comparison to the states.
Territories with Their Establishment Years
Andaman and Nicobar Islands: Formed 5. Option (3) is correct.
on 1st November 1956 as per the States Explanation:
Reorganisation Act of 1956.
1. Article 360 deals with the emergency
Chandigarh: Established on 1st November provisions of the Indian Constitution. It
1966 through the Punjab Reorganisation Act gives the President the power to declare a
of 1966. financial emergency if he is satisfied that
Dadra and Nagar Haveli and Daman and the financial stability or credit of India is
Diu: Unified on 26th January 2020 under the threatened.
Dadra and Nagar Haveli and Daman and Diu 2. Article 324 deals with the establishment of
(Merger of Union Territories) Act of 2019. the Election Commission of India. It also
(Daman and Diu - 1962, Dadra and Nagar lays down the powers and functions of the
Haveli - 1961) Commission.
Delhi: Founded on 1st November 1956 under 3. Article 161 deals with the pardoning
the States Reorganisation Act of 1956. powers of the Governor. It gives the
Jammu and Kashmir: Created on 31st October Governor the power to grant pardons,
2019 according to the Jammu and Kashmir reprieves, remissions and commutations
Reorganisation Act of 2019. of sentences.
114 OSWAAL CUET (UG ) 10 Mock Test Papers GENERAL TEST

4. Article 143 deals with the advisory 11. Option (1) is correct.
jurisdiction of the Supreme Court. It gives Explanation: All the arithmetic and logical
the Supreme Court the power to give operations are performed using a digital
advisory opinions on any question of law circuit called Arithmetic Logic Unit (ALU). The
or fact, that the President may refer to it. ALU has direct output and input access to the
processor controller, main memory (RAM) and
6. Option (3) is correct.
input/output devices.
Explanation: The Dead Sea is a saline lake
situated at the boundary of Jordan to the east
12. Option (1) is correct.
and Israel and the West Bank to the west. Explanation: Amino acids such as citrulline,
Located within the Jordan Rift Valley, it is ornithine, and gamma-aminobutyric acid
primarily fed by the Jordan River. Notably, are categorized as non-protein amino acids.
the surface of the lake rests at an elevation Citrulline, for instance, is synthesized from
of 430.5 meters below sea level, marking its ornithine and carbamoyl phosphate as part of
shores as the Earth's most profoundly situated the urea cycle. This amino acid is considered
terrestrial point. non-essential. Similarly, ornithine, another
7. Option (3) is correct. non-protein amino acid, holds a significant role
in the urea cycle. On the other hand, gamma-
Explanation: Nyishi is one of the primary
aminobutyric acid (GABA) functions as the
ethnic groups and influential tribes in the
primary inhibitory neurotransmitter within
northeastern state of Arunachal Pradesh.
the central nervous system of mammals. GABA
Bangai and Dafla also understand it in some
serves to hinder the postsynaptic regeneration
zones of Arunachal Pradesh.
of action potential.
8. Option (3) is correct.
13. Option (4) is correct.
Explanation: Masai tribe: The Masai tribe is
Explanation: Liver: The liver, which holds the
a Nilotic ethnic group inhabiting southern
Kenya and northern Tanzania. They are distinction of being the body's largest organ,
known for their distinctive red garments, weighs approximately 1.2 to 1.5 kg in an adult
elaborate beadwork, and warrior culture. human. Positioned within the abdominal
Bedouin tribe: The Bedouin tribe is a nomadic cavity, its chief role involves the synthesis of
Arab ethnic group that inhabits the desert bile.
regions of North Africa, the Middle East, and Small Intestine: Serving as the primary site
Central Asia. They are known for their camel for nutrient absorption, the small intestine
herding and their traditional way of life. is responsible for finalizing the digestion
Hausa tribe: The Hausa tribe is a West African process.
ethnic group that inhabits northwestern Large Intestine: Unprocessed and unabsorbed
Nigeria and adjacent southern Niger. They are materials from the small intestine find their
the largest ethnic group in West Africa, and way into the large intestine. This segment of
their language, Hausa, is the lingua franca of the digestive system is also responsible for
much of the region. the absorption of water, certain minerals, and
Bushmen tribe: The Bushmen tribe is a pharmaceutical compounds.
Khoisan ethnic group that inhabits the Pancreas: With a composite nature
Kalahari Desert in Southern Africa. They are encompassing both exocrine and endocrine
known for their hunter-gatherer lifestyle and
functions, the pancreas generates an alkaline
their unique language and culture.
pancreatic fluid laden with enzymes through
9. Option (2) is correct. its exocrine component. Simultaneously,
its endocrine component releases essential
Explanation: When the fiscal deficit is high,
hormones like insulin and glucagon.
there is no direct impact on the prices. When
the government spends more money then 14. Option (4) is correct.
what it earned during the fiscal year, then it is
Explanation: The International Development
known as fiscal deficit.
Association (IDA) is administered by the
10. Option (4) is correct. international Bank for Reconstruction and
Development (IBRD) to provide emergency
Explanation: A logic gate is a straight forward
assistance to the poorest developing countries.
building block of a digital circuit. Most logic
gates have two inputs and one output. There It is a member of World Bank group and is
are seven fundamental logic gates: AND, OR, headquartered in Washington, D.C. United
XOR, NOT, NOR, XNOR and NAND. States of America.
MOCK TEST Paper-7 115
15. Option (4) is correct. The sphygmomanometer is used to measure
Explanation: Padma Bhushan: This accolade blood pressure. It consists of an inflatable cuff
is bestowed in recognition of outstanding that is wrapped around the arm, a pressure
contributions in any domain, encompassing gauge, and a stethoscope. The cuff is inflated
services provided by government officials. It until the blood flow is stopped, and then the
pressure is slowly released. The stethoscope
stands as the third-highest civilian honour in
is used to listen to the sounds of the blood
India and was inaugurated in 1954.
flowing through the arteries. The blood
Bharat Ratna: The highest civilian award is pressure is measured as the pressure at which
presented for extraordinary accomplishments the first sound is heard and the pressure at
in the promotion of art, literature, science, and which the last sound is heard.
noteworthy contributions to public service. In The electrometer is used to measure electric
a historic event in 1987, Khan Abdul Ghaffar charge or electrical potential difference. It is a
Khan became the first foreign national to be device that can detect and measure very small
honoured with this award. electric charges. Electrometers are used in a
Padma Vibhushan: This distinction is granted variety of applications, such as studying the
for exceptional and illustrious achievements properties of atoms and molecules, detecting
across various fields, encompassing services cosmic rays, and measuring the electrical
rendered by government officials. It holds activity of the brain.
the status of being the second-highest civilian 19. Option (4) is correct.
honour in India.
Padma Shri: Recognizing exceptional service Explanation: Dangi is not a folk dance
belonging to the union territory of Jammu
across various sectors, including contributions
and Kashmir. Some folk dances of Jammu and
by government servants, this award is
Kashmir are – Bhand Pather, Bachha Nagma,
conferred as the fourth-highest civilian Hafiza Dance, Bhand Jashan and Wuegi-
honour in India. Nachun. Rouf dance form is practiced on
festive occasions like Eid and Ramzan days are
16. Option (4) is correct. going.
Explanation: An exothermic reaction is a 20. Option (1) is correct.
chemical reaction that emits energy via
Explanation: Abhinav Bindra is the first Indian
light or heat, and it is the opposite of the
Endothermic reaction. Exothermic reactions gold Medal winner in individual category in
include burning natural gas, respiration, and Olympic games. He is a professional shooter.
the decay of vegetables into compost. He won gold in the 10 m Air Rifle event at
the 2008 Beijing Olympic games. It was also
17. Option (4) is correct. India’s first gold medal since 1980, when the
Explanation: Prussic acid is the chemical name Men’s Field Hockey team won the gold.
of the deadly poison—Cyanide. It is a highly
21. Option (2) is correct.
poisonous liquid with the chemical formula
HCN. Explanation: (a) Single equivalent discount for
20% and 15%
18. Option (2) is correct.
 20  15 
  20  15   %  32%
Explanation: The sonometer is used to study  100 
the laws of vibrating strings. It is a long, thin Single equivalent discount for 32% and 10%
string that is stretched between two supports.
 32  10 
The string is attached to a device that can   32  10    38.8%
 100 
vibrate at different frequencies. The sound
produced by the vibrating string can be (b) Single equivalent discount for 25% and 12%
analyzed to learn about the properties of the  25  12 
  25  12   34%
string.  100 
The hydrometer is used to measure the specific Single equivalent discount for 34% and 8%
gravity (relative density) of liquids. It is a float  34  8 
  34  8  %
with a weighted bottom and a graduated scale.  100 
The float is placed in the liquid, and the level = 42 – 2.72 = 39.28%
of the liquid on the scale indicates the specific Hence, the successive discounts of 25%, 12%
gravity of the liquid. and 8% is better.
116 OSWAAL CUET (UG ) 10 Mock Test Papers GENERAL TEST

22. Option (1) is correct. 27. Option (2) is correct.


Explanation: We have, Explanation: LCM of 15, 18 and 36.
Mean of x, x + 2, x + 4, x + 6 and x + 8 = 11 2 15, 18, 36
x+x+2+x+4+x+6+x+8
∴ = 11 2 15, 9, 18
5
⇒ 5x + 20 = 55 3 15, 9, 9
⇒ x=7 3 5, 3, 3
5 5, 1, 1
23. Option (3) is correct.
1, 1, 1
Explanation: ∴ LCM of 15, 18 and 36
Let P be the principal, then, A = 2P. = 2 × 2 × 3 × 3 × 5 = 180
R 
T So, the required number = 180k + 9, where k

Now, A  P 1   is a positive integer
 100 
5
∴ 180k + 9 is divisible by 11, which is true
 R  for k = 6.
 2P  P  1  
 100  Now, put k = 6
On cubing both sides, ∴ Required number = 180 × 6 + 9
15 = 1080 + 9 = 1089
R 
⇒ 8 P  P  1  
 100  28. Option (1) is correct.
∴ Required time = 15 years Explanation: We know that,
M1 D1 T1 M 2 D 2 T2
24. Option (4) is correct. =
W1 W2
Explanation: Given number is divisible by 12
if it is divisible by 3 and 4 18  10  8 M 2  16  6

This number is divisible by 3, if the sum of its 3 5
digits is divisible by 3, or 6 × 10 × 8 × 5 = M2 × 16 × 6
∴ The sum of digits of the given number =
22 + x [divisible by 3 for x = 2] or M  6  10  8  5  2400  25 men
2
16  6 96
25. Option (2) is correct.
29. Option (4) is correct.
Explanation: Given that:
Number of balls in bag: a + 2b + a − 2b 3
Explanation: Given: =
red = 6 a + 2b − a − 2b 1
blue = 4 By componendo and dividendo, we get
white = 10
a  2b  a  2b  a  2b  a  2b
So, probability of a ball being neither white
nor blue = probability of ball being white a  2b  a  2b  a  2b  a  2b
6 3 1
\ Required probability  
( 6  4  10 ) 3 1
6 3
  a  2b 3 1
20 10 ⇒ 
a  2b 3 1
26. Option (3) is correct.
On squaring, we get
Explanation: Given:
Average weight of students = 48.6kg a + 2b 3 + 1 + 2 3 4 + 2 3
Number of girls = 50 – 20 = 30 = =
a − 2b 3 + 1 − 2 3 4 − 2 3
Total weight of 50 students
= (48.6 × 50) kg = 2430 kg a  2b 2  3
⇒ 
Total weight of 20 boys a  2b 2  3
= (54 × 20) kg = 1080 kg By componendo, and dividendo, we get
So, the average weight of girls
a  2b  a  2b 2  3  2  3
 2430  1080   1350  
= kg   a  2b  a  2b 2  3  2  3
30   kg = 45 kg
   30 
MOCK TEST Paper-7 117

⇒ 2a = 4 ⇒
a
=
2
 1 
2

4b 2 3 2b 3  4x2  4   a  
 a
a 4
⇒ = 1
b 3  a
a
∴ a:b =4: 3
1 1 
 x2  1   a  
30. Option (4) is correct. 2 a
Explanation: Given that, x2  1
A  Expression 
x  x2  1
N M
1 1 
a
O
2  
a
B L C 
1 1  1 1 
a   a 
∆ABC is an equilateral triangle. 2  a  2

a
OA = 16 cm
As per question, we have 1 1 
a
AB= BC = CA = 2x cm. 2  
a

2 1
OA = AL
3 a
3 3 × 16
So, AL = OA = = 24 cm. 1  1 
2 2  a a 
2  a
Now, In ∆ABL,
BL = x cm, and ∠ALB = 90° 1
  a  1
2
∴ AL = AB 2  BL2  4 x 2  x 2  3x
33. Option (2) is correct.
or 3x = 24
Explanation:
or x = 8 3 cm.
Total production of type E
∴ Semi-perimeter of ∆ABC = 3x
Cars = (20 + 42 + 40 + 35 + 43)
 3  8 3  24 3 cm.
= 180 thousands
31. Option (1) is correct. 180 = 360˚
360
Explanation: Let the positive number be x, ∴ 42 thousand= × 42 
180
According to the questions,
x2 – 21x = 100 = 84˚
2
⇒ x – 21x – 100 = 0 34. Option (2) is correct.
⇒ x2 – 25x + 4x – 100 = 0
Explanation: Given that,
⇒ x (x – 25) + 4(x – 25) = 0
Radius of wheel = 21 cm
⇒ (x – 25) (x + 4) = 0
No. of revolutions = 10
⇒ x – 25 or x + 4 = 0
Now, distance covered by wheel in one
⇒ x = 25 or x = –4
Revolution = circumference of wheel = 2πr
∴ x = 25 [Qx ≠ –4]
= 2 × 22 × 21
32. Option (3) is correct. 7
= 132 cm
Explanation:
Distance covered in 10 revolutions = 132 × 10
Given that
1 = 1320 cm
On squaring both sides, we get 2 x  a 
a 35. Option (2) is correct.
2
 1  Explanation: Given that,
( 2 x )2   a  
 a Two sides of triangle = 5 cm and 8 cm
1 Since,
or 4 x 2  4  a   2  4
a In a triangle the sum of the two sides is
1 greater than the remaining/third side
a 2 So, now check by options
a
118 OSWAAL CUET (UG ) 10 Mock Test Papers GENERAL TEST

5+2<8 39. Option (4) is correct.


5+3=8 Explanation: Suppose the three numbers are
5 + 8 < 14 x, y and z.
But 5 + 4 > 8 Given that, x + y + z = 280
x : y = 2 : 3 and y : z = 4 : 5
Length of third side = 4 cm
We know that,
36. Option (1) is correct. x : y : z
2 : 3 → 3
Explanation: Let the number of oranges be x.
C.P. of 8 oranges = `34 4 ←4:5
8 : 12 : 15 [each column multiplied]
34
C.P. of x oranges = x 12
8 nd
2 number = × 280
S.P. of 12 oranges = `57 (8 + 12 + 15)
= 12 × 8 = 96
57
S.P. of x oranges = x
12 40. Option (3) is correct.
Profit = `45
Explanation: Let the principal be `x.
S.P. – C.P. = 45
57 34 According to the question,
⇒ x  x  45 x + S.I. for 2 years = `5182(i)
12 8
x + S.I. for 3 years = `5832(ii)
114 x  102 x
⇒  45 On subtracting eq. (i) from eq. (ii), we get
24 S.I. for 1 year
12 = `(5832 – 5182) = `650
⇒ x = 45
24 ∴ S.I. for 2 years
⇒ x = 45 × 2 = 90 = `(2 × 650) = `1300
∴ Principal = `(5182 – 1300) = `3882
37. Option (4) is correct.
Explanation: C.P. of 50 pens = 50 × 50 = ` 2500 41. Option (4) is correct.
2500 × 110 Explanation:
For profit of 10%, S.P. = = ` 2750
100 P
father
40 × 50 × 95
S.P. of 40 pens at a loss of 5% = R T
100
daughter
= ` 1900
Q S U
So, S.P. of remaining 10 pens = 2750 – 1900 sister
daughter
= ` 850 Z
850  500
∴ Required, gain % =  100 = 70% Hence, from the figure, it is clear that R is the
500 maternal grandmother of Z.
38. Option (2) is correct. 42. Option (2) is correct.
Explanation: By Using Alligation Rule, we get Explanation:
Milk-I Milk-II Correct relation between the given classes
Reptiles
3 7
5 2 10 Aquatic
Crocodiles Animals
3
7 2 2 3
− −
10 3 3 5

21−−20
21 20 1010−−99 43. Option (1) is correct.
= ==
30
30 15
15 Explanation: According to the statements,
1 1 Plants
= =
30 15 X
Ornaments Tree
1 1
=
30 15
1 1 I. () II. ()
So, the required ratio = : =1:2
30 15 Hence, both conclusions follow.
MOCK TEST Paper-7 119
44. Option (1) is correct. From option (3),
Explanation: Common in circles and squares 16 80 144
only which is 8. Hence option (1) is the correct ×5
answer.
×9
45. Option (3) is correct.
From option (2),
Explanation: Time = 11 : 9
Time in mirror = 00 : 51 6 30 54
Means 12 : 51 . Hence, option (3) is the correct ×5
answer. ×9
46. Option (4) is correct. From option (1),
Explanation: 8 35 72
Given : 1000 O 5 N 66 M 33 =? ×5-5
×9
÷ + –
So, (8, 35, 72) do not have the same relationship
∴ 1000 ÷ 5 + 66 – 33 = 233
as the given triad.
47. Option (1) is correct.
Explanation: As ‘Tea’ comes under the 51. Option (2) is correct.
category of ‘Beverages’, same as ‘Medicine’
comes under ‘Analgesics’. Explanation:
As, Y O U ; T O O K ;
48. Option (4) is correct.
C O U P O O N
Explanation:
Fast and furious co mo jo (i) M Y; P L A C E and

do it fast cha mo ga (ii) L C Q V A K E

she did it ga la nop (iii) I N


I J
From equation (i), (ii) and (iii), do = cha
Similarly, C O M P I L E
49. Option (2) is correct.
K O L Q I V E
Explanation:
Given
52. Option (3) is correct.
Shweta Palak
27 344 Explanation: Follow the logic and the pattern.
Hence option (3) is the correct answer.
Now, Total girls = Position from left + position 53. Option (4) is correct.
from right – 1
⇒ 74 = 34 + Position from right – 1 Explanation: According to the statements,
⇒ Position from right = 74 – 33 = 41 conclusion II does not follow.
Hence, Palak’s position from right end = 41st Only, conclusion I follows.

50. Option (1) is correct. 54. Option (2) is correct.


Explanation: Explanation:
As,
1
23 115 207 3
×5 2
⇒ 2
×9
3 1
Now, from option (4),
21 105 189
×5 Hence, by joining all the parts we can see the
×9 correct figure.
120 OSWAAL CUET (UG ) 10 Mock Test Papers GENERAL TEST

55. Option (2) is correct. 58. Option (1) is correct.


Explanation:
Explanation:
+5 +5 +5 +5

AB GH MN ST YZ
+1 +1 +1 +1 +1

59. Option (1) is correct.


will complete the figure.

56. Option (4) is correct. Explanation:

Explanation: According to the given information,


Unity Tower > Pearl Tower > Sky Tower…(i)
Mother Unity Tower = Cyber Tower …(ii)
Indus Tower > Pearl Tower …(iii)
Amit Wife Lady From equation (i), (ii) and (iii)
Grand son
Unity Tower = Cyber Tower >
Indus Tower > Pearl Tower > Sky Tower
Mother
Hence, Pearl Tower is the second shortest.
Son
The lady is Amit’s wife. 60. Option (2) is correct.
57. Option (2) is correct. Explanation:
Explanation: 12 12 13 26 28 84 87
C 9 km B N ×1 +1 ×2 +2 ×3 +3
NW
NE
6 km
6 km W E

D SW SE
3 km E A S

∴ Required distance AE = BC – DE
= 9 – 3 = 6 km


SOLUTIONS OF
Question Paper 8
1. Option (3) is correct. associated with the Indian Audit and Accounts
Explanation: Sindhu is the Vedic name for Service during the post-independence era. In
the Indus River. The Indus River is the recognition of his contributions to the civil
longest river in Pakistan and the second- service, he was honoured with the Padma
longest river in India. It flows through the Bhushan, the third-highest civilian award in
India, in 1954.
Himalayas, the Hindu Kush, and the Thar
Desert. Sukumar Sen held the distinction of being
the first Chief Election Commissioner of
Vitasta is the Vedic name for the Jhelum River.
India, holding office from 21 March 1950 to
The Jhelum River is a tributary of the Indus
19 December 1958. Under his leadership, the
River. It flows through the Kashmir Valley and
Election Commission impeccably orchestrated
into Pakistan. India's first two general elections, which took
Asikni is the Vedic name for the Chenab River. place in 1951–52 and 1957.
The Chenab River is a tributary of the Indus The Attorney General of India assumes the
River. It flows through the Punjab region of pivotal role of the government's primary legal
Pakistan and India. counsel and representative in the Supreme
Parushni is the Vedic name for the Ravi River. Court of India. M. C. Setalvad was the
The Ravi River is a tributary of the Indus River. pioneering Attorney General of India, while
It flows through the Punjab region of India the present incumbent is R. Venkataramani.
and Pakistan. The inception of the initial Finance Commission
Vipas is the Vedic name for the Beas River. The took place through a Vice Presidential Order
Beas River is a tributary of the Indus River. dated 22.11.1951, with Shri K.C. Neogy as its
It flows through the Himachal Pradesh and inaugural chairman. This commission was
Punjab regions of India. formed on 6th April 1952. Subsequently, fifteen
Finance Commissions have been established
Sutudri is the Vedic name for the Sutlej River.
at five-year intervals. The President appoints
The Sutlej River is a tributary of the Indus
the Finance Commission under Article 280
River. It flows through the Himachal Pradesh,
of the Constitution, primarily to provide
Punjab, and Haryana regions of India.
recommendations on the equitable allocation
2. Option (1) is correct. of tax revenues between the Union and the
States, as well as among the States themselves.
Explanation: SAARC was established in 1985
and has 8 member countries: Afghanistan, 4. Option (4) is correct.
Bangladesh, Bhutan, India, Maldives,
Explanation: In a splendid event, the victors of
Nepal, Pakistan, and Sri Lanka. BIMSTEC
the 69th National Film Awards were unveiled.
was established in 1997 and has 7 member
Allu Arjun received recognition as the Best
countries: Bangladesh, Bhutan, India,
Actor for his role in the movie "Pushpa,"
Myanmar, Nepal, Sri Lanka, and Thailand.
whereas Alia Bhatt and Kriti Sanon were
Of the given options only Bangladesh, honoured with the Best Actress Award for
Bhutan, and India are members of both their performances in "Gangubai Kathiawadi"
SAARC and BIMSTEC. Pakistan, Maldives, and "Mimi" respectively. "The Nambi Effect"
and Afghanistan are the only members of secured the esteemed title of Best Feature
SAARC, while Thailand and Myanmar are the Film, while "The Kashmir Files" clinched the
only members of BIMSTEC. Nargis Dutt Award for Best Film on National
3. Option (1) is correct. Integration.
• list of winners at the 69th National Film
Explanation: V. Narahari Rao served as the
Awards:
inaugural Comptroller and Auditor General
• Best Feature Film: Rocketry
(CAG) of India, holding the position from
1948 to 1954. An Indian civil servant, he was • Best Director: Nikhil Mahajan, Godavari
122 OSWAAL CUET (UG ) 10 Mock Test Papers GENERAL TEST

• Best Popular Film Providing Wholesome 7. Option (1) is correct.


Entertainment: RRR Explanation: India holds the position of the
• Nargis Dutt Award for Best Feature Film on world's largest millet producer. In the year
National Integration: The Kashmir Files 2020, two types of millet from India, namely
• Best Actor: Allu Arjun, Pushpa Pearl Millet (Bajra) and Sorghum (Jowar),
• Best Actress: Alia Bhatt, Gangubai jointly constituted approximately 19 per cent
Kathiawadi and Kriti Sanon, Mimi of the global millet production. Among these,
• Best Supporting Actor: Pankaj Tripathi, Pearl Millet's share in the world production
Mimi was 40.51 per cent, while Sorghum accounted
• Best Supporting Actress: Pallavi Joshi, The for 8.09 per cent in the same year. Noteworthy
Kashmir Files millet-producing states in India encompass
• Best Child Artist: Bhavin Rabari, Chhello Rajasthan, Karnataka, Maharashtra, Uttar
Show Pradesh, Haryana, Gujarat, Madhya
Pradesh, Tamil Nadu, Andhra Pradesh, and
5. Option (4) is correct.
Uttarakhand. Collectively, these ten states
Explanation: Lakhpati Didi Scheme: The were responsible for nearly 98 per cent of
primary objective of this initiative by the millet production within India during the
government is to empower women in rural 2020-21 period. More specifically, the millet
areas, aiming to generate two crores of production landscape was dominated by six
financially self-sufficient "lakhpati didis" or states: Rajasthan, Karnataka, Maharashtra,
prosperous sisters. This scheme is aligned with Uttar Pradesh, Haryana, and Gujarat,
the broader mission of reducing poverty and contributing to over 83 per cent of the total
enhancing economic empowerment. millet yield. Rajasthan, in particular, stood out
Under the Lakhpati Didi Scheme, women by contributing 28.61 per cent of India's entire
residing in villages will undergo skill millet production.
development training to enable them to earn 8. Option (1) is correct.
an annual income exceeding Rs 1 lakh.
Explanation: The largest fish-exporting region
The scheme involves the provision of drones
to women's Self-Help Groups (SHGs) for of the world is the Northeast Atlantic region. It
agricultural tasks. accounts for about 30% of the global fish export
market. The region includes countries such as
This endeavour intends to blend technology
Norway, Iceland, and the Faroe Islands. These
with the agricultural sector's transformation,
countries have a long history of fishing and
concurrently bolstering the status of women
aquaculture, and they have developed highly
in rural societies.
efficient fishing and processing industries.
Approximately 15,000 SHGs composed of
women will receive comprehensive training in The Northeast Atlantic region is also home to
drone operation and maintenance. some of the most productive fishing grounds
in the world. The region's cold, nutrient-rich
This training not only opens doors to fresh
waters support a wide variety of fish species,
income-generating avenues but also equips
including cod, haddock, and salmon.
women with state-of-the-art proficiencies.
The other regions you mentioned are also
Drones hold immense potential to
important fish exporters, but they do not
revolutionize agriculture through precision
farming, real-time crop surveillance, and export as much fish as the Northeast Atlantic
effective pest management. region. The Northwest Pacific region is the
second largest fish exporting region, followed
Additionally, the program encompasses
by the Southeast Asian region.
training women in a range of skills such as
crafting LED bulbs and plumbing, among 9. Option (4) is correct.
others.
Explanation: Hatti tribe: The Hatti tribe is
6. Option (1) is correct. found in the state of Himachal Pradesh,
primarily in the Chamba district. They are
Explanation: The thermal growth causes the known for their distinct cultural practices and
Eiffel Tower to be taller during summers. are part of the larger Gaddi community in the
Thermal elaboration is when material changes region.
measurements, it experiences temperature Narikoravan tribe: The Narikoravan tribe
differences. Materials, the tower made of iron resides in the state of Tamil Nadu, primarily
experiences thermal expansion. in urban areas like Chennai. Historically, they
MOCK TEST Paper-8 123

were involved in various professions like Martyrs Day or Shaheed Diwas is celebrated
street performance, acrobatics, and fortune- on 30 January every year to commemorate
telling. Over time, they have faced social and the death anniversary of Mahatma Gandhi.
economic challenges. Mahatma Gandhi was the leader of the
Toda tribe: The Toda tribe is indeed from Indian independence movement and is
Tamil Nadu, primarily residing in the Nilgiri considered the father of modern India. He was
Hills. They are known for their unique culture assassinated on 30 January 1948.
and distinctive barrel-shaped huts. The 13. Option (2) is correct.
Toda people traditionally practised buffalo
pastoralism and have a rich mythology and Explanation: Mihir sen was a famous long
religious system. distance swimmer. He was first Indian to
Binjhia tribe: The Binjhia tribe is predominantly swim across English Channel from Dover
found in the state of Jharkhand. They are one to Calais in 1958. He was the only man who
of the indigenous communities of the region, swim the oceans of five continents in one year.
with their own cultural traditions, languages, 14. Option (1) is correct.
and way of life. The Binjhia tribe was already
listed as ST in Jharkhand and Odisha but not Explanation: Louis Pasteur was a French
in Chhattisgarh. Now, this tribe is added to the microbiologist who is considered the father
list of STs in Chhattisgarh. of microbiology. He made many important
discoveries in the field of microbiology,
10. Option (2) is correct. including the discovery of fermentation.
Explanation: Fiber optics (or optical fiber), Edward Jenner was an English physician who
refers to the medium and the technology is credited with developing the first smallpox
associated with a high-speed data transmission. vaccine. The smallpox vaccine is a suspension
It contains tiny glass or plastic filaments that of cowpox virus that is injected into the body
carry light beams. The light signals do not to protect against smallpox.
travel at the speed of light because of the James Watson and Francis Crick were American
denser glass layers, instead travelling about molecular biologists who discovered the
30% slower than the speed of light. structure of DNA. DNA is the genetic material
11. Option (2) is correct. that contains the instructions for building and
maintaining an organism.
Explanation: The male part of a flower is
Alexander Fleming was a Scottish
stamen. It is composed of the filament and the
bacteriologist who discovered penicillin.
anther. It bears the pollen grain. The female
part of a flower is called the pistil and it is Penicillin is an antibiotic that is used to treat
composed of ovary, the stigma, and the style. bacterial infections.

12. Option (4) is correct 15. Option (3) is correct.


Explanation: World Hindi Day is celebrated Explanation: Bal Gangadhar Tilak was a
on 10 January every year to promote the freedom fighter and social reformer who is
Hindi language. It was first celebrated in 2006 considered the "Father of Indian Unrest". He
under the instructions of then-Prime Minister was a strong advocate of Swaraj, or self-rule,
Manmohan Singh. The date of 10 January was for India.
chosen as Hindi was spoken for the first time Dr Rajendra Prasad was the first President of
in the United Nations General Assembly on 10 India. He was a respected lawyer and politician
January 1949. who was known for his honesty and integrity.
National Youth Day is celebrated on 12 January He was given the nickname "Deshbandhu",
every year to mark the birth anniversary of which means "friend of the nation".
Swami Vivekananda. Swami Vivekananda was Dadabhai Naoroji was an economist and
a Hindu monk and philosopher who played politician who is considered the "Grand old
a major role in the introduction of Indian man of India". He was a strong advocate for
philosophy to the West. He is also considered Indian independence and was the first Indian
the father of modern India. to be elected to the British Parliament.
National Girl Child Day is celebrated on 24 Chittaranjan Das was a freedom fighter and
January every year to raise awareness about lawyer who was known for his eloquence and
his commitment to social justice. He was given
the girl child and to promote her rights. The
the nickname "Desh Ratna", which means
day was first celebrated in 2011 by the Ministry
"jewel of the nation".
of Women and Child Development.
124 OSWAAL CUET (UG ) 10 Mock Test Papers GENERAL TEST

16. Option (2) is correct. 21. Option (2) is correct.


Explanation: Marfati songs are traditional folk Explanation: By using the rule of alligation,
songs of Bangladesh. Purbeli songs are folk we get
songs of Nepal. Cartman’s songs, Baul songs, Variety I Variety II
Jari gaan, Kabi gaan and Pala gaan are some (` 12) (` 7)
other traditional Bangladeshi songs.
(` 8)
17. Option (3) is correct.
Explanation: Savitribai Phule belongs to 8-7=1 12 - 8 = 4
Maharashtra. She was born on January 3, So, the required ratio = 1 : 4
1831, and was married to Jyotirao Phule. Her
husband decided to educate her in order 22. Option (4) is correct.
to elevate the status of women. She was a
Explanation: After arranging the data in
social reformer and worked a lot for women ascending order.
empowerment. She died in 1897 of Plague 3, 5, 6, 7, 9, 9, 10
while helping the people affected with Here the number of observations is odd.
Bubonic Plague epidemic. Her writings – th
 n + 1
Kavya Phule; Go, Get Education; Bavan Kashi So, Median =  term
 2 
Subodh Ratnakar.
th
 7 + 1
18. Option (3) is correct. = term
 2 
Explanation: The Archaeological Survey of = 4 th term
India (ASI) made a remarkable discovery of a
=7
1,300-year-old stupa right within a mining site
in the Jajpur district of Odisha. This location
served as the source of Khondalite stones 23. Option (2) is correct.
used in the beautification project of the Shree Explanation: Single equivalent discount for
Jagannath Temple in Puri during the 12th 10% and 12%
century.
Preliminary assessments suggest that the  12  10 
  12  10   %  20.8%
stupa could be around 4.5 meters in height,  100 
possibly dating back to the 7th or 8th century.
Single equivalent discount for 20.8% and 5%
This significant find was made at Parabhadi,
situated in close proximity to Lalitagiri, a  20.8  5 
  20.8  5   %  24.76%
prominent Buddhist complex renowned for  100 
its numerous stupas and monasteries.
Lalitgiri holds a special place among the three 24. Option (3) is correct.
sites (Lalitagiri, Ratnagiri, and Udayagiri) due
Explanation: 4x573y is divisible by 72.
to its belief as the most sacred. It is here that a
massive stupa was excavated, housing a relic ∴ 4x573y is divisible by 8 and 9.
of Buddha inside a stone casket. 73y is divisible by 8.

19. Option (3) is correct. ∴y=6


4x573y is divisible by 9.
Explanation: The substance used as a
Then 4 + x + 5 + 7 + 3 + 6 = 27
replacement for chlorine in bleaching
is Hydrogen peroxide. It is an excellent ⇒ 25 + x = 27 ⇒ x = 27 – 25 = 2
substitute for bleach because of the oxidation ∴x+y=2+6=8
of colouring matter by nascent oxygen. It is
used for bleaching of wool, silk and cotton.
25. Option (2) is correct.
Explanation:
20. Option (1) is correct.
T
 R 
Explanation: In the pure state, Tungsten A  P1 
has the most elevated melting point, i.e.,  100 
2
3422°C or 6192°F. It also has the most  R 
increased tensile force and most low 4000  2000  1  
 100 
vapor pressure. Helium is the chemical 2
component with the lowest melting point,  R 
 2 1 
while the most increased one is Carbon.  100 
MOCK TEST Paper-8 125

R ...(i) 29. Option (4) is correct.


 1  2
100 Explanation: Let the number be x.
 R 
T According to the question,
Now, 8000  2000  1   7% of x = 84
 100  7
 4  ( 2 )T [ From eq (i )]  x  84
100
4
 ( 2)  ( 2) T
8400
x  1200
7
⇒ T = 4 years
30. Option (1) is correct.
26. Option (1) is correct.
Explanation:
Q3=3×1
Explanation: Total possible outcome when a 5=5×1
pair 7=7×1
of dice thrown = 36 9=3×3
According to the question, possible ∴ LCM of 3, 5, 7 and 9 = 3 × 5 × 7 × 3 = 315
We know that, largest 4-digit number = 9999
Outcomes = (1, 2), (1, 4), (2, 1), (2, 3), (2, 5),
(3, 2), (3, 4), (4, 1), (4, 3), (5, 2), (5, 6), (6, 1), 315 9999 31
(6, 5), (1, 6) 945
So required probability = 14/ 36 549
= 7/18 315
234
27. Option (2) is correct. So, the greatest 4-digit number divisible by
315
= 9999 – 234
Explanation:
= 9765
Given, Q The difference between each divisor and
remainder is 2.
x3 3
1  ∴ Required number = 9765 – 2 = 9763
100 5
31. Option (1) is correct.
On squaring both sides, we get
Explanation:
3 Given, first term a = –20
x 9
1− = last term l = 28
100 25
and n = 17
x3 9 n
or =1− ∴ Required sum = ( a + l )
100 25 2
17
x 3 25 −9 16 = ( 20  28 )
or = = 2
100 25 25
17  8
=  68
16  100 2
or x 3   64
25
32. Option (2) is correct.
3
or x = ( 4 )
3
Explanation: Distance covered
5
or x = 4   30  25km
6
Time Taken = 3 hours 20 minutes
28. Option (1) is correct. 1 10
 3 hours  hours
3 3
Explanation: Distance
Total production of cars in 2018 = 270 So, speed 
Time
thousands = 38 + 40 + 54 + 68 + 70 = 270
thousands  25 
 km / h
10 
Production of type C cars = 54 thousands  
 3 
Now, 270 thousand ≡ 360˚
 25  3 
  km / h
360  10 
∴ 54 ≡ × 54 = 72˚
270 = 7.5 km/hr
126 OSWAAL CUET (UG ) 10 Mock Test Papers GENERAL TEST

33. Option (4) is correct. Now, ∠BCD = ∠DCA = 30°


Explanation: Given that, ∠DCE = 180°
A circular wire of length = 168 cm ∴ ∠ACE = 180° – 30° = 150°
Rectangle sides ratio = 5 : 7 AC = CE
Now, Breadth of rectangle = 5x cm 30°
Length of rectangle = 7x cm ∴ ∠CAE = ∠CEA = = 15°
2
Now, perimeter = 2 (l + b)
168 = 2 (7x + 5x) 39. Option (3) is correct.
168 Explanation:
or 12x = = 84
2 1 1
84 2 days’ work of (B + C)’s = 2   
or x = =7  20 30 
12
32
Diagonal of rectangle =  7 x 2   5x 2 = 2  
 60 
= 49 x 2 + 25x 2 5 1
= 2 
= 74 x 2  74  7 2  3626 cm 60 6
1 5
34. Option (4) is correct. Now remaining work = 1 
6 6
Explanation: 5
∴ A’s complete work =  18  15 days
100 6
For A, C.P. of horse = 4800 × = ` 6000
80
6000 × 115 40. Option (2) is correct.
For B, S.P. = = ` 6900
100
B’s profit = ` (6900 – 4800) = ` 2100 7− 3
Explanation: Given: N =
7+ 3
35. Option (4) is correct.
( 7  3 )( 7  3 )
Explanation: 
( 7  3 )( 7  3 )
Here, x = 5%, R = 240, y = 8%
7  3  2 7  3 10  2 21
R 240  
∴ C.P. =  100   100 = ` 8000 73 4
yx 85
2( 5  21 ) 5  21
36. Option (3) is correct.  
4 2
Explanation: Here, a = 7, b = 16, c = 43 and 1 2 2 5  21
d = 79 Now,   
bc − ad N 5  21 5  21 5  21
So, the required number x =
( a + d ) −(b + c ) 5  21
16 × 43 − 7 × 79 
= 2
(7 + 79 ) −(16 + 43)
1 5  21 5  21 10
688 − 553 35  N    5
= = =5 N 2 2 2
86 −79 7
37. Option (1) is correct. 41. Option (1) is correct.
Explanation: Explanation:
+2 +2 +2 +2 +2
In an equilateral ∆ABC,
∠A = ∠B = ∠C = 60°
a, cd, fgh, jklm, opqrs, uvwxyz
AB = BC = CA
1 2 3 4 5 6
∴ AX = BY = CZ
42. Option (1) is correct.
38. Option (4) is correct.
Explanation:
Explanation:
Correct relation between the given classes
Given that,
∆ABC is an equilateral triangle Literates Teachers
AC = CE Graduates
A

B C E
MOCK TEST Paper-8 127
43. Option (3) is correct. Similarly,
Explanation: D E C O D E B F A P B F
According to the statements,
–2
Tree +1
X –2
Pomeg- +1
Plum
ranates
–2
+1
I. () II. () III. ()
50. Option (2) is correct.
Hence, all the conclusions follow.
Explanation:
According to the given information,
44. Option (3) is correct. Q<P<T<S<R
Explanation: Hence, Q will have the least marks.
51. Option (2) is correct.
Left Right
Explanation: 17 + 15 = 32
E B A C D
52. Option (1) is correct.
Center
Explanation:
45. Option (2) is correct. As, D R O N E
Explanation: +1 +1 +3 +1 +1
Given : 30 × 3 – 3 = 13 S E R F O
Similarly,
÷ + H O U S E
+1 +1 +3 +1 +1
⇒ 30 ÷ 3 + 3 = 13 P I X F T

46. Option (3) is correct. 53. Option (4) is correct.


Explanation: Neither conclusion can be drawn Explanation:
from the given statements. C 2 km B
Hence, neither conclusion I nor conclusion II N
follow.
3 km 3 km W E
47. Option (1) is correct.
D E S
Explanation: 2 km A
3 km
A A, B D, C I, D P, E Y,
∴ Required distance AE = DE – DA
2 2 2 2 2 = 3 – 2 = 1 km
1 =1 2 =4 3 =9 4 = 16 5 = 25
54. Option (1) is correct.
48. Option (4) is correct. Explanation:
Explanation: Open, Type, Save, Print, Close. 5 8 13 20 29 40 53 68 85
Hence, option 4 is the correct answer.
+3 +5 +7 +9 +11 +13 +15 +17
49. Option (1) is correct.
2 2 2 2 2 2 2
Explanation:
55. Option (1) is correct.
As,
F I G U R E D J E V P F Explanation:

–2
+1
–2
+1
–2
+1 will complete the given figure.
128 OSWAAL CUET (UG ) 10 Mock Test Papers GENERAL TEST

56. Option (2) is correct. 58. Option (1) is correct.


Explanation: Explanation:
E 25 m D
3 5m
G N
B 20 m C
1 2 25 m
W E
4 5 20 m
S

F Starting point
A

5 1 4 GB = GC – BC {Q GC = ED}
= 25 – 20 = 5 m
2 ∴ Required distance = AF = GB = 5 m

59. Option (4) is correct.


3
Explanation:
Hence, by joining all the parts we can see the
E F
correct figure.
father
57. Option (3) is correct. H J
sister
Explanation: son r
te
sis
Husband G D
Veena
m
ot
he Thus, D is the niece of H.
r
60. Option (3) is correct.
Sister Explanation:
Son Kavi India is a country.
Similarly,
Therefore, Veena is the mother of Kavi.
Delhi is a capital.


SOLUTIONS OF
Question Paper 9
1. Option (1) is correct. 3. Option (3) is correct.
Explanation: Rudradaman (130 AD-150 AD), Explanation: Article 123 of the Constitution of
one of the most renowned Saka rulers in India, India deals with the power of the President
achieved distinction not only for his military to promulgate ordinances during the recess
triumphs but also for his notable public of Parliament. An ordinance is a law that is
endeavors. enacted by the President on his own authority.
His dominion spanned across a substantial It has the same force and effect as an Act of
portion of western India, encompassing Parliament, but it ceases to have effect after a
regions such as Sindh, Gujarat, Konkan, period of six weeks from the date of reassembly
Narmada Valley, Malwa, and Kathiawar, of Parliament.
excluding the Nashik and Pune areas. Article 213 of the Constitution of India
His historical prominence arises from his deals with the power of the Governor to
extensive efforts to enhance the Sudarshan promulgate ordinances during the absence
Lake, an initiative that dates back to the reign of of the Legislative Assembly of a State. The
Chandragupta Maurya. This undertaking took Governor can promulgate ordinances on the
place in the semi-arid zone of Kathiawar. This same grounds as the President can promulgate
remarkable feat is detailed in the celebrated ordinances under Article 123.
Junagarh or Girnar Inscription, precisely dated So both Article 123 and 213 are related to the
to the Shaka year 72, equivalent to 150-151 CE. ordinance in the Indian Constitution.
2. Option (1) is correct. 4. Option (4) is correct.
Explanation: Article 52: This article establishes Explanation: United Nations Educational,
the position of the President of India. It states Scientific and Cultural Organization
that there shall be a President of India who (UNESCO): UNESCO is headquartered in
shall be the head of the state and exercise Paris, France. It is a specialized agency of the
powers and functions in accordance with the United Nations aimed at promoting peace and
Constitution. security through international cooperation in
Article 74: This article deals with the Council education, the sciences, and culture.
of Ministers and their relationship with the
Organization for Economic Cooperation
President. It states that there shall be a Council
and Development (OECD): The OECD
of Ministers with the Prime Minister as its head
is headquartered in Paris, France. It is an
to aid and advise the President who shall, in
the exercise of his functions, act in accordance intergovernmental economic organization
with such advice. with a mission to promote policies that
Article 78: This article outlines the improve the economic and social well-being of
responsibilities of the Prime Minister regarding people around the world.
the furnishing of information to the President. International Council on Monuments
It states that it shall be the duty of the Prime and Sites (ICOMOS): ICOMOS is an
Minister to communicate to the President all international non-governmental organization
decisions of the Council of Ministers relating headquartered in Paris, France. It is dedicated
to the administration of the affairs of the to the conservation of cultural heritage places
Union and proposals for legislation. worldwide.
Article 61: This article outlines the procedure World Intellectual Property Organization
for the impeachment of the President. It states (WIPO): WIPO is headquartered in Geneva,
that when a President is to be impeached for
Switzerland. It is a specialized agency of the
violation of the Constitution, a charge can be
United Nations that deals with intellectual
initiated by either House of Parliament. The
property matters globally, including patents,
procedure for investigation and trial in such
cases is specified in this article. copyrights, trademarks, and related issues.
130 OSWAAL CUET (UG ) 10 Mock Test Papers GENERAL TEST

5. Option (4) is correct. The acquisition of Lavasa is a major


Explanation: Ukraine – The Bread Basket of development for the real estate sector in India.
Europe: Ukraine is often referred to as the Lavasa is a planned hill station located in the
"Bread Basket of Europe" due to its fertile Pune district of Maharashtra. It is a popular
soil and significant agricultural production, tourist destination and is also being developed
particularly of grains like wheat. Its agricultural as a residential and commercial hub.
resources have historically contributed to The acquisition of Lavasa by Darwin
providing food for both its own population Platform Infrastructure is expected to boost
and other regions. the development of the hill station. Darwin
Bhutan – Land of the Thunder Dragon: Platform Infrastructure has a strong track
Bhutan is commonly known as the "Land record of developing infrastructure projects
of the Thunder Dragon" due to its dramatic and is expected to bring in new investment
landscape of steep mountains, deep valleys, and expertise to Lavasa.
and the prevalence of thunderstorms in the
region. The dragon is a symbol of Bhutanese 9. Option (2) is correct.
mythology and culture. Explanation: An operating system is a
North Korea – The Hermit Kingdom: North software that performs all basic tasks such as
Korea is often referred to as "The Hermit managing files and memory. Process control,
Kingdom" because of its isolationist policies I/O processing, and control of peripheral
and restricted interactions with the outside devices such as disks drivers and printers. It
world. The term highlights the secretive and acts as a link between the computer user and
closed nature of the country's government the computer hardware.
and society.
10. Option (1) is correct.
Afghanistan – Graveyard of Empires: This
phrase, "Graveyard of Empires," is more Explanation: Nagappattinam, Chinsura,
accurately associated with Afghanistan. Machilipatnam: The Dutch East India
It conveys the historical difficulty that Company (VOC) established trading posts
various empires have encountered when in Nagappattinam in present-day Tamil
attempting to conquer and maintain control Nadu, Chinsura in present-day West Bengal,
over Afghanistan due to its rugged terrain, and Machilipatnam in present-day Andhra
tribal society, and fierce resistance from local Pradesh. These trading posts were strategically
populations. situated along the coast to facilitate trade and
commerce.
6. Option (4) is correct.
Surat, Bharuch, Agra: The Dutch also had
According to Pascal’s principle, pressure is trading centers in Surat and Bharuch, both
equal to the force divided by the area on which located in present-day Gujarat. Surat was a
it acts. When there is an increase in pressure at major port city, and the Dutch established
any point in a confined fluid, there is an equal their presence there for trade. Agra, situated
increase at every other point in the container. in present-day Uttar Pradesh, was another
location where the Dutch set up a trading post.
7. Option (4) is correct. Cochin, Ahmedabad, Patna: Cochin, located in
Explanation: Monopsony is a market form in present-day Kerala, was an important trading
which one buyer faces many sellers. It is an hub for the Dutch in South India. Ahmedabad,
example of imperfect competition. in present-day Gujarat, was known for its
textile trade, and Patna, in present-day Bihar,
8. Option (1) is correct. was a center for indigo production.
Explanation: Lavasa, India's inaugural The Dutch East India Company aimed to
privately developed hill station, was acquired establish these trading centers to tap into
by Darwin Platform Infrastructure for ` 1.8 India's valuable trade commodities, including
thousand crore. spices, textiles, and other goods. These
The acquisition was approved by the National locations served as bases for their commercial
Company Law Tribunal (NCLT) on August activities and interaction with local merchants
23, 2023. Darwin Platform Infrastructure is a and markets. Over time, the Dutch presence in
Mumbai-based company that is involved in India underwent changes due to geopolitical
the development of infrastructure projects. shifts and competition from other European
colonial powers.
MOCK TEST Paper-9 131
11. Option (4) is correct. 15. Option (1) is correct.
Explanation: A famous Nepali cultural symbol Explanation: The yolk sac is a membranous
called Chepu is an adorning divinity. They are sac attached to an embryo, providing early
also called the caretaker of the house as it wards nourishment. It functions as the developmental
off evil energies as per the Nepali culture. circulatory system of the human embryo,
before internal circulation begins. It transfers
12. Option (3) is correct. the nutrients in weeks 2 and 3 when the utero-
Explanation: The Panama Canal serves as placental circulation is established.
a link between the Atlantic Ocean and the 16. Option (4) is correct.
Pacific Ocean. This waterway, situated across
the Panama isthmus, creates a connection Explanation: Ritu Karidhal Srivastava, an
between the two great oceans. Its significance Indian scientist and aerospace engineer, stands
in shaping 20th-century global trade routes as the key figure behind the monumental
is undeniable. Operational since 1914, the Chandrayaan-3 Mission. Widely recognized
canal's history traces back to its initial French as the 'Rocket Woman of India,' Ritu
development efforts in 1881, later entrusted to Karidhal has notably spearheaded significant
the United States in 1904. space endeavors such as Chandrayaan-2,
The canal encompasses engineered features Mangalayaan, and India's Mars Orbiter
like artificial lakes, enhanced and man-made Mission (MOM).
channels, and three sets of locks. Additionally, Ritu Karidhal Srivastava holds a prominent
it incorporates an artificial reservoir, Alajuela position as a senior scientist within the Indian
Lake (known as Madden Lake during the Space Research Organisation (ISRO). Born
American era), which functions as a vital in Lucknow, Uttar Pradesh, she pursued her
component of the canal's water management academic journey at Lucknow University.
system. The canal's existence substantially Karidhal earned her Bachelor of Science degree
reduces the need to circumnavigate the in Physics from the University of Lucknow.
entirety of South America, offering a direct After completing her M.Sc. in Physics from
route that significantly shortens travel the same institution, she embarked on the
distances. university's Physics Department's doctoral
program. Subsequently, she engaged in
13. Option (4) is correct.
teaching within the same department and
Explanation: Zojila Pass: The Zojila Pass is devoted around six months as a research
located in the Ladakh region of Jammu and scholar.
Kashmir. It is the highest motorable pass in the Furthermore, she pursued a master's degree
Himalayas, with an elevation of 11,578 feet. in aerospace engineering at IISc, Bangalore,
Jelep La: The Jelep La is located in the North further highlighting her commitment to
Sikkim district of Sikkim. It is the gateway to advancing her knowledge and expertise in the
the Chumbi Valley, which is a strategic area field.
bordering Tibet. 17. Option (1) is correct.
Khardung La: The Khardung La is located in
the Ladakh region of Jammu and Kashmir. It Explanation: John Harrison invented
is the second highest motorable pass in the “Chronometer”. He was a self-educated english
carpenter and clockmaker. Chronometer is
Himalayas, with an elevation of 18,380 feet. a long-sought-after device for solving the
Bomdi La: The Bomdi La is located in the problem of calculating longitude while at sea.
Tawang district of Arunachal Pradesh. It is His invention help in increasing the safety of
the gateway to the Tawang Valley, which is a long-distance sea travel.
Buddhist pilgrimage site. 18. Option (1) is correct.
Muling La connects Uttarakhand and Tibet
and is situated in the north of Gangotri. Explanation: Gypsum is a mineral seen in
It is situated in the north of Gangotri. It is crystals and masses named gypsum rock.
It is generally learned as Calcium sulphate
situated at an elevation of 5669 m in the Great
dihydrate. Gypsum utilizes in the manufacture
Himalayas. Muling La is also known as Mo-lin of wallboard, cement, Plaster of Paris, etc.
Shank’ou, Muling La, Muling Pass.
19. Option (2) is correct.
14. Option (4) is correct.
Explanation: Hydrogen is not a noble gas. Any
Explanation: The cerebellum is important of the gaseous components helium, Neon,
for making postural adjustments in order to Argon, krypton, Xenon, and Radon, settling
maintain balance. It controls the maintenance Group (zero) (18) of the periodic table, is noble
of posture, balance and equilibrium. gases as they are unreactive.
132 OSWAAL CUET (UG ) 10 Mock Test Papers GENERAL TEST

20. Option (3) is correct. 23. Option (2) is correct.


Explanation: Silambam (Tamil Nadu): Explanation: Length of racing track
Silambam is a traditional martial art form from = x metre
Tamil Nadu. It involves the use of a bamboo According to question
staff as the primary weapon. Silambam
x x
practitioners showcase their skills through   10
100 120
various techniques, strikes, and defensive 6 x  5x
maneuvers with the staff. It's also considered   10
600
a part of Tamil culture and heritage. x
  10
Kalaripayattu (Kerala): Kalaripayattu is one 600
of the oldest known martial arts in the world, ⇒ x = 10 × 600 = 6000 metres
originating from Kerala. It is a comprehensive
= 6 km
system that includes strikes, kicks, grappling,
weaponry, and healing methods. Kalaripayattu
has had a significant influence on many other
24. Option (3) is correct.
martial arts and forms the foundation for Explanation:
various traditional art forms in Kerala. Given: 3rd term of A.P. = 18 and 12th term =
Thang-Ta (Manipur): Thang-Ta is a traditional 72
martial art of Manipur, a state in northeastern Using: nth term of A.P. = a + (n - 1) d
India. It involves both armed combat (using 18 = a + (3 - 1) d
swords and spears) and empty-handed ⇒ a + 2d = 18
techniques. Thang-Ta is deeply rooted in the ⇒ a + 2d = 18 (1)
cultural and historical heritage of Manipur and 72 = a + (12 - 1) d
and is often performed in traditional festivals. ⇒ a + 11 d = 12 (2)
from equation (1) and (2) ⇒
Mardani Khel (Maharashtra): Mardani
a = 6 and d = 6
Khel is a traditional martial art form from
So, required series = 6, 12, 18, ………
Maharashtra, particularly associated with
the warrior community of Marathas. It
25. Option (3) is correct.
emphasizes physical fitness, weaponry skills,
and combat techniques. The name "Mardani Explanation:
Khel" translates to "Manly Games," and it has A 3-digit number = 100x + 10y + z
historical significance in the Maratha history. ∴ Sum of the digits = x + y + z
21. Option (1) is correct. and difference = 100x + 10y + z – (x + y + z)
= 99x + 9y
Explanation: Given: x and y are prime
numbers such that x > y. = 9 (11 x + y)
So, we can say that it is a multiple of 3 and 9
Also, LCM of x and y = 161
∴ xy = 161 = 23 × 7
26. Option (1) is correct
Thus, x = 23, y = 7
Explanation:
Now, 3y – x = 3(7) – 23 = –2
Given,
22. Option (3) is correct. first term a = –10
Explanation: Successive discounts of x% and last term l = 26
y% and number of terms n = 13
 xy  n
x  y  % ∴ Required sum = (a + l)
 100  2
20  10  13
∴ Required discount   20  10  %
= ( 10  26 )
 100  2
= 30 – 2
13
=  16  104
= 28% 2
MOCK TEST Paper-9 133
27. Option (2) is correct. 32. Option (1) is correct.
Explanation: Possible outcomes of the toss Explanation:
are The total production type C and type E cars
HHH, HHT, HTH, THH, TTH, HTT, THT, TTT. in 2013
Total number of outcomes = 8
= 52 + 36 = 88
Out of which, favorable outcomes(for hanif
to lose match) = 6 Total production of type D cars in 2014 and
So, the probability = Favorable outcomes/ 2016 and type E cats in 2017.
Total no. of outcomes = 42 + 46 + 55 = 143
6 88 8
= ∴ Required ratio = = = 8 : 13
8 143 13
3
=
4 33. Option (1) is correct.
Explanation: By using the rule of alligation,
28. Option (3) is correct.
we get
Explanation: Suppose Kamal ate x apples, Mixture I Mixture II
According to the question Darjeeling tea Darjeeling tea

140 14 x 70  10 4 2
x  70   70  x   50
100 10 14 11 7
6
19
29. Option (3) is correct. 6 2 4 6
− −
19 7 11 19
Explanation:
Given, 42 − 38 76 − 66
= =
0.03  0.3  a  0.3  0.3  b 19 × 7 11 × 19

On squaring both sides, we get 4 10


= =
0.03  0.3  a  0.09  0.09  b 19 × 7 11 × 19
a 0.09  0.09 0.0081 So, the required ratio
or    0.9
b 0.03  0.3 0.009 4 10 4 10
= : = :
19 × 7 11 × 19 7 11
30. Option (1) is correct.
= 44 : 70 = 22 : 35
Explanation:
side 34. Option (3) is correct.
In-radius =
2 3 Explanation:
side T
Circum – size decreases =  R 
3 A  P  1  R  T
A  P  1  100 
side side  100
According  3 question,
toRthe
∴ Required ratio = :  
2 3 3 800  P  1  R 3
= =
3 :2 3 1:2 800  P  1  100 
 100  4
 R 
31. Option (2) is correct. and 840  P  1  R  4
and 840  P  1  100 
Explanation: Let the average marks of the  100  (ii)
remaining 30 students be x. Then, On dividing equation (ii) by eq.(i), we get
According to the question, 840 R
 1
800 100
30  85  30  x
65 = 21 R
60  1
20 100
⇒ 65 × 60 = 2550 + 30x
R 21 21  20 1
⇒ 3900 = 2550 + 30x   1 
100 20 20 20
⇒ 30x = 3900 – 2550 = 1350
1
1350  R  100
x= =45 20
⇒ 30 ⇒ R = 5% per annum
134 OSWAAL CUET (UG ) 10 Mock Test Papers GENERAL TEST

35. Option (1) is correct. 1 1


(A +C)’s 1 day’s work will be = +
Explanation: Given: a = 64 and b = 289 30 10
13 4 2
∴= a =64 8 =  
30 30 15
and=b = 289 17
1 15 3
∴ Time taken =   days
 
1

Now, a b  b a
2 20 2 8
3 3
1 Total time = 9   9 days.
 8  17  17  8  2 8 8
1
38. Option (3) is correct.
 25  9  2

1
Explanation: Let the principal and S.I. be 10x
1
 5  32  2 2 and 3x, respectively. Then
S.I.  100 3x  100
36. Option (4) is correct. Time = 
Principal  Rate 10 x  6
Explanation: Given that, 3 100 300
=    5 years
The inner and outer radii of a circular track = 10 6 60
29 metre and 23 metre
Now, we know that, 39. Option (2) is correct.
Area of circular track = π(R2 – r2) Explanation: Suppose the numbers be 5x, 7x
(Where R = outer radius, r = inner radius) and 12x.
22 As per question
= (292 – 232) 5x + 12x = 7x + 50 or 17x – 7x = 50
7
22 or 10x = 50 or x =5
= (29 + 23) (29 – 23)
7 So, the required sum = 5x + 7x + 12x = 24x
22 = 24 × 5 = 120
= × 52 × 6 m2
7
40. Option (2) is correct.
22
∴ cost of levelling the track = × 52 × 6 × 7
7 Explanation:
1 1
= 22 × 52 × 6 = ` 6864 Given: 2 2 + 2 + +
2+ 2 2 −2
37. Option (1) is correct.
 1 1 
Explanation:  2 2  2    
 2  2 2  2
1 1 1 1
First two days work =         
2 2 2 2 
 30 20   30 10   2 2  2 
2 1 1 1 1 1  
 2 2 2 2 
  
=     
30 20 10 15 20 10 −2 2
=2 2+ 2+
4  3  6 13 4 −2
= 
60 60 =2 2+ 2− 2 =2 2

First 8 days work =


52 = 2 × 1.4142 = 2.8284 (Q 2 = 1.4142 )
60
41. Option (3) is correct.
52 60  52 8 2
Remaining work = 1    
60 60 60 15 Explanation:
First fix the position of P and S. Then, arrange
So, (A + B)’s 1 day’s work
the position of other friends.
1 1 23 5
=    S
30 20 60 60
T Q
1
=
12
Right of R
2 1 85 3
Remaining work =   
15 12 60 60
1 U R
=
20 P
MOCK TEST Paper-9 135
42. Option (4) is correct. 48. Option (1) is correct.
Explanation: Explanation:
Given : 25 + 5 × 7 – 12 ÷ 3 = 26
Poor

÷ +
Villgers Males ∴ 25 ÷ 5 × 7 - 12 + 3 = 26

49. Option (4) is correct.


43. Option (4) is correct.
Explanation:
Explanation:
As,
According to the statements,
C D F : G H J
Magicians +4
Doctors
+4
Farmers +4
Same as,

I. () II. () K L N : O P R


Hence, only conclusion I follows. +4
+4
44. Option (1) is correct.
+4
Explanation:
r
the 50. Option (3) is correct.
bro
P R T
daughter Explanation: In the mirror image left becomes
Q m S right and right becomes left. Hence option 3 is
oth father the correct answer.
er
U
51. Option (1) is correct.
Thus, P is the aunt of Q. (where, Q is the
mother of U) Explanation:
According to the given information,
45. Option (3) is correct. E

Explanation: Given conclusions are not the G C


results of the given statements.
Hence, neither conclusion I nor conclusion II
follows. H F

46. Option (1) is correct.


Explanation: A
N
B
NW D
NE
90˚
W E Hence, D is between A and B.
Starting SW SE
point
45˚ S 52. Option (4) is correct.
1 km
Explanation: In the mirror image left becomes
It is clear from the diagram he is now in North-
East direction. right and right becomes left. Hence, option (D)
is the correct answer.
47. Option (2) is correct.
53. Option (2) is correct.
Explanation:
Explanation:
A E I, E I O, I O U, O U A As, 72 : 14 → 7 × 2 = 14
And 96 : 54 → 9 × 6 = 54
Alphabetical order of vowels is A, E, I, O, U Same as, 87 : ? → 8 × 7 = 56
136 OSWAAL CUET (UG ) 10 Mock Test Papers GENERAL TEST

54. Option (4) is correct. 57. Option (1) is correct.


Explanation:
Explanation:
C 5 km B
N
4 km
5 km A W E
F
S
D E
6 km
will complete the given figure. Thus, to reach in starting point we have to run
from F to A, i.e., in West direction.

55. Option (1) is correct. 58. Option (4) is correct.


Explanation:
Explanation: 1 5 9 17 25 37 49 65 81 101

4 4 4 8 8 12 12 16 16 20
5 4
5

2 2 +4 +4 +4 +4
3
1 3
1 59. Option (2) is correct.
Hence, by joining all the parts we can see the Explanation:
correct figure. U
father
L sister Q
H
56. Option (3) is correct. brother
mother
R
Explanation: sister Y
Hence, L is Y’s mother’s sister.
Saksham Father Wife
60. Option (4) is correct.
son
daughter Explanation:
Capital of Afghanistan is Kabul.
Nidhi Similarly,
Capital of China is Beijing.
Hence, Saksham is the father of Nidhi.

SOLUTIONS OF
Question Paper 10
1. Option (4) is correct. to promote and celebrate the linguistic
Explanation: The Allahabad Pillar inscription, diversity of India while recognizing the
also known as the Prayag Prashasti, is an significance of Hindi as one of the country's
inscription in Sanskrit composed by Harisena, official languages.
the court poet and minister of Samudragupta. 3. Option (3) is correct.
The inscription is engraved on a pillar initially
erected by the Mauryan emperor Ashoka in Explanation: Sucheta Kriplani was India's first
woman chief minister. Sucheta Kripalani was
the 3rd century BCE. The inscription describes
a freedom fighter and politician. She was born
the military conquests and achievements of
in Ambala, Punjab, in 1908. She was a close
Samudragupta. associate of Mahatma Gandhi and participated
The inscription mentions that Samudragupta actively in the Indian independence
defeated 9 kings in the north, including the movement. After independence, she served
rulers of the Naga, Yaudheya, and Malava as a Member of Parliament and as the Chief
dynasties. It also mentions that he defeated Minister of Uttar Pradesh. She was a strong
12 kings in the south, including the rulers of advocate for women's rights and social justice.
the Pallava, Pandya, and Chola dynasties. She died in 1974.
In addition, the inscription states that Sarojini Naidu was the first woman Governor
Samudragupta reduced all the Atavika states of the state of Uttar Pradesh. Sarojini Naidu
to vassalage. was a poet, freedom fighter, and politician.
She was born in Hyderabad, Andhra Pradesh,
2. Option (4) is correct. in 1879. She was known as the "Nightingale of
India" for her beautiful poetry. She was also a
Explanation: National Good Governance Day:
prominent figure in the Indian independence
National Good Governance Day is observed in
movement. After independence, she served
India on December 25th every year to mark the as the Governor of Uttar Pradesh and as the
birthday of former Prime Minister Atal Bihari Governor of West Bengal. She died in 1949.
Vajpayee. The day aims to promote awareness
about the importance of good governance and 4. Option (2) is correct.
its role in building a better society.
Explanation: In 1953, Vijaya Lakshmi Pandit
Armed Forces Flag Day: Armed Forces Flag became the first woman President of the
Day is observed on December 7th every year United Nations General Assembly. She was
in India. The day is dedicated to honouring India’s ambassador to Ireland from 1955 to
and supporting the Indian Armed Forces 1961 (during this time she was also the Indian
personnel and veterans. It is an opportunity High Commissioner to the United Kingdom).
for citizens to contribute to the Armed Forces
Flag Day Fund, which provides welfare 5. Option (3) is correct.
assistance to military personnel. Explanation: An image scanner (or scanner) is
Vijay Diwas: Vijay Diwas, also known as a device that scans images optically. Convert
Victory Day, is celebrated on December 16th typed text, handwriting or objects into digital
in India. It commemorates the victory of the images. A scanner is an input devices. Scanned
Indian Armed Forces over Pakistan during the images can be saved in various file formats
Indo-Pakistani War of 1971. The war resulted such as JPEG and PDF.
in the liberation of Bangladesh (then East 6. Option (2) is correct.
Pakistan) and the surrender of Pakistani forces
in Dhaka. Explanation: The router is in the third
layer, the network layers. Used to connect
Hindi Day: Hindi Day, also known as Hindi networks together. The Internet is made up
Diwas, is celebrated on September 14th every of many interconnected routers. Routers can
year in India. It marks the adoption of the intelligently move data from one network to
Hindi language as the official language of the another using network protocols such as TCP/
Indian Republic in 1949. This day is observed IP.
138 OSWAAL CUET (UG ) 10 Mock Test Papers GENERAL TEST

7. Option (1) is correct. Durgapur Steel Plant, in West Bengal, was set
Explanation: The Battle of Ghaghra, fought up in collaboration with the government of
in 1529, was a major battle for the conquest of the United Kingdom and started production
India by the Mughal Empire. It followed the in 1962.
first Battle of Panipat in 1526 and the Battle Bokaro Steel Plant was set up in 1964 at Bokaro
of Khanwa in 1527. The forces of Babur, now with Russian collaboration.
Emperor Zahir ud-Din Muhammad Babur,
10. Option (4) is correct.
were joined by Indian allies in the battle
against the Eastern Afghan Confederates Explanation: The Wind turbines remake the
under Sultan Mahmud Lodi and the Sultanate kinetic energy into mechanical energy. Kinetic
of Bengal under Sultan Nusrat Shah. energy is the sort of moving energy. The edges
The Battle of Khatauli was fought in 1518 spin a shaft that is linked to a generator that
between the Lodi dynasty under Ibrahim Lodi produces electricity.
and the Kingdom of Mewar under Rana Sanga,
11. Option (3) is correct.
during which Mewar emerged victoriously.
The Battle of Gagron was fought in 1519 Explanation: The thymus gland is located
between Sultan Mahmud Khalji II of Malwa between our lungs and behind our sternum.
and Rana Sanga of Mewar. The conflict The pituitary gland (master gland) is located
occurred in Gagron and resulted in Sanga's in the brain between the pineal gland and
victory, with him taking Mahmud captive and hypothalamus. The hypothalamus is also
annexing significant territory. located in brain between the pituitary gland
The Third Battle of Panipat took place in and thalamus.
1761 between the Maratha Empire and the
12. Option (2) is correct.
Durrani Empire led by Ahmad Shah Durrani
(also known as Ahmad Shah Abdali). The Explanation: Taiga ecosystem is an example of
battle resulted in a significant defeat for the a forest ecosystem. Taiga is generally referred
Marathas and had far-reaching consequences to in North America as a boreal forest or snow
on the political landscape of India. forest, is a biome characterized by coniferous
forests consisting mostly of pines, spruces, and
8. Option (3) is correct.
larches. The taiga or boreal forest has been
Explanation: The Mughal emperor called the world’s largest land biome.
commemorated with a mosque bearing his
name in Fatehabad is Humayun. The mosque
13. Option (4) is correct.
is known as the Humayun's Mosque or the Explanation: The Indian regions of West Bengal,
Adhai Din Ka Jhonpra. It was built in 1530 by Arunachal Pradesh, Sikkim, and Assam share
Humayun, the second Mughal emperor, to their borders with Bhutan. The demarcation
commemorate his victory over the Afghans at line spanning 699 kilometres serves as the
the Battle of Chausa. The mosque is located in regional frontier that separates the Kingdom
the town of Fatehabad, in the Indian state of of Bhutan from the Republic of India. This
Haryana. The Humayun's Mosque is a two- border acts as the sole terrestrial connection for
storeyed structure with a square plan. The Bhutan, as its boundary with China remains
lower storey is made of red sandstone and completely sealed. The establishment of the
the upper storey is made of white marble. The Bilateral Border Management and Security
mosque has three domes and four minarets. for the Indian-Bhutan Group was undertaken
The interior of the mosque is decorated to jointly evaluate and safeguard the shared
with intricate carvings and inscriptions. The border between the two nations.
Humayun's Mosque is one of the earliest 14. Option (2) is correct.
surviving examples of Mughal architecture. It
is a UNESCO World Heritage Site. Explanation: The Society for Worldwide
Interbank Financial Telecommunication
9. Option (4) is correct. (SWIFT), legally S.W.I.F.T. SC. SWIFT is the
Explanation: The Rourkela Steel Plant was world’s largest electronic payment system
set up in 1959 in the Sundargarh district that facilities the exchange of messages
of Odisha in collaboration with Germany. between banks and financial institutions. Most
The Bhilai Steel Plant was established with international interbank communications use
Russian collaboration in the Durg District of the SWIFT network. It is headquartered in La
Chhattisgarh and started production in 1959. Hulpe, Belgium.
MOCK TEST Paper-10 139
15. Option (4) is correct. Mycoplasmas are the smallest free-living
Explanation: Indian Prime Minister Narendra bacteria known to exist. They do not have a
Modi was awarded the highest civilian honour cell wall, which makes them resistant to many
antibiotics, including penicillin. They are also
Grand Cross of the Order of Honor by Greek
very flexible and can change shape, which
President Katerina N. Sakellaropoulou in
makes them difficult to target with antibiotics.
Athens. The award is the highest civilian
Mycoplasmas can cause a variety of diseases,
honour Greece can bestow on a foreign head of including pneumonia, urethritis, and
government. The award recognizes PM Modi's meningitis. They are also a common cause of
outstanding contribution to strengthening infections in people with weakened immune
bilateral relations between Greece and systems.
India. The citation given along with the
award highlights PM Modi's commitment to 18. Option (4) is correct.
enhancing political, economic, cultural, and Explanation: Corundum is one of the
people-to-people relations between the two Aluminum. It is the crystalline state of
countries. He is also the first foreign head of aluminum oxide. After Diamond, it is the
government to be honoured with the award hardest comprehended biological substance.
since 2017. The award reflects PM Modi's Its more delicate types are gemstones,
efforts to strengthen ties between India and sapphire, and ruby.
Greece, which share a history of friendship
and cooperation.
19. Option (2) is correct.
Explanation: The headquarters of the
16. Option (1) is correct. International Olympic Committee is located in
Explanation: Pascal (Pa): The Pascal is the SI Lousanne, Switzerland. IOC was founded by
unit of pressure. It is defined as the force of Pierre de Coubertin and Demetrios Vikelas on
one newton per square meter. Pressure is force 23 June 1894. The role of IOC is to supervise,
per unit area. For example, the pressure of support and monitor the organisation of the
the atmosphere at sea level is about 101,325 games.
pascals.
20. Option (2) is correct.
Joule (J): The joule is the SI unit of work. It
is defined as the energy expended when the Explanation: Mahuri is a traditional musical
force of one newton acts through a distance instrument belonging from the state of
of one meter. Work is the product of force and Odisha. It is an auspicious instrument. It is
distance. For example, the work done in lifting often part of the music arrangements during
a 10-kilogram object at a height of 1 meter is marriages. It is often found in folk theatres like
100 joules. Rama Nataka and dances like Chhau too.
Fahrenheit (°F): The Fahrenheit scale is a
21. Option (4) is correct.
temperature scale that is commonly used in
the United States and Canada. It is defined as Explanation: Let the marked price of the article
the temperature at which the freezing point of = ` x.
water is 32 degrees Fahrenheit and the boiling According to the question,
point of water is 212 degrees Fahrenheit. 80 85
x   3060
Lightyear (ly): The lightyear is a unit of 100 100
distance that is equal to the distance that light
3060 × 100 × 100
travels in one year. It is about 9.46 trillion ⇒x= = ` 4500
kilometres. 80 × 85

17. Option (2) is correct. 22. Option (3) is correct.


Explanation: The bacteria that exhibit Explanation:
penicillin resistance due to its absence of a cell  R 
T

A = P1+
wall is Mycoplasmas. Spirochetes are bacteria  100 
that have spiral-shaped cells. They are motile T
 7 
and can move through fluids by rotating their 30000 + 4347 = 30000  1 +
 100 
bodies. They are not resistant to penicillin. T
34347  107 
Cyanobacteria are photosynthetic bacteria ⇒ =
that have a cell wall. They are not resistant to 30000  100 
2 T
penicillin. Bdellovibrios are parasitic bacteria 11449  107   107 
⇒ =  = 
that attack other bacteria. They inject their 10000  100   100 
DNA into the host cell and take over its On comparing, we get
functions. They are not resistant to penicillin.
T = 2 years
140 OSWAAL CUET (UG ) 10 Mock Test Papers GENERAL TEST

23. Option (4) is correct. 27. Option (2) is correct.


Explanation: Sum of given series Explanation: Let B’s income be ` 100.

 1  2  3 ∴ A’s income = ` 125


  1     1     1    ... till the term
 n  n  n So, the required percent  125  100  100
125
1 2 3 2500
= (1+1+1.... till n terms)      ... = = 20%
n n n 125
n terms
28. Option (1) is correct.
1
= n  (1  2  3  ... n terms) Explanation:
n
Given that, in ∆ABC,
1 n ( n  1) PQ || BC,
=n 
n 2
Now, ∠APQ = ∠ABC and ∠AQC = ∠ACB
( n  1) A
 n
2
n 1 P Q

2
B C
24. Option (2) is correct. ∴ from AA – Similarity theorem
Explanation: Let the distance be x km. ∆APQ ~ ∆ABC
So, according to the question, AB AC 5
  
x x AP AQ 3
 3
62 62
AC 5
or 1 1
x x AQ 3
or  3
4 8 AC  AQ 5  3
or 
2x  x AQ 3
or 3
8
QC 2
or x = 3 × 8 = 24 km or =
AQ 3
AQ 3
25. Option (4) is correct. or =
QC 2
Explanation: or AQ : QC = 3 : 2
x2 – y2 = 80 29. Option (4) is correct.
x–y=8
Explanation:
x 2  y 2 80 Given that,
x  y    10
x y 8 ∆ABC is equilateral triangle
Required average A
x + y 10
= = =5
2 2

26. Option (3) is correct. B D C


Explanation: 2
Side AB = cm
Total production of type A cars in 2015 and 3
type B cars in 2014 = (56 + 48) thousand BC 2
=
BD =
= 104 thousand 2 2 3
Total production of type C cars in 2017 and 1
= cm.
type E cars in 2018 = (57 + 60) thousand 3
= 117 thousand Using Pythagoras theorem, we have
∴ Required ratio = 104 : 117 4 1 3
AD = AB 2  BD 2     1 cm.
=8:9 3 3 3
MOCK TEST Paper-10 141
30. Option (4) is correct. 36. Option (3) is correct.
Explanation: We have Explanation: Given that,
tn+2 = tn + tn+1 Radius of wheel = 14 cm
No. of revolutions = 15
If n = 1, then t3 = t1 + t2 = 3
We know that,
If n = 2, then t4 = t2 + t3 = 3 + 2 = 5 Distance covered by wheel in one
If n = 3, then t5 = t3 + t1 = 3 + 5 = 8 Revolution = circumference of wheel = 2πr
31. Option (3) is correct. 22
or 2 × × 14 = 88 cm
Explanation: Here, N = 15, T = 42, t = 1.6 7
∴ Weight of new oarsman = (Nt + T) kg Distance covered in 15 revolutions = 15 × 88
= (42 + 15 × 1.6) kg = 1320 cm
= (42 + 24) kg 37. Option (2) is correct.
= 66 kg
Explanation: Given that,
32. Option (2) is correct. P S

Explanation: Let the number be x


O
So x + 12 = 112 or x + 12 = 672
6
or x = 672 – 12 = 660
Q R
660
∴ Right answer = + 12 = 110 + 12 = 122
6 Square (PQRS) side = 20 cm
Now, PR = diagonal of square
33. Option (4) is correct.
PR = 2 × side
Explanation:
or PR = 2 ×20 = 20 2 cm
Let the C.P. of Cloth = ` x per metre
∴ Required Answer = (PQ + QO + OP) +
According to the question, (QR + RO + OQ) +
90 900 (SR + RO + OS) + (SP + SO + OP)
x 9x  10
100 90 = 4PQ + 4PR
To gain 15%,
 
 4  20  4  20 2 cm
 10  115 
S.P. = `   = ` 11.50 per metre.   80  80 2  cm
 100 
34. Option (1) is correct. 38. Option (2) is correct.
Explanation: Work done by Ram and Shyam
Explanation: Given: 3+ 2
in 4 days
3− 2
3 1
On rationalising the denominator, we get = 4   
 3 2  3 2  20 15 
    
9  4  4  13 13
 3 2  3 2 = 4    60  15
 60 
 
2
3 2
  13 2
2
  3 2 Remaining work = 1  
32 15 15
3+ 2 Thus, Shyam’s complete the work
So, = ( 3 + 2 )2 = 3 + 2
3− 2 2
=  15  2 days
15
35. Option (1) is correct.
Explanation: Decrease in time = 25% = x
39. Option (1) is correct.
So percentage increase in speed Explanation: According to the question,
x S.I. for 2 years 6 months
= × 100
100 − x = (`5500 – `4000) = `1500
25 × 100 100 1 5
= = = 33 % S.I. for years = `1500
75 3 3 2
142 OSWAAL CUET (UG ) 10 Mock Test Papers GENERAL TEST

1500  2 3000
44. Option (4) is correct.
S.I. for 1 year =  = `600
5 5 Explanation:
∴ S.I. for 2 years = `1200
Also, Principal = `(4000 – 1200) = `2800
2 3 3 2
S.I.  100 1200  100 1
Now, Rate =  ⇒
Principal  Time 2800  2 4
120000 1 4
=
5600
150 3 Hence, by joining all the parts we can see the
= = 21 % per annum
7 7 correct figure.
40. Option (1) is correct. 45. Option (3) is correct.
Explanation: Suppose the numbers of boys be Explanation: Angle traced by hour hand in 12
27x. hours = 360°
and number of girls be 23x. Angle traced by hour hand in 6 hours
As per question, 20 minutes i.e. 19/3 hours = (360° ÷ 12) ×
(19 ÷ 3) =190°
27x – 23x = 200
200 Hence, the correct answer is 190°.
or 4x = 200 or x = or x = 50
4
So, the number of boys = 27 × 50 = 1350
46. Option (4) is correct.
Explanation:
41. Option (1) is correct. So,
Explanation:
Given: 15 + 15 × 15 – 15 ÷ 15 = 15

÷ – will complete the given figure.


⇒ 15 + 15 × 15 ÷ 15 - 15 = 15 47. Option (2) is correct.
42. Option (1) is correct. Explanation:
Father
Explanation:

Indians
Mother Vikram

Females Voters

Kumar son

43. Option (3) is correct. ∴ Kumar is the son of Vikram.


Explanation: 48. Option (1) is correct.
According to the statements,
Explanation: As per the question, today is
Monday.
Bottles Glasses Bowls or We have to find the day after 62 days that is
the 63rd day.
The numbers of odd days in 62 days are
Bottles
• 56/7
Glasses
• 56 days + 6 odd days
The day on the 56th day is Monday.
Bowls The day in 62nd day will be
Monday + 6 days
Sunday
I. () II. () III. ()
So, the 63rd day will be
Hence, conclusion II and either conclusion I or Sunday +1 day ⇒ Monday.
III follow. Hence, option (1) is the correct answer.
MOCK TEST Paper-10 143
49. Option (4) is correct. 55. Option (3) is correct.
Explanation: Conclusion I can be proved right Explanation:
using statement I and statement II, but facts
3 18 48 93 153
given in conclusion II does not follow any of
the given statements.
15 30 45 60
Hence, only conclusion I follows.
50. Option (2) is correct.
15×1 15×2 15×3 15×4
Explanation:
As, WOLF → FLOW 56. Option (2) is correct.
Same as, DRAW → WARD Explanation:
Note: Here, alphabets of given words are
YXP
written in reverse order.
ZQR
51. Option (2) is correct. PZ
Explanation: A  sing (i), (ii) and (iii), sitting arrangement is as
U
follows :
P L A C A R D T P E Y E V H
+4
+4
+4 Y X P Z Q R
–4 Thus, Y and R are sitting on an extreme ends.
+4
+4 57. Option (2) is correct.
+4
Similarly, Explanation:
M O N S T E R Q S R O X I V J N R V Z

+4 +4 +4 +4 +4
+4
–4 58. Option (1) is correct.
+4
+4 Explanation:
+4 brother
P K S mo
52. Option (3) is correct. the
r
daughter
Explanation: brother
Y J R
According to the given information, sister
R Thus, from the figure, J is not the daughter of P.
P Q
59. Option (4) is correct.
Explanation:
A year includes months.
S Similarly,
T
Hence, P is sitting second to the left of T. An hour includes minutes.
53. Option (2) is correct. 60. Option (2) is correct.
Explanation:
Explanation:
As, 6 : 252 → (6)3 + (6)2 = 216 + 36
= 252 C N
Same as, NW
NE
5 : 150 → (5)3 + (5)2 = 125 + 25
= 150 A B W E
54. Option (2) is correct. SW SE
S
Explanation: D
D A C B
2 km 2 km 6 km From figure, D is in South-East direction with
respect to C.
10 km
∴ Distance between C and B = 6 km. 

WRITING YOUR NOTES

Just in case you have forgotten today, takedown your notes!


But why is it so important?
Tools for the hands are tools for the brain writes Hetty Roessingh.
Handwritten notes are a powerful tool for encrypting embodied cognition and in turn supporting the
brain’s capacity for recuperation of information. If that sounds so scientific then in simple words:
Writing notes by hand help you in:
Increasing your comprehension Strengthening your memory Igniting your creativity
Engaging your mind Increasing your attention span
Are these reasons enough to get you started?

1. .........................................................................................................................................
2. .........................................................................................................................................

3. .........................................................................................................................................

4. .........................................................................................................................................

5. .........................................................................................................................................

6. .........................................................................................................................................

7. .........................................................................................................................................

8. .........................................................................................................................................

9. .........................................................................................................................................

10. ..........................................................................................................................................

11. ..........................................................................................................................................

12. ..........................................................................................................................................

13. ..........................................................................................................................................

14. ..........................................................................................................................................

15. ..........................................................................................................................................

16. ..........................................................................................................................................

17. ..........................................................................................................................................

18. ..........................................................................................................................................

19. ..........................................................................................................................................

20. ..........................................................................................................................................
Attention ambitious
achievers!

Prepare to conquer the CUET(UG)


exam with Oswaal360!
Unlock the path to success with double the value of
your purchase and gain access to:

Captivating Challenging Realistic Mock


Video Lectures Practice Tests Tests

Don't miss out on our upcoming series of Exclusive


Mock Tests, designed to boost your confidence and
ensure success. Prepare for other competitive exams
like NEET, JEE, and more, and pave the way to a bright future.

Scan now and Unlock the


door to your dream college
with Oswaal360 by your side! Share your review of the
CUETGT books and help your fellow
students & juniors. Scan to Review
General Test
Attention ambitious
achievers!

Prepare to conquer the CUET(UG)


exam with Oswaal360!
Unlock the path to success with double the value of
your purchase and gain access to:

Captivating Challenging Realistic Mock


Video Lectures Practice Tests Tests

Don't miss out on our upcoming series of Exclusive


Mock Tests, designed to boost your confidence and
ensure success. Prepare for other competitive exams
like NEET, JEE, and more, and pave the way to a bright future.

Scan now and Unlock the


door to your dream college
with Oswaal360 by your side! Share your review of the
CUETGT books and help your fellow
students & juniors. Scan to Review
General Test

You might also like